Re: [obm-l] Probabilidade

2009-08-03 Por tôpico Ralph Teixeira
Ha apenas 3 tipos de casas: canto (A), centro do lado (B) e o centro em si (C).

Se a peca estah em C, ela tem 100% de chance de ir para B.
Se a peca estah em A, ela tem 100% de chance de ir para B.
Enfim, se a peca estah em B, ela tem 1/3 de chance de ir para C e 2/3
de ir para A.

SOLUCAO 1:
A matriz de transicao eh portanto P=[0 1 0; 2/3 0 1/3; 0 1 0].
Queremos calcular P^10. Voce pode diagonalizar, ou entao fazer no
braco... Um truque eh notar que P^3=P, portanto
P^10=P^8=P^4=...=P^2=[2/3 0 1/3; 0 1 0; 2/3 0 1/3]. Assim, a chance de
voltar ao centro em 10 movimentos eh o numero inferior aa direita,
1/3.

SOLUCAO 2:
Usando a notacao acima, um trajeto serah do tipo CBABABCBABC, digamos,
com 11 caracteres. Note que todos os caracteres de posicao impar tem
de ser B! Afinal, de B voce TEM QUE IR para A ou C; e de A ou C voce
TEM QUE IR para B.

Entao esqueca os 9 primeiros movimentos; com certeza apos o nono
movimento, temos que estar numa casa do tipo B, e os 9 primeiros
movimentos nao interessam! Agora, a partir dali de B, a probabilidade
de a proxima casa (apos o 10o movimento) ser C eh 1/3. Entao acabou,
esta eh a resposta!

Abraco, Ralph.

2009/8/3 Bluesman bluesman2...@uol.com.br:
 Olá,

 Alguma idéia sobre a solução do problema abaixo?

 Um tabuleiro quadrado tem nove casas. Uma peça sobre o tabuleiro pode
 mover-se para as casas lateral esquerda, lateral direita, lateral acima ou
 lateral abaixo, se não for obstruída em um ou dois destes movimentos estando
 sobre a borda do tabuleiro. Considere que a peça inicialmente está no centro
 do tabuleiro e é movida aleatoriamente na superfície deste. Determine a
 probabilidade de que, após 10 movimentos, a peça esteja de volta ao centro.

 Obrigado e um abraço.

=
Instruções para entrar na lista, sair da lista e usar a lista em
http://www.mat.puc-rio.br/~obmlistas/obm-l.html
=


[obm-l] Re: [obm-l] O Problema Impossível

2009-08-03 Por tôpico Ralph Teixeira
Estou meio sem tempo agora, entao vou colocar minha versao do problema
e minha versao da solucao, que acho mais clara do que a maioria que eu
vi por ai (apesar de ser trabalhosa de explicar) Tenho certeza que
esta versao eh facilmente adaptavel a outros problemas do tipo, e vira
um algoritmo para botar no computador no caso finito.

Abraco, Ralph.

---///---

ENUNCIADO: Marcos pensa dois números naturais positivos (possivelmente
iguais); a soma destes dois números ele entrega ao matemático Alcides,
enquanto o produto ele entrega à matemática Beatriz. Alcides e Beatriz
conversam sem mostrar seus números um ao outro:
A: “Só com esta soma, eu não sei quais são eram os dois números originais”.
B: “Com este produto, também não dá para descobrir”.
A: “Mesmo assim, continuo sem saber quem eram os números originais”.
B: “E eu continuo sem saber quem eles eram”.
A: “Ah! Neste caso, eu sei quem eram os números originais!”.
B: “Se você agora sabe, então eu também sei!”.
Quais eram os números originais?

SOLUCAO:
 Sejam m e n os números que Marcos pensou (digamos, m=n). O
objetivo eh montar uma tabela com todas” as possibilidades para m e
n, e todas as possiveis conversas em cada caso. Assim, considere a
conversa que Alcides teria com Beatriz em cada caso (denotaremos por
“s” a frase “Eu sei quem são os números m e n” e por “n” a frase
“Ainda não sei quem são m e n”.).
 Alcides vê a soma. Se ela for 2 ou 3, Alcides sabe os números (1
e 1 no primeiro caso e 1 e 2 no segundo). Qualquer outra soma faz com
que Alcides fique em dúvida, pois qualquer soma maior que 4 pode ser
alcançada de várias maneiras. A conversa começa então assim em cada
hipotese:

n\m 1   2   3   4   5   6   7   8
1 s s   n   n   n   n   n   n
2   n   n   n   n   n   n   n
3   n   n   n   n   n   n
4   n   n   n   n   n
5   n   n   n   n
6   n   n   n
7   n   n
8   n
A seguir, Beatriz olha o produto. Se o produto for 1, 2 ou um número
primo p, Beatriz saberá que os números são 1 e 1, 1 e 2 ou 1 e p,
respectivamente. Caso contrário, Beatriz ainda é incapaz de decidir
qualquer coisa, pois haverá várias opções e todas elas vêm com um “n”
do Alcides. A tabela da conversa fica assim:

n\m  1  2   3   4   5   6   7   8
1  ss   ss  ns  nn  ns  nn  ns  nn
2   nn  nn  nn  nn  nn  nn  nn
3   nn  nn  nn  nn  nn  nn
4   nn  nn  nn  nn  nn
5   nn  nn  nn  nn
6   nn  nn  nn
7   nn  nn
8   nn

Como continuar a tabela? Nos casos “ss”, a conversa acabou. Nos casos
“ns”, Alcides será capaz de deduzir que os números são 1 e p, e
portanto descobri-los a partir da soma que tem em mãos (assim, os
casos correspondentes a 1 e p com p primo maior do que 2 levam à
conversa “nss”). Enfim, em todos os outros casos do tipo “nn” Alcides
vê a soma que tem em mãos e ainda é incapaz de decidir em que “casa”
ele está na tabela acima – com uma exceção: se a soma fosse 4, Alcides
decidiria que (m,n)=(1,3) ou (m,n)=(2,2), e seria capaz de distinguir
entre as duas opções baseado na conversa até aqui (que, no primeiro
caso, é “ns”, mas, no segundo, é “ss”). Assim, com a resposta de
Alcides, a tabela se torna:

n\m  1  2   3   4   5   6   7   8
1  ss   ss  nss nnn nss nnn nss nnn
2   nns nnn nnn nnn nnn nnn nnn
3   nnn nnn nnn nnn nnn nnn
4   nnn nnn nnn nnn nnn
5   nnn nnn nnn nnn
6   nnn nnn nnn
7   nnn nnn
8   nnn

Agora é a vez de Beatriz. Se o produto for 1, 2 ou primo, já sabemos o
que acontece. Se Beatriz tem algum outro produto em mãos, ela tem de
considerar várias hipóteses, e, como mostra a tabela acima, em todas
elas a conversa foi “nnn” até aqui – com uma exceção: se o produto for
4, Beatriz tem como decidir entre as únicas possibilidades (m,n)=(2,2)
ou (1,4) observando a tabela acima (que indica a conversa “nns” no
primeiro caso e “nnn” no segundo). Então, temos:

n\m12   3   4   5   6   7   8
1ss ss  nss nnnsnss nss 
2   nnss
3   

Re: [obm-l] Probabilidade

2009-08-03 Por tôpico Ralph Teixeira
1. Sim, a configuracao eh esta mesma.
2. Uso chance no sentido de probabilidade mesmo, direto. Se a peca
estah na casa B, a probabilidade de ela ir dali para a casa C eh 1/3,
e a probabilidade de ir para A eh 2/3.
3. Entendi do enunciado que movimentos diagonais nao valiam (entendi
lateral como movimento para uma casa com um LADO em comum com esta
entao lateral acima significaria apenas para cima, de fato).

Abraco, Ralph.

2009/8/3 Bluesman bluesman2...@uol.com.br:
 Olá Ralph,

 Muito bom, a resposta está correta. Continuo apenas com as seguintes
 dúvidas:

 1) Não entendi a configuração das casas. Seria a seguinte?

 ABA
 BCB
 ABA
 2) Você está usando a palavra chance em que sentido?
 Chance de ocorrência de um evento é a razão entre a probabilidade de sua
 ocorrência e a probabilidade de sua não ocorrência.

 3) Os movimentos possíveis são apenas os laterais e diagonais (acima ou
 abaixo), quando possíveis? Neste caso, os cantos teriam cada um dois
 movimentos possíveis; as duas casas acima e abaixo do centro, 4 movimentos
 possíveis; o centro seis movimentos possíveis e as duas casas restantes 3
 movimentos cada?

 Grato de antemão e parabéns pela brilhante solução.


 - Original Message - From: Ralph Teixeira ralp...@gmail.com
 To: obm-l@mat.puc-rio.br
 Sent: Monday, August 03, 2009 4:49 PM
 Subject: Re: [obm-l] Probabilidade


 Ha apenas 3 tipos de casas: canto (A), centro do lado (B) e o centro em si
 (C).

 Se a peca estah em C, ela tem 100% de chance de ir para B.
 Se a peca estah em A, ela tem 100% de chance de ir para B.
 Enfim, se a peca estah em B, ela tem 1/3 de chance de ir para C e 2/3
 de ir para A.

 SOLUCAO 1:
 A matriz de transicao eh portanto P=[0 1 0; 2/3 0 1/3; 0 1 0].
 Queremos calcular P^10. Voce pode diagonalizar, ou entao fazer no
 braco... Um truque eh notar que P^3=P, portanto
 P^10=P^8=P^4=...=P^2=[2/3 0 1/3; 0 1 0; 2/3 0 1/3]. Assim, a chance de
 voltar ao centro em 10 movimentos eh o numero inferior aa direita,
 1/3.

 SOLUCAO 2:
 Usando a notacao acima, um trajeto serah do tipo CBABABCBABC, digamos,
 com 11 caracteres. Note que todos os caracteres de posicao impar tem
 de ser B! Afinal, de B voce TEM QUE IR para A ou C; e de A ou C voce
 TEM QUE IR para B.

 Entao esqueca os 9 primeiros movimentos; com certeza apos o nono
 movimento, temos que estar numa casa do tipo B, e os 9 primeiros
 movimentos nao interessam! Agora, a partir dali de B, a probabilidade
 de a proxima casa (apos o 10o movimento) ser C eh 1/3. Entao acabou,
 esta eh a resposta!

 Abraco, Ralph.

 2009/8/3 Bluesman bluesman2...@uol.com.br:

 Olá,

 Alguma idéia sobre a solução do problema abaixo?

 Um tabuleiro quadrado tem nove casas. Uma peça sobre o tabuleiro pode
 mover-se para as casas lateral esquerda, lateral direita, lateral acima ou
 lateral abaixo, se não for obstruída em um ou dois destes movimentos
 estando
 sobre a borda do tabuleiro. Considere que a peça inicialmente está no
 centro
 do tabuleiro e é movida aleatoriamente na superfície deste. Determine a
 probabilidade de que, após 10 movimentos, a peça esteja de volta ao
 centro.

 Obrigado e um abraço.

 =
 Instruções para entrar na lista, sair da lista e usar a lista em
 http://www.mat.puc-rio.br/~obmlistas/obm-l.html
 =
 =
 Instruções para entrar na lista, sair da lista e usar a lista em
 http://www.mat.puc-rio.br/~obmlistas/obm-l.html
 =


=
Instruções para entrar na lista, sair da lista e usar a lista em
http://www.mat.puc-rio.br/~obmlistas/obm-l.html
=


Re: Res: [obm-l] Juros compostos

2009-08-03 Por tôpico Ralph Teixeira
To contigo, Nehab. Acabei de dar aula disso semana passada; quando
voce tem uma taxa nominal de 60% capitalizada semestralmente, nao
tem porcaria nenhuma subindo 60%. Este 60% nao significa absolutamente
nada, eh uma pessima maneira de descrever o que estah acontecendo,
maneira que infelizmente o sistema financeiro insiste em usar.

Po, vou mais longe -- eu nao gosto nem de usar as **taxas** efetivas,
isto eh, nao gosto de usar percentagens para medir aumento ou reducao!
Por mim, quando eu montar a minha colonia em Marte, a lei vai ser: os
bancos divulgam os MULTIPLICADORES (mensais ou anuais ou sei lah).
Entao meu banco vai dizer Te damos um multiplicador anual de 1.09 ao
inves de Pagamos 9% ao ano. Com essa lei, ninguem nunca mais vai
errar achando que dois aumentos de 9% dao um aumento de 18%...

Em Marte, tudo vai ser muito bom. Enquanto isso... :)

Abraco, Ralph.

09/8/4 Carlos Nehab ne...@infolink.com.br:
 Oi, Lafayette,

 Seja muito benvindo à lista...

 Como você tocou num ponto que eu, pessoalmente, odeio e acho que os
 matemágicos em geral têm a maior dificuldade de engolir (inclusive eu), só
 quero complementar: a galera que milita na área de juros prá cá e juros prá
 lá adora usar os termos taxa nominal e taxa efetiva, e é isso que você
 apropriadamente mencionou, qdo falou em jeito de escrever...

 Ou seja, se escreve pensando em juros simples (taxa nominal - que na minha
 opinião é uma enganação) mas se faz a conta com juros compostos (taxa
 efetiva)...

 Vou contar um caso quase engraçado:

 Há MUITOS anos (muitos MESMO) um amigo pediu prá eu compor uma planilhinha
 que calculasse os pagamentos de prestações de imóveis (para quitação de
 prestaç~eos e parcelas intermediárias de forma antecipada ou com atrasos) e
 saldos dai decorrentes, usando a maldita taxa de 12% ao ano (mais correção)
 e eu comentei que mais dia menos dia ele ia encontrar um advogado (ou
 matemático) pentelho que jamais concordaria com esta terminologia.  Num deu
 outra.   Um advogado o acionou pois argumentou que no contrato estava
 escrito apenas juros de 12% ao ano e, portanto, os juros (ao mês) deviam ser
 tais que  (1+t)^n = 1,12..., ou seja, ele queria reajuste mensal de
 1,12^(1/12)-1 = 0,9489%... e não de 1%.

 Ai, ele (o amigo, naturalmente) me perguntou o que eu achava e minha
 resposta foi cretina, mas honesta com meu ponto de vista: eu disse que em
 minha modestíssima opinião, eu concordava com o advogado...  E quase perdi o
 amigo...

 Finalizo, polemizando (é claro): esta terminologia é ou não é completamente
 cretina?

 Abraços :-D
 Nehab

 Lafayette Jota escreveu:

 Vamos lá:

 Primeiro, o uso do termo taxa de 60% a.a. com capitalização trimestral.

 Neste caso, começe notando que a aplicação não rende 60% a.a. Isto é apenas
 um jeito de escrever.
 Quando a palavra capitalização + prazo é usada, a taxa informada é
 simplesmente uma taxa calculada como se fosse a juros simples.

 Traduzindo então:
 1. Taxa de 60% a.a. com capitalização trimestral = rendimento composto de
 60/4 = 15% ao trimestre.

 Compondo ao final de um ano, temos (1,152)^4 = 1,74900625
 (74,90% a.a)

 2. Taxa de 68% a.a. com capitalização semestral = rendimento composto de
 68/2 = 34% ao semestre.
 Compondo ao final de um ano, temos (1,34)^2 = 1,7956
 (79,56% a.a.)


 3. Taxa de 72% a.a com capitalização mensal = rendimento composto de 72/12 =
 6% ao mês.
 Compondo ao final de um ano, temos (1,06)^12 = 2,012196471835550329409536

 (101,22% a.a.)
 Supondo os capitais A, B, C

 A soma dos montantes é:
 1,749A + 1,7956B + 2,0122C = 194.871,60

 Mas A = 2B
 B = C/4  C = 4B

 Então
 1,749*2B + 1,7956B + 2,0122*4B = 194.871,60
 3,498B + 1,7956B + 8,0488B = 194.871,60

 B = 14605,44
 A = 29210,88
 C = 58421,75

 Resultados aproximados. E que resultados feios, viu!

 Grande abraço,
 Lafayette - novo aqui na lista.

 
 De: Marcus marcusaureli...@globo.com
 Para: obm-l@mat.puc-rio.br
 Enviadas: Segunda-feira, 3 de Agosto de 2009 17:51:31
 Assunto: [obm-l] Juros compostos

 Alguém me ajuda nessa questão?

 Um investidor aplicou três capitais no regime de capitalização composta
 durante 1 ano. O primeiro a 60 % ao ano com capitalização trimestral; o
 segundo a

 68 % ao ano, com capitalização semestral; e o terceiro a 72 % ao ano, com
 capitalização mensal. Sabendo-se que no final do período de aplicação a soma
 dos

 montantes era de R$ 194.871,60 e que o primeiro capital é o dobro do segundo
 e este por sua vez é a quarta parte do terceiro, determine o valor dos três
 capitais.

 
 Veja quais são os assuntos do momento no Yahoo! + Buscados: Top 10 -
 Celebridades - Música - Esportes

 =
 Instruções para entrar na lista, sair da lista e usar a lista em
 http://www.mat.puc-rio.br/~obmlistas/obm-l.html
 =


[obm-l] Re: [obm-l] Re: [obm-l] Triângulo e mediana

2009-07-27 Por tôpico Ralph Teixeira
Estou com o Luiz. Sejam ABC o triangulo, M o medio de BC, e X o tal
circulo inscrito. Suponha spdg que o ponto de tangencia de X com BC
estah em BM. Sejam P e Q os pontos onde o circulo corta a mediana AM.

Como AP=PQ=MQ=x, temos:

Pot(A,X)=2x^2=Pot(M,X)

Agora olhe para as tangentes saindo de A e de M:
2x^2=(p-a)^2=(p-c-a/2)^2
p-a=p-c-a/2
a=2c

Agora, pela Lei das Medianas:
c^2+b^2-2(3x)^2=a^2/2
Botando a=2c e 2x^2=(p-a)^2=((b-c)/2)^2, vem
c^2+b^2-9(b-c)^2/4=2c^2
Daqui sai b=2.6c ou b=c. A solucao b=c nao presta (triangulo c,c,2c
degenerado). Entao o triangulo tem lados c,2c e 2.6c, ou seja, eh o
triangulo 5,10,13, com alguma semelhanca. Como este aqui tem area
S=raiz(p(p-a)(p-b)(p-c))=raiz(14.9.4.1)=6raiz(14), eu dei sorte e
achei os lados: 5, 10 e 13.

Abraco, Ralph.

P.S.: Se eu nao tivesse dado sorte, usaria uma razao de semelhanca k
para modificar os lados de maneira a chegar aa area pedida.


2009/7/27 luiz silva luizfelipec...@yahoo.com.br:
 Ola Carlos,

 Não conhecia.

 Aparentemente, o que vou descrever gera a uma solução (não fiz as contas) :
 se usarmos potência, conseguiremos determinar os lados do triângulo em
 função de duas variáveis a e b. Após isso, pode-se expressar a mediana em
 função de uma destas variáváveis (novamente, através da potência de um
 ponto) e, através da fórmula da mediana, podemos encontrar a em função de b
 ou b em função de a. Assim, teremos os lados expressos através de uma única
 variável.

 Podemos agora usar a fórmula da área com o perímetro, para achar o valor da
 variável que aparece nas expressões representando os lados e, assim,
 determinar os lados do triângulo.

 Particularmente, achei essa possível solução muito braçal..por isso não
 fiz as contas...sendo assim, com certeza deve haver uma soluão mais
 elegante.

 Abs
 Felipe

 --- Em dom, 26/7/09, Carlos Gomes cgomes...@uol.com.br escreveu:

 De: Carlos Gomes cgomes...@uol.com.br
 Assunto: [obm-l] Triângulo e mediana
 Para: obm-l obm-l@mat.puc-rio.br
 Data: Domingo, 26 de Julho de 2009, 23:57

 Olá gente...alguém conhece essa?

 O Circulo inscrito no triângulo ABC divide  mediana traçada de A em três
 segmentos de mesma medida. Se a área de ABC é 6.Raiz(14). Calcule as medidas
 dos lados desse triângulo.

 valew, cgomes

 
 Veja quais são os assuntos do momento no Yahoo! + Buscados: Top 10 -
 Celebridades - Música - Esportes

=
Instruções para entrar na lista, sair da lista e usar a lista em
http://www.mat.puc-rio.br/~obmlistas/obm-l.html
=


Re: [obm-l] Probabilidade- problema das moedas de Bertrand (adaptado)

2009-07-20 Por tôpico Ralph Teixeira
Oi, Pedro.

Sua solucao me parece clara, limpa e correta. Eu passei um tempo aqui
procurando o erro da minha, jah que eu tinha feito do jeito complicado
e nao natural (tipo esvazie o balde e recaia no caso anterior, para
quem conhece a piada).  Levei varios minutos ateh perceber que as
nossas respostas NAO sao diferentes... :P :P :P :P

(E obrigado, eh sempre bom saber que alguem le minhas... prolixidades...)

Abraco,
   Ralph (de Bremen, onde ontem vimos o Tao, o Yoccoz, o
Bollobas, o Gowers, o
Lovasz, o Smirnov, e o Reiher, todos no palco ao mesmo tempo)

P.S.: Acho que vou colocar no meu sig Talvez *alguem* leia isso. :)

2009/7/20 Pedro Cardoso pedrolaz...@hotmail.com:

 Sobre o problema C do Ralph, em que eu tentei por todas as informações
 necessáias,
 pra quem pegar a conversa no meio não ter que ficar adivinhando algumas
 coisas.

 PROBLEMA C

 Temos inicialmente 3 caixas,
 caixa 1 com 2 moedas de ouro (O1O2),
 caixa com 2 moedas de prata (P1P2),
 e caixa 3 com uma moeda de cada (O3P3).

 Escolhe-se uma caixa ao acaso, e seleciona-se uma moeda,
 que eh reposta na sua caixa (ou não, pois não faz diferença). Novamente,
 escolhe-se uma caixa ao acaso, DIFERENTE da primeira caixa, e retira-se uma
 SEGUNDA moeda. Sabendo que a primeira moeda eh de ouro, qual a chance de a
 segunda ser de ouro tambem?

 -

 Bom, no caso as possibilidades de se tirar duas moedas de ouro são:

 o1o3 = 1/3*1/2 * 1/2*1/2 (escolho caixa 1, escolho o1; escolho caixa 2,
 escolho o3)
 o3o1 = 1/3*1/2 * 1/2*1/2 (parecido)
 o2o3 = 1/3*1/2 * 1/2*1/2 (parecido)
 o3o2 = 1/3*1/2 * 1/2*1/2 (parecido)

 Somando tudo, dá 1/3 * 1/8 * 4 = 1/6
 Daí, ainda usando as nomenclaturas do Ralph,

 P(OO|OX) = 1/6 / 1/2 = 1/3.

 Vejam aí se não errei alguma coisa.

 Enfim, Ralph, com a sua explicação sobre o PROBLEMA B, acho que ficou
 bem mais fácil.

 Aliás, só para te encorajar a continuar: eu aposto que bastante
 gente lê os e-mails que você escreve, inclusive aqueles em que você diz algo
 como
 Talvez ninguém leia isso. Aprendi bastante coisa de combinatória com
 você. Obrigado mesmo!


 Date: Wed, 15 Jul 2009 04:19:40 -0300
 Subject: Re: [obm-l] Probabilidade- problema das moedas de Bertrand
 (adaptado)
 From: ralp...@gmail.com
 To: obm-l@mat.puc-rio.br

 Oi, Claudio.

 Explica um pouquinho melhor a variacao que voce estah pedindo Digo
 isso porque, no problema original, nao ha uma segunda moeda sendo
 RETIRADA. No original, a pergunta eh se a moeda retirada eh de ouro,
 qual a chance de a outra moeda DESTA MESMA CAIXA ser de ouro tambem?.
 Ela nem retirada eh

 Se voce vai retirar uma segunda moeda, tem de explicar COMO a segunda
 retirada eh feita. Entao vejamos: temos inicialmente 3 caixas, caixa 1
 com 2 moedas de ouro (O1O2), caixa com 2 moedas de prata (P1P2), e
 caixa 3 com uma moeda de cada (O3P3).

 PROBLEMA A: Escolhe-se uma caixa ao acaso, e seleciona-se uma moeda,
 que eh reposta na sua caixa. Novamente, escolhe-se uma caixa ao acaso,
 independentemente da primeira escolha, e retira-se uma SEGUNDA moeda.
 Sabendo que a primeira eh de ouro, qual a chance de a segunda ser de
 ouro tambem?
 RESPOSTA: Retiradas independentes, entao a informacao da primeira
 moeda nao diz nada. Resposta 3/6=1/2.

 PROBLEMA B: Idem ao A, mas a primeira moeda nao eh reposta.
 RESPOSTA: Fica melhor se desenhar uma arvore com quase 36 ramos...
 Bom: ha 6 maneiras de tirar duas moedas de ouro: O1O2, O2O1, O1O3,
 O2O3, O3O1, O3O2. As duas primeiras somam 1/9 (escolher caixa 1 duas
 vezes); as duas proximas somam 1/3.1/3.1/2 (caixa 1, depois caixa 2,
 moeda O3); e a terceira tem probabilidade 1/3.1/2.1/3. Somando tudo,
 Pr(OO)=2/9.
 Agora, a probabilidade da primeira moeda ser de ouro eh 1/2. Entao, a
 probabilidade pedida eh Pr(OO|OX)=(2/9)/(1/2)=4/9.
 Outra maneira de fazer: a primeira moeda veio da caixa com OO com 2/3
 de chance; neste caso, a chance da segunda ser O eh 1/3+1/3.1/2=1/2
 (na segunda retirada, 1/3 de pegar a mesma caixa, e 1/3 de pegar a
 caixa OP). Se a primeira veio de OP, a segunda soh eh se voce
 escolher a caixa OO, isto eh, 1/3 de chance. Juntando tudo:
 Pr(OO|OX)=2/3.1/2+1/3.1/3=4/9

 PROBLEMA C: A segunda caixa TEM DE SER DIFERENTE DA PRIMEIRA; neste
 caso nao faz diferenca se a primeira moeda eh reposta ou nao Deixo
 esse pra voces. Resposta: 2/3.1/4+1/3.1/2=1/3.

 Abraco, Ralph.

 2009/7/14 Claudio Dias claudiomd...@hotmail.com:
  Oi, Walter.
 
  O problema original é dessa forma( resposta 2/3). Ele acaba induzindo a
  mesma caixa. Mas se não tivesse que ser da mesma caixa. Explo. a
  primeira
  retirada era da segunda caixa e a segunda da primeira ou a primeira
  retirada
  é da caixa 1 e a segunda da caixa 2. Esse foi o questionamento.
 
  
  Date: Tue, 14 Jul 2009 21:22:18 -0300
  Subject: Re: [obm-l] Probabilidade- problema das moedas de Bertrand
  (adaptado)
  From: wtade...@gmail.com
  To: obm-l@mat.puc-rio.br
 
  Oi, Claudio
 
  A pergunta 

Re: [obm-l] Probabilidade- problema das moedas de Bertrand (adaptado)

2009-07-15 Por tôpico Ralph Teixeira
Oi, Claudio.

Explica um pouquinho melhor a variacao que voce estah pedindo Digo
isso porque, no problema original, nao ha uma segunda moeda sendo
RETIRADA. No original, a pergunta eh se a moeda retirada eh de ouro,
qual a chance de a outra moeda DESTA MESMA CAIXA ser de ouro tambem?.
Ela nem retirada eh

Se voce vai retirar uma segunda moeda, tem de explicar COMO a segunda
retirada eh feita. Entao vejamos: temos inicialmente 3 caixas, caixa 1
com 2 moedas de ouro (O1O2), caixa  com 2 moedas de prata (P1P2), e
caixa 3 com uma moeda de cada (O3P3).

PROBLEMA A: Escolhe-se uma caixa ao acaso, e seleciona-se uma moeda,
que eh reposta na sua caixa. Novamente, escolhe-se uma caixa ao acaso,
independentemente da primeira escolha, e retira-se uma SEGUNDA moeda.
Sabendo que a primeira eh de ouro, qual a chance de a segunda ser de
ouro tambem?
RESPOSTA: Retiradas independentes, entao a informacao da primeira
moeda nao diz nada. Resposta 3/6=1/2.

PROBLEMA B: Idem ao A, mas a primeira moeda nao eh reposta.
RESPOSTA: Fica melhor se desenhar uma arvore com quase 36 ramos...
Bom: ha 6 maneiras de tirar duas moedas de ouro: O1O2, O2O1, O1O3,
O2O3, O3O1, O3O2. As duas primeiras somam 1/9 (escolher caixa 1 duas
vezes); as duas proximas somam 1/3.1/3.1/2 (caixa 1, depois caixa 2,
moeda O3); e a terceira tem probabilidade 1/3.1/2.1/3. Somando tudo,
Pr(OO)=2/9.
Agora, a probabilidade da primeira moeda ser de ouro eh 1/2. Entao, a
probabilidade pedida eh Pr(OO|OX)=(2/9)/(1/2)=4/9.
Outra maneira de fazer: a primeira moeda veio da caixa com OO com 2/3
de chance; neste caso, a chance da segunda ser O eh 1/3+1/3.1/2=1/2
(na segunda retirada, 1/3 de pegar a mesma caixa, e 1/3 de pegar a
caixa OP). Se a primeira veio de OP, a segunda soh eh  se voce
escolher a caixa OO, isto eh, 1/3 de chance. Juntando tudo:
Pr(OO|OX)=2/3.1/2+1/3.1/3=4/9

PROBLEMA C: A segunda caixa TEM DE SER DIFERENTE DA PRIMEIRA; neste
caso nao faz diferenca se a primeira moeda eh reposta ou nao Deixo
esse pra voces. Resposta: 2/3.1/4+1/3.1/2=1/3.

Abraco, Ralph.

2009/7/14 Claudio Dias claudiomd...@hotmail.com:
 Oi, Walter.

 O problema original é dessa forma( resposta 2/3). Ele acaba induzindo a
 mesma caixa. Mas se não tivesse que ser da mesma caixa. Explo. a primeira
 retirada era da segunda caixa e a segunda da primeira ou a primeira retirada
 é da caixa 1 e a segunda da caixa 2. Esse foi o questionamento.

 
 Date: Tue, 14 Jul 2009 21:22:18 -0300
 Subject: Re: [obm-l] Probabilidade- problema das moedas de Bertrand
 (adaptado)
 From: wtade...@gmail.com
 To: obm-l@mat.puc-rio.br

 Oi, Claudio

 A pergunta não se resumiria em Se a moeda selecionada é de ouro, qual  a
 probilidade de ser da caixa 1?.
 Tentei fazer a árvore e saiu assim:

 Ramo 1: P(cx1).P(ouro) = (1/3). (1) (seleciona a caixa 1 e sempre sai ouro)
 Ramo 2: P(c2).P(ouro) = (1/3).(1/2) (seleciona a caixa 2 e sai um ouro com)
 Ramo 3: P(cx3).P(ouro) = (1/3).(0) (seleciona a caixa 3 e não tem ouro)

 P(ouro) = (1/3).(1)+(1/3).(1/2) = 1/3 + 1/6 = 3/6 = 1/2
 P(cx1/ouro) = P(cx1 e ouro)/P(ouro) = (1/3)/(1/2) =2/3

 Fiz besteira?

 Abraços

 2009/7/14 Fabio Bernardo prof_fabioberna...@yahoo.com.br

 Vc só esqueceu de postar o problema... Rs...

 - Original Message -
 From: Claudio Dias
 To: obm-l@mat.puc-rio.br
 Sent: Tuesday, July 14, 2009 12:28 PM
 Subject: [obm-l] Probabilidade- problema das moedas de Bertrand (adaptado)
 Caros colegas da lista.

 Essa semana me deparei com o problema de probabilidade sobre as moedas de
 Bertrand. No momento da sua resolução, fui questionado sobre a possibilidade
 da segunda moeda, não necessariamente, ser da mesma caixa. Pensei em
 trabalhar a probabilidade condicional na união das três caixas ( C1 U C2 U
 C3 ), ou seja, P(C1 U C2 U C3 / O). Achei 8/9. É possível?
 Tentei fazer uma árvore e não obtive esse resultado.

 Desde já, agradeço a oportunidade de discussão.

 Claudio Dias




 
 Conheça os novos produtos Windows Live. Clique aqui!


 --


 
 Quer uma internet mais segura? Baixe agora o novo Internet Explorer 8. É
 grátis!

=
Instruções para entrar na lista, sair da lista e usar a lista em
http://www.mat.puc-rio.br/~obmlistas/obm-l.html
=


[obm-l] Re: [obm-l] solução intragavel

2009-07-04 Por tôpico Ralph Teixeira
Se a0, entao a concavidade da parabola y=f(x)=ax^2+bx+c eh para cima.
Faca um esbocinho desta parabola, cortando o grafico em duas raizes.
Note que um numero m estaria entre as raizes se, e somente se,
f(m)=0.

Analogamente, se a0, a concavidade eh para baixo; m estaria entre as
raizes se, e somente se, f(m)=0.

Juntando os dois casos: para que m esteja entre as raizes da
quadratica, a e f(m) devem ter sinais opostos (ou f(m)=0), isto eh,
devemos ter a.f(m)=0.

Abraco, Ralph.

P.S.: Alem disso, eh interessante notar que, se af(m)=0 para algum m,
entao a quadratica TEM que ter raizes reais. Afinal, para x muito
grande, f(x) tem o sinal de a; mas f(m) tem o sinal oposto ao de a!
Entao f(x) tem de ter uma raiz real acima de m (e outra abaixo).

2009/6/29 Thelio Gama teliog...@gmail.com:
 Boa noite professores,
 Não consegui digerir a explicação em vermelho da questão abaixo. Por que
 tenha sinal contrário da concavidade da função e por que  a.f(3)=0    e
 não     a.f(3)=0 ???
 Agradeço se puderem dar uma explicação.
 Abraço a todos!
 Thelio
 Seja f(x) = ax² + (1– a)x + 1, onde a é um número real diferente de zero.
 Determine os valores de a para os quais as raízes da equação f(x) = 0 são
 reais e o número x = 3 pertence ao intervalo fechado compreendido entre as
 raízes.
 Para que x pertença ao intervalo fechado entre as raízes, é necessário que o
 valor da função no ponto 3 que é f(3) = 6a+4, tenha sinal contrário da
 concavidade da função do segundo grau ou f(3)=0; logo: a.f(3)=0. logo
 -2/3=a0.

=
Instruções para entrar na lista, sair da lista e usar a lista em
http://www.mat.puc-rio.br/~obmlistas/obm-l.html
=


[obm-l] Re: [obm-l] solução intragavel

2009-07-04 Por tôpico Ralph Teixeira
Ah, perdao, errei: onde eu disse cortando o grafico, leia-se
cortando o eixo Ox.

Abraco, Ralph.

2009/7/4 Ralph Teixeira ralp...@gmail.com:
 Se a0, entao a concavidade da parabola y=f(x)=ax^2+bx+c eh para cima.
 Faca um esbocinho desta parabola, cortando o grafico em duas raizes.
 Note que um numero m estaria entre as raizes se, e somente se,
 f(m)=0.

 Analogamente, se a0, a concavidade eh para baixo; m estaria entre as
 raizes se, e somente se, f(m)=0.

 Juntando os dois casos: para que m esteja entre as raizes da
 quadratica, a e f(m) devem ter sinais opostos (ou f(m)=0), isto eh,
 devemos ter a.f(m)=0.

 Abraco, Ralph.

 P.S.: Alem disso, eh interessante notar que, se af(m)=0 para algum m,
 entao a quadratica TEM que ter raizes reais. Afinal, para x muito
 grande, f(x) tem o sinal de a; mas f(m) tem o sinal oposto ao de a!
 Entao f(x) tem de ter uma raiz real acima de m (e outra abaixo).

 2009/6/29 Thelio Gama teliog...@gmail.com:
 Boa noite professores,
 Não consegui digerir a explicação em vermelho da questão abaixo. Por que
 tenha sinal contrário da concavidade da função e por que  a.f(3)=0    e
 não     a.f(3)=0 ???
 Agradeço se puderem dar uma explicação.
 Abraço a todos!
 Thelio
 Seja f(x) = ax² + (1– a)x + 1, onde a é um número real diferente de zero.
 Determine os valores de a para os quais as raízes da equação f(x) = 0 são
 reais e o número x = 3 pertence ao intervalo fechado compreendido entre as
 raízes.
 Para que x pertença ao intervalo fechado entre as raízes, é necessário que o
 valor da função no ponto 3 que é f(3) = 6a+4, tenha sinal contrário da
 concavidade da função do segundo grau ou f(3)=0; logo: a.f(3)=0. logo
 -2/3=a0.


=
Instruções para entrar na lista, sair da lista e usar a lista em
http://www.mat.puc-rio.br/~obmlistas/obm-l.html
=


[obm-l] Re: [obm-l] Re: [obm-l] Re: [obm-l] Integral 'difíc il'

2009-05-27 Por tôpico Ralph Teixeira
Tá... bom, então eu acho que ele errou na digitação, pois aquela integral,
pô, diverge Não consigo ver onde eu teria errado... :(

Quanto ao Mathematica, só consigo chegar ao e-3/2 cometendo um erro
esquisito: supondo ln(0)=0. Afinal, a integral de dentro seria:

Int[0,e^x] x^2+1/y dy = x^2e^x+ln(e^x)-ln(0)

Se absurdamente fizermos ln(0)=0 (ou, sei lá, como ele não existe eu o
ignoro e continuo o resto, já que fui mal programado por alguém), a integral
original daria:

Int[0,x]x^2e^x+xdx=e-3/2...

(Será que não é e-3/2+Inf??)

Contra o livro e contra o software! Coragem!

Abraço,
  Ralph

2009/5/27 Angelo Schranko quintern...@yahoo.com.br

Ralph, obrigado pela análise.
Também tenho vários argumentos para a não existência de tal integral,
contudo, sua resposta pelo Mathematica dá -3/2 + e

De fato está escrito corretamente!

Está no exercício 55 do livro Numerical Methods for Engineers and
Scientists, Joe D. Hoffman.

http://www.4shared.com/file/18204220/5da74c3c/Numerical_Methods_for_Engineers_and_Scientists_2nd_Edition.html?s=1

Obrigado

--- Em ter, 26/5/09, Ralph Teixeira ralp...@gmail.com escreveu:

 De: Ralph Teixeira ralp...@gmail.com
 Assunto: [obm-l] Re: [obm-l] Integral 'difícil'
 Para: obm-l@mat.puc-rio.br
 Data: Terça-feira, 26 de Maio de 2009, 22:20
 Oi, Angelo.

 Vi aqui por alto, talvez eu esteja falando bobagem...
 Eu acho que esta integral iterada nao existe. O problema eh
 que a integral de dentro, que eh impropria pois y^-1 eh
 descontinua em y=0, diverge! De fato:
 Int[0,e^x] (x^2+y^-1) dy = x^2.y+lny (y de 0 ateh e^x)
 = lim(b-0) (x^2.e^x+x)-(x^2.b+lnb) = -Inf ?!?

 Pensando de outro jeito: note que f(x,y)=x^2+y^(-1)
 eh positiva na regiao R que voce deu (0x1,
 0ye^x). Agora, considere o retangulozinho
 S:0x1, ayb onde a,b sao bem pequenos (bom,
 eu soh preciso de a,b1). Se a integral de f em R
 existisse, seria maior ou igual que a integral de f em S,
 certo (pois f eh positiva, e S estah contido em R)?
 Mas:

 Int(0,1)Int(a,b) x^2+y^(-1) dydx=Int(0,1)
 x^2(b-a)+ln(b/a) dx = (b-a)/3 + ln(b/a)
 Mantendo b fixo e tomando a-0, isto se aproxima
 de +Inf. Entao, a sua integral eh maior do que a integral em
 S, que por sua vez fica maior que qualquer numero
 positivo Ela nao pode existir! Confere a digitacao da
 questao para a gente?


 Abraco,
   Ralph

 2009/5/26 Angelo Schranko quintern...@yahoo.com.br


 Pessoal, alguém pode me ajudar por favor???
 Como resolver analiticamente a seguinte integral dupla?


 Int[0,1]Int[0, e^x](x^2 + y^-1)dydx

 Obrigado.

 R. -3/2 + e


  Veja quais são os assuntos do momento no Yahoo!
 +Buscados
 http://br.maisbuscados.yahoo.com


 =
 Instruções para entrar na lista, sair da lista e usar a
 lista em
 http://www.mat.puc-rio.br/~obmlistas/obm-l.html

 =





 Veja quais são os assuntos do momento no Yahoo! +Buscados
http://br.maisbuscados.yahoo.com

=
Instruções para entrar na lista, sair da lista e usar a lista em
http://www.mat.puc-rio.br/~obmlistas/obm-l.html
=


[obm-l] Re: [obm-l] Integral 'difícil'

2009-05-26 Por tôpico Ralph Teixeira
Oi, Angelo.

Vi aqui por alto, talvez eu esteja falando bobagem... Eu acho que esta
integral iterada nao existe. O problema eh que a integral de dentro, que eh
impropria pois y^-1 eh descontinua em y=0, diverge! De fato:
Int[0,e^x] (x^2+y^-1) dy = x^2.y+lny (y de 0 ateh e^x) = lim(b-0)
(x^2.e^x+x)-(x^2.b+lnb) = -Inf ?!?

Pensando de outro jeito: note que f(x,y)=x^2+y^(-1) eh positiva na regiao R
que voce deu (0x1, 0ye^x). Agora, considere o retangulozinho S:0x1,
ayb onde a,b sao bem pequenos (bom, eu soh preciso de a,b1). Se a
integral de f em R existisse, seria maior ou igual que a integral de f em S,
certo (pois f eh positiva, e S estah contido em R)? Mas:
Int(0,1)Int(a,b) x^2+y^(-1) dydx=Int(0,1) x^2(b-a)+ln(b/a) dx = (b-a)/3 +
ln(b/a)
Mantendo b fixo e tomando a-0, isto se aproxima de +Inf. Entao, a sua
integral eh maior do que a integral em S, que por sua vez fica maior que
qualquer numero positivo Ela nao pode existir! Confere a digitacao da
questao para a gente?

Abraco,
  Ralph
2009/5/26 Angelo Schranko quintern...@yahoo.com.br


 Pessoal, alguém pode me ajudar por favor???
 Como resolver analiticamente a seguinte integral dupla?

 Int[0,1]Int[0, e^x](x^2 + y^-1)dydx

 Obrigado.

 R. -3/2 + e


  Veja quais são os assuntos do momento no Yahoo! +Buscados
 http://br.maisbuscados.yahoo.com

 =
 Instruções para entrar na lista, sair da lista e usar a lista em
 http://www.mat.puc-rio.br/~obmlistas/obm-l.html
 =



Re: [obm-l]

2009-05-18 Por tôpico Ralph Teixeira
Oi, Samuel. A pergunta eh boa. A resposta... Bom, depende:

ENUNCIADO 1: Seja f:R-R uma funcao e a um numero real fixo. Suponha que
f(ax)=af(x) para todo x real. Entao f(x)=Ax para algum A fixo.

FALSO. Por exemplo, sejam f(x)={3x se x eh racional; 7x se x eh irracional}
e a=2. Note que vale f(2x)=2f(x) para todo x, mas f nao eh linear.

ENUNCIADO 2: Seja f:R-R uma funcao. Suponha que f(ax)=af(x) para quaisquer
a,x reais. Entao f(x)=Ax para algum A fixo.

VERDADEIRO. Basta tomar x=1 e notar que f(a)=f(1).a para todo a real, isto
eh, f(x)=f(1).x para todo x real. Entao A=f(1).

Abraco,
  Ralph

2009/5/18 Samuel Wainer sswai...@hotmail.com

 Se f:R-R então se {f(x)= Ax} A constante,então f(ax) = af(x). Mas o
 recíproco é verdadeiro?
 f(ax)=af(x) = f(x)= Ax ?

 grato


 --
 Descubra uma nova internet. Internet Explorer 8. 
 Mergulhe.http://brasil.microsoft.com.br/IE8/mergulhe/?utm_source=MSN%3BHotmailutm_medium=Taglineutm_campaign=IE8



Re: [obm-l] DG: [Era: serie para ln(2)]

2009-05-10 Por tôpico Ralph Teixeira
Tem razao, Ponce. Voce disse quando UM dos lados vai para zero, eu tinha
lido OS LADOS. Li errado.

Abraco,
   Ralph

2009/5/9 Rogerio Ponce abrlw...@gmail.com

 Oi Ralph,
 o triangulo degenerado que eu dei tinha apenas um lado nulo, para
 forcar que os angulos fossem zero ou que um dos vetores fosse zero (e
 nesse ultimo caso, a resultante continuaria a ter seu modulo igual 'a
 soma dos modulos dos vetores).
 Portanto, se aumentarmos apenas um pouquinho o lado nulo, a
 igualdade se desfaz, e teremos simplesmente:
  ma+mb+mc  a+b+c
 Como esse pouquinho pode ser tao pequeno quanto o siqueira, o valor
 para K e' mesmo 1.

 Abracao,
 Rogerio Ponce



 2009/5/9 Ralph Teixeira ralp...@gmail.com:
   Poxa, o Ponce, com sua vasta esperiencia de decadas e decadas
 matematicas,
  ressuscitou a questao de qual eh a melhor desigualdade do tipo
  ma+mb+mc=k(a+b+c) que a gente consegue escrever?, que estava em
  http://www.mail-archive.com/obm-l@mat.puc-rio.br/msg43875.html
  e mostrou que aqueles 3/2 que a gente achou ha decadas NAO era a melhor
  cota.
 
  (A gente tambem achou que 3/4.(a+b+c)=ma+mb+mc, e tem um argumento lah
 que
  diz que esses 3/4 eh a melhor desigualdade)
 
  Agora fiquei curioso -- qual eh o menor valor possivel de k para garantir
  que a desigualdade acima vale? E antes que alguem diga, o argumento de
 que
  num triangulo degenerado vale a igualdade porque dah 0=0 nao me convence
 --
  afinal, o que eu quero eh o menor valor de k, e esse 0=0 vale para
 qualquer
  k.
 
  Abraco,
   Ralph
 
  2009/5/9 Rogerio Ponce abrlw...@gmail.com
 
  Ola' Nehab, Santa Rita, Luis Lopes e pessoal da lista,
  estou gostando dessas histórias !
 
  ...menos, é claro, da intenção do Nehab em me incluir na lista dos
  quase coroas, visto que ele já conhecia o Bourbaki de trás pra
  frente, há mais de 10 anos, quando o único conjunto que eu conhecia
  era o dos Beatles...
 
  Xiii me entreguei
 
  Mas voltando 'a vaca fria, vamos resolver o problema do Santa Rita, ou
  seja,
  vamos tentar encontrar algum triangulo tal que o seu perimetro seja
  igual a soma das suas medianas.
 
  Entao, considere um triangulo ABC, e seja D o ponto medio do lado BC.
  Pois agora imagine os vetores AB e AC, com origem em A.
 
  Repare que a soma desses 2 vetores vale exatamente o dobro da mediana
 AD.
 
  Por outro lado, a gente sabe que a soma de dois vetores quaisquer
  vale, no maximo, a soma dos dois modulos.
 
  Portanto, a mediana AD vale no maximo a metade da soma dos
  comprimentos AB e AC, ou seja,
  2*AD = AB + AC
 
  Repita essa desigualdade para as outras medianas, e some tudo.
  Fica facil concluir que:
   A SOMA DAS MEDIANAS E' SEMPRE MENOR OU IGUAL AO PERIMETRO DO TRIANGULO.
 
  Alias, essa igualdade so' acontece se os angulos entre os vetores
  forem zero, o que significa que o triangulo tem que ser degenerado.
  E, de fato, isso acontece quando um dos lados do triangulo tem
 comprimento
  zero.
 
  []'s
  Rogerio Ponce
 
 
 
 
  2009/5/6 Carlos Nehab ne...@infolink.com.br:
   Caramba,
  
   Falam em antiguidades e mencionam logo meu nome.  Não sei porque...
 :-)
   .
  
   Você já mencionaram dois maiores monstros do passado em Geometrias
   (imaginem... o quanto passado...o meu passado! hahaha).
   O Virgilio de Athayde Pinheiro e o Célio Pinto de Almeida (que depois
   foi
   dono da construtora que levava seu nome).
   O primeiro, um sábio, um verdadeiro mestre, de corpo e alma (falava
   grego
   fluentemente, era um poço infinito de conhecimento, inclusive sobre
   história
   da Matemática, aspecto tão negligenciado hoje em dia (para os alunos
   fica a
   horrível sensação que tudo em matemática sempre foi do mesmo jeito
   semprecomo se matemática fose uma descoberta dos deuses e não dso
   homens...).
  
   Tive o privilégio de ter sido aluno do Virgílio em Geometria
 Descritiva
   e
   Perspectiva(s).
   Do segundo élio) fui aluno de Desenho Geométrico (ai incluidas as
   Cônicas):
   um monstro e um extraordinário professor.
  
   Mas havia um outro monstro sagrado, tímido e introspectivo, que foi
   professor do IME e da UFF (Dep de Matemática) - Luiz Oswaldo - e tive
   oportunidade de ser aluno dele em ambas as escolas.  No IME, de
   Geometria, e
   na UFF de Teoria dos Números e de Geometria (foi através dele que
   conheci e
   me extasiei com o livro do Niven - Irrational Numbers, já mencionado
   algumas
   vezes por aqui).
  
   Eram do Luiz Oswaldo a grande maioria das questões de Geometria dos
   concursos de admissão ao IME na década de 65 a75, inclusive as
 questões
   de
   Geometria da prova de 72/73 onde tive o prazer de trabalhar com ele
 (eu
   já
   dava aula lá) e participar de forma intensa no massacre da prova de
   Álgebra
   daquele ano. ;-)
  
   Para quem não se lembra eu e o Ponce (um quase coroa da lista) já
   escrevemos por aqui causos engraçados sobre o Luiz Oswaldo,
 inclusive
   sua
   ridícula e única gravata de seu sovina vestuário.
  
   Mas eu

Re: [obm-l] DG: [Era: serie para ln(2)]

2009-05-10 Por tôpico Ralph Teixeira
Eh, isso ai. Eh que, na outra discussao, a gente deixou escapar este limite
superior, deixando-o em 3/2.

Abraco,
Ralph

2009/5/9 Carlos Yuzo Shine cysh...@yahoo.com


 Oi Ralph,

 Eu só comecei a acompanhar a discussão agora, mas o que acontece quando
 fazemos um lado TENDER a zero? Não tender a dar 1? Aí eu acho que o
 argumento do Ponce mostra que o menor k é 1, não?

 Enfim, eu pensei muito rápido e posso estar enganado (agora mesmo estou
 meio apressado...).

 []'s
 Shine


 --- On Sat, 5/9/09, Ralph Teixeira ralp...@gmail.com wrote:

  From: Ralph Teixeira ralp...@gmail.com
  Subject: Re: [obm-l] DG: [Era: serie para ln(2)]
  To: obm-l@mat.puc-rio.br
  Date: Saturday, May 9, 2009, 11:49 AM
  Poxa, o Ponce, com sua vasta esperiencia de decadas e
  decadas matematicas,
  ressuscitou a questao de qual eh a melhor
  desigualdade do tipo
  ma+mb+mc=k(a+b+c) que a gente consegue escrever?,
  que estava em
  http://www.mail-archive.com/obm-l@mat.puc-rio.br/msg43875.html
  e mostrou que aqueles 3/2 que a gente achou ha decadas NAO
  era a melhor
  cota.
 
  (A gente tambem achou que 3/4.(a+b+c)=ma+mb+mc, e tem
  um argumento lah que
  diz que esses 3/4 eh a melhor desigualdade)
 
  Agora fiquei curioso -- qual eh o menor valor possivel de k
  para garantir
  que a desigualdade acima vale? E antes que alguem diga, o
  argumento de que
  num triangulo degenerado vale a igualdade porque dah 0=0
  nao me convence --
  afinal, o que eu quero eh o menor valor de k, e esse 0=0
  vale para qualquer
  k.
 
  Abraco,
   Ralph
 
  2009/5/9 Rogerio Ponce abrlw...@gmail.com
 
   Ola' Nehab, Santa Rita, Luis Lopes e pessoal da
  lista,
   estou gostando dessas histórias !
  
   ...menos, é claro, da intenção do Nehab em me
  incluir na lista dos
   quase coroas, visto que ele já conhecia o
  Bourbaki de trás pra
   frente, há mais de 10 anos, quando o único conjunto
  que eu conhecia
   era o dos Beatles...
  
   Xiii me entreguei
  
   Mas voltando 'a vaca fria, vamos resolver o
  problema do Santa Rita, ou
   seja,
   vamos tentar encontrar algum triangulo tal que o seu
  perimetro seja
   igual a soma das suas medianas.
  
   Entao, considere um triangulo ABC, e seja D o ponto
  medio do lado BC.
   Pois agora imagine os vetores AB e AC, com origem em
  A.
  
   Repare que a soma desses 2 vetores vale exatamente o
  dobro da mediana AD.
  
   Por outro lado, a gente sabe que a soma de dois
  vetores quaisquer
   vale, no maximo, a soma dos dois modulos.
  
   Portanto, a mediana AD vale no maximo a metade da soma
  dos
   comprimentos AB e AC, ou seja,
   2*AD = AB + AC
  
   Repita essa desigualdade para as outras medianas, e
  some tudo.
   Fica facil concluir que:
A SOMA DAS MEDIANAS E' SEMPRE MENOR OU IGUAL AO
  PERIMETRO DO TRIANGULO.
  
   Alias, essa igualdade so' acontece se os angulos
  entre os vetores
   forem zero, o que significa que o triangulo tem que
  ser degenerado.
   E, de fato, isso acontece quando um dos lados do
  triangulo tem comprimento
   zero.
  
   []'s
   Rogerio Ponce
  
  
  
  
   2009/5/6 Carlos Nehab ne...@infolink.com.br:
Caramba,
   
Falam em antiguidades e mencionam logo meu nome.
  Não sei porque... :-) .
   
Você já mencionaram dois maiores monstros do
  passado em Geometrias
(imaginem... o quanto passado...o meu passado!
  hahaha).
O Virgilio de Athayde Pinheiro e o Célio Pinto
  de Almeida (que depois foi
dono da construtora que levava seu nome).
O primeiro, um sábio, um verdadeiro mestre, de
  corpo e alma (falava grego
fluentemente, era um poço infinito de
  conhecimento, inclusive sobre
   história
da Matemática, aspecto tão negligenciado hoje
  em dia (para os alunos fica
   a
horrível sensação que tudo em matemática
  sempre foi do mesmo jeito
semprecomo se matemática fose uma descoberta
  dos deuses e não dso
homens...).
   
Tive o privilégio de ter sido aluno do Virgílio
  em Geometria Descritiva e
Perspectiva(s).
Do segundo élio) fui aluno de Desenho
  Geométrico (ai incluidas as
   Cônicas):
um monstro e um extraordinário professor.
   
Mas havia um outro monstro sagrado, tímido e
  introspectivo, que foi
professor do IME e da UFF (Dep de Matemática) -
  Luiz Oswaldo - e tive
oportunidade de ser aluno dele em ambas as
  escolas.  No IME, de
   Geometria, e
na UFF de Teoria dos Números e de Geometria (foi
  através dele que conheci
   e
me extasiei com o livro do Niven - Irrational
  Numbers, já mencionado
   algumas
vezes por aqui).
   
Eram do Luiz Oswaldo a grande maioria das
  questões de Geometria dos
concursos de admissão ao IME na década de 65
  a75, inclusive as questões
   de
Geometria da prova de 72/73 onde tive o prazer de
  trabalhar com ele (eu
   já
dava aula lá) e participar de forma intensa no
  massacre da prova de
   Álgebra
daquele ano. ;-)
   
Para quem não se lembra eu e o Ponce (um
  quase coroa da lista) já

[obm-l] Re: [obm-l] Re: [obm-l] Re: [obm-l] probabilidades c aça-niquel

2009-05-10 Por tôpico Ralph Teixeira
Eh, aquele enunciado original ganhou nota 4 (de 10) na ENTOR (Escala de
Nariz Torcido do Ralph). Nao eh TERRIVEL, mas poderia ser melhor.

A do Palmerim nao me faz torcer o nariz nao (ENTOR=0). Ainda temos que
pressupor que as frutas sao equiprovaveis (o que me incomoda um pouco, na
barriga, mas o nariz ficou reto) e que as 4 frutas que aparecem sao
independentes entre si (esta nao me incomoda tanto, eh uma hipotese mais
natural). Tambem, se fosse para ser 100% rigoroso, ia ficar um enunciado
muito feio... :) :)

Abraco,
 Ralph

P.S: Ainda to tentando ver se dah para ser mais rigoroso sem perder a
clareza. Consegui:
 Numa máquina de caça-níquel, cada resultado é formado por 4 simbolos
independentes. Cada simbolo eh escolhido ao acaso dentre 6 frutas
diferentes. Calcule a probabilidade de um resultado apresentar duas frutas
iguais e duas outras frutas diferentes entre si.
Acho que ficou mais feio, talvez ateh menos claro... O nariz deu uma fungada
aqui, mas ficou no 0 Mas, pelo menos, o enunciado deixa explicito que os
simbolos sao independentes e as frutas igualmente provaveis (ao acaso
**costuma** indicar isto, o que tambem eh discutivel).
Note que agora nao eh necessario dizer que os simbolos podem repetir -- isto
eh uma consequencia da independencia dos simbolos. :) :)

2009/5/10 Palmerim Soares palmerimsoa...@gmail.com

 Olá Thelio e Mestre Ralph
 Muito pertinentes os comentários do Ralph. Realmente, deve haver extremo
 esmero na formulação dos enunciados, sobretudo quando o assunto é
 probabilidades, que costuma causar confusão justamente por causa das
 ambiguidades nos enunciados.

 Bem, não custa, agora, apresentar uma solução diferente para esta questão:

 Exibir um resultado é uma tarefa que pode ser realizada em 3 etapas
 sucessivas:
 1ª Etapa) Escolha das 3 frutas que irão comparecer no resultado: C(6,3)=20;
 2ª Etapa) Escolha da fruta que comparecerá 2 vezes no resultado: C(3,1)=3;
 3ª Etapa) Escolha das 2 posições destinadas às frutas desiguais: A(4,2)=12
 Pelo Princípio Fundamental da Contagem há: 20 X 3 X 12 = 720 possíveis
 resultados, e, como escreveu o Ralph, já que eles são todos igualmente
 prováveis, a probabilidade pedida
 é 720/1296=5/9.
 Vale notar que a 3ª etapa poderia ser: posicionar as 4 frutas, ou seja,
 fazer uma permutação de 4 elementos, sendo 2 repetidos, o que equivaleria
 exatamente a calcular os anagramas da palavra BALA. (B de Banana, A de
 Amora, L de Laranja e A de Ameixa)
 Então (só para o Thelio ter uma visão geral) o que se quer nesse problema,
 em última análise, é fazer uma permutação de 4 elementos, sendo que dois
 deles são iguais entre si. Mas antes de fazer essa permutação com
 elementos repetidos, precisamos escolher as frutas, o que foi feito nas
 etapas 1 e 2.

 Finalmente, acho que um enunciado que seria aprovado pelo Mestre Ralph
 seria assim:
 (corrija-me se estiver errado, mestre, porque quero aplicar essa questão em
 um simulado)

  Numa máquina de caça-níquel, cada resultado é formado por 4 quaisquer de
 6 frutas diferentes. Supondo que um resultado pode apresentar frutas
 repetidas, calcule a probabilidade de um resultado apresentar duas frutas
 iguais e duas outras frutas diferentes entre si.

 Abraços,
 Palmerim


 2009/5/8 Ralph Teixeira ralp...@gmail.com

 Oi, Thelio.

 Vamos fazer as seguintes hipóteses:

 a) O resultado é formado por 4 símbolos; (isto está bem explícito em
 4 quaisquer...)
 b) Cada símbolo pode ser uma de seis frutas, que designarei por A, B,
 C, D, E, F (também razoavelmente explícito em de 6 frutas
 diferentes...);
 c) Um resultado pode apresentar símbolos iguais (por exemplo, pode ser
 AADE) -- isto está dito, mas com um português ligeiramente ambíguo;
 digo isso pois **gramaticalmente** podendo haver repetição poderia
 se referir a 4 símbolos ou a 6 frutas... mas faz mais sentido se
 for 4 símbolos, podendo haver repetição, que é a minha
 interpretação; a outra interpretação, 6 frutas diferentes, podendo
 haver repetição é meio contraditória...
 d) Em cada símbolo, cada fruta tem a mesma probabilidade de aparecer
 (razoável, mas não é nem um pouco óbvio; aliás, só vou supor isso
 porque tenho que resolver o problema e ele não indicou as
 probabilidades de cada fruta; num caça-níqueis de verdade, isto não
 costuma ser verdadeiro);
 e) Os 4 símbolos são independentes entre si, isto é, o símbolo que
 aparece na primeira janela não afeta de maneira alguma o símbolo da
 segunda (bem razoável, mas também não é certo no caso geral).
 f) O que o enunciado quer é a probabilidade de aparecerem 3 frutas
 distintas, sendo uma delas repetida (se eu quisesse ser muito muito
 chato, diria que AABB tem duas frutas AA iguais e duas frutas BB
 desiguais **da primeira** -- não acho que era isso que o enunciado
 tinha em mente, acho que eles querem dizer, duas frutas iguais e
 duas OUTRAS, desiguais ENTRE SI.). Em linguagem de pôquer: qual é a
 chance de dar um par?

 Agora sim, com tudo destrinchado, eu consigo resolver o

Re: [obm-l] DG: [Era: serie para ln(2)]

2009-05-09 Por tôpico Ralph Teixeira
Poxa, o Ponce, com sua vasta esperiencia de decadas e decadas matematicas,
ressuscitou a questao de qual eh a melhor desigualdade do tipo
ma+mb+mc=k(a+b+c) que a gente consegue escrever?, que estava em
http://www.mail-archive.com/obm-l@mat.puc-rio.br/msg43875.html
e mostrou que aqueles 3/2 que a gente achou ha decadas NAO era a melhor
cota.

(A gente tambem achou que 3/4.(a+b+c)=ma+mb+mc, e tem um argumento lah que
diz que esses 3/4 eh a melhor desigualdade)

Agora fiquei curioso -- qual eh o menor valor possivel de k para garantir
que a desigualdade acima vale? E antes que alguem diga, o argumento de que
num triangulo degenerado vale a igualdade porque dah 0=0 nao me convence --
afinal, o que eu quero eh o menor valor de k, e esse 0=0 vale para qualquer
k.

Abraco,
 Ralph

2009/5/9 Rogerio Ponce abrlw...@gmail.com

 Ola' Nehab, Santa Rita, Luis Lopes e pessoal da lista,
 estou gostando dessas histórias !

 ...menos, é claro, da intenção do Nehab em me incluir na lista dos
 quase coroas, visto que ele já conhecia o Bourbaki de trás pra
 frente, há mais de 10 anos, quando o único conjunto que eu conhecia
 era o dos Beatles...

 Xiii me entreguei

 Mas voltando 'a vaca fria, vamos resolver o problema do Santa Rita, ou
 seja,
 vamos tentar encontrar algum triangulo tal que o seu perimetro seja
 igual a soma das suas medianas.

 Entao, considere um triangulo ABC, e seja D o ponto medio do lado BC.
 Pois agora imagine os vetores AB e AC, com origem em A.

 Repare que a soma desses 2 vetores vale exatamente o dobro da mediana AD.

 Por outro lado, a gente sabe que a soma de dois vetores quaisquer
 vale, no maximo, a soma dos dois modulos.

 Portanto, a mediana AD vale no maximo a metade da soma dos
 comprimentos AB e AC, ou seja,
 2*AD = AB + AC

 Repita essa desigualdade para as outras medianas, e some tudo.
 Fica facil concluir que:
  A SOMA DAS MEDIANAS E' SEMPRE MENOR OU IGUAL AO PERIMETRO DO TRIANGULO.

 Alias, essa igualdade so' acontece se os angulos entre os vetores
 forem zero, o que significa que o triangulo tem que ser degenerado.
 E, de fato, isso acontece quando um dos lados do triangulo tem comprimento
 zero.

 []'s
 Rogerio Ponce




 2009/5/6 Carlos Nehab ne...@infolink.com.br:
  Caramba,
 
  Falam em antiguidades e mencionam logo meu nome.  Não sei porque... :-) .
 
  Você já mencionaram dois maiores monstros do passado em Geometrias
  (imaginem... o quanto passado...o meu passado! hahaha).
  O Virgilio de Athayde Pinheiro e o Célio Pinto de Almeida (que depois foi
  dono da construtora que levava seu nome).
  O primeiro, um sábio, um verdadeiro mestre, de corpo e alma (falava grego
  fluentemente, era um poço infinito de conhecimento, inclusive sobre
 história
  da Matemática, aspecto tão negligenciado hoje em dia (para os alunos fica
 a
  horrível sensação que tudo em matemática sempre foi do mesmo jeito
  semprecomo se matemática fose uma descoberta dos deuses e não dso
  homens...).
 
  Tive o privilégio de ter sido aluno do Virgílio em Geometria Descritiva e
  Perspectiva(s).
  Do segundo élio) fui aluno de Desenho Geométrico (ai incluidas as
 Cônicas):
  um monstro e um extraordinário professor.
 
  Mas havia um outro monstro sagrado, tímido e introspectivo, que foi
  professor do IME e da UFF (Dep de Matemática) - Luiz Oswaldo - e tive
  oportunidade de ser aluno dele em ambas as escolas.  No IME, de
 Geometria, e
  na UFF de Teoria dos Números e de Geometria (foi através dele que conheci
 e
  me extasiei com o livro do Niven - Irrational Numbers, já mencionado
 algumas
  vezes por aqui).
 
  Eram do Luiz Oswaldo a grande maioria das questões de Geometria dos
  concursos de admissão ao IME na década de 65 a75, inclusive as questões
 de
  Geometria da prova de 72/73 onde tive o prazer de trabalhar com ele (eu
 já
  dava aula lá) e participar de forma intensa no massacre da prova de
 Álgebra
  daquele ano. ;-)
 
  Para quem não se lembra eu e o Ponce (um quase coroa da lista) já
  escrevemos por aqui causos engraçados sobre o Luiz Oswaldo, inclusive
 sua
  ridícula e única gravata de seu sovina vestuário.
 
  Mas eu tenho os livros do Virgílio de Descritiva, os do Célio, de Cônicas
 e
  de outras cositas deles.
 
  Quanto ao problema proposto pelo Santa Rita (perímetro e medianas) eu tb
 não
  o havia visto ainda e de fato, como o Luis mencionou, não seria um
 problema
  digamos clássico, pois não é muito comum, na bibliografia, a
 sistematização
  de problemas contendo somas, diferenças etc.  (vide uma das bíblias em
  Wernick, W. Triangle Constructions with Three Located Points. Math.
 Mag.
  55, 227-230, 1982.) e diversos outros papers que vão completando a lista
 do
  Wernick.  Eu tenho estes textos que me foram enviados por meu filho.
 
  Vou tentar resolver o citado problema, mas não juro que seja
 indeterminado,
  pois a soma das medianas varia entre 3/4 e 3/2 do perímetro de um
  triângulo...   Bolas dirão, e daí?  Bem nada como sexto sentido de
  matemágico 

[obm-l] Re: [obm-l] probabilidades caça-niquel

2009-05-08 Por tôpico Ralph Teixeira
Oi, Thelio.

Vamos fazer as seguintes hipóteses:

a) O resultado é formado por 4 símbolos; (isto está bem explícito em
4 quaisquer...)
b) Cada símbolo pode ser uma de seis frutas, que designarei por A, B,
C, D, E, F (também razoavelmente explícito em de 6 frutas
diferentes...);
c) Um resultado pode apresentar símbolos iguais (por exemplo, pode ser
AADE) -- isto está dito, mas com um português ligeiramente ambíguo;
digo isso pois **gramaticalmente** podendo haver repetição poderia
se referir a 4 símbolos ou a 6 frutas... mas faz mais sentido se
for 4 símbolos, podendo haver repetição, que é a minha
interpretação; a outra interpretação, 6 frutas diferentes, podendo
haver repetição é meio contraditória...
d) Em cada símbolo, cada fruta tem a mesma probabilidade de aparecer
(razoável, mas não é nem um pouco óbvio; aliás, só vou supor isso
porque tenho que resolver o problema e ele não indicou as
probabilidades de cada fruta; num caça-níqueis de verdade, isto não
costuma ser verdadeiro);
e) Os 4 símbolos são independentes entre si, isto é, o símbolo que
aparece na primeira janela não afeta de maneira alguma o símbolo da
segunda (bem razoável, mas também não é certo no caso geral).
f) O que o enunciado quer é a probabilidade de aparecerem 3 frutas
distintas, sendo uma delas repetida (se eu quisesse ser muito muito
chato, diria que AABB tem duas frutas AA iguais e duas frutas BB
desiguais **da primeira** -- não acho que era isso que o enunciado
tinha em mente, acho que eles querem dizer, duas frutas iguais e
duas OUTRAS, desiguais ENTRE SI.). Em linguagem de pôquer: qual é a
chance de dar um par?

Agora sim, com tudo destrinchado, eu consigo resolver o problema. Há
6.6.6.6=1296 possíveis resultados, todos igualmente prováveis graças a
(d) e (e). Quantos são da forma XXYZ (ou permutações)?
i) Primeiro, vou escolher as frutas que vão aparecer na minha
sequencia: note que X é bem distinto de Y e Z, que são intercambiáveis
neste momento. Há 6 maneiras de escolher X; agora, há C(5,2) maneiras
de escolher as frutas Y e Z. Então há 6.C(5,2)=60 maneiras de escolher
as frutas que aparecerão no meu resultado.
ii) Mas ainda temos que determinar a ordem em que as frutas aparecerão
no resultado. Há 4 lugares para Y, restam 3 lugares para Z e os outros
têm de ser X. Ou seja, para cada escolha das frutas X, Y e Z que vão
aparecer (onde X é a letra a ser repetida), há 4.3=12 maneiras de
posicioná-las.
iii) Juntando tudo, são 60.12=720 possíveis resultados do tipo um
par. Como eles são todos igualmente prováveis, a probabilidade pedida
é 720/1296=5/9.

Bom, espero não ter errado bobagens, estou meio sem tempo para
conferir o que escrevi.

Abraço,
 Ralph

2009/5/7 Thelio Gama teliog...@gmail.com:
 Bom dia Professores,
 estou bastante confuso com o seguinte problema e agradeço se puderem fazer a
 gentileza de explicá-lo :
 Numa máquina de caça-níquel, cada resultado é formado por 4 quaisquer de 6
 frutas diferentes, podendo haver repetição. Calcule a probabilidade de um
 resultado apresentar duas frutas iguais e outras duas desiguais.
 Obrigado,
 Thelio

=
Instruções para entrar na lista, sair da lista e usar a lista em
http://www.mat.puc-rio.br/~obmlistas/obm-l.html
=


Re: [obm-l] Limite

2009-04-16 Por tôpico Ralph Teixeira
O que o Marcelo fez funciona (ele tirou a exponencicao do ln, que fica
entao multiplicando do lado de fora do ln; depois ele se preocupou soh
com o limite dentro de exp, tem que esquecer o exp por um instante --
esse limite de dentro eh que foi feito por L'Hopital) e eu acho que eh
o jeito mais rapido. Mas, se ainda nao sabemos L'Hopital, temos a
seguinte opcao: vamos fazer os limites laterais trocando variaveis.

Pela direita, quando x - 1, vou tomar y=1/(x-1). Note que x - 1+
sse  y - +Inf. Assim
lim(x - 1+) x^[1/(1-x)] = lim(y - +Inf) (1+1/y)^(-y) = 1 / lim (y -
+Inf) (1+1/y)^y=1/e
(imagino que este limite fundamental do denominador tenha sido feito
previamente, talvez ateh como a definicao de e, senao temos que
trabalhar mais)

Para x - 1-, vou tomar z=1/(1-x). Note que, de novo, z - +Inf (eu
mudei a variavel porque odeio trabalhar com -Inf, eu sempre me
enrolo). Entao x=1-1/z, e:
lim(x - 1-) x^[1/(1-x)] = lim(z - +Inf) (1-1/z)^z = 1/e
(Este tambem jah deve ter sido feito... senao, faca agora z=h+1 e escreva:
lim (h - +inf) (1-1/(h+1))^(h+1) = lim (h/(h+1))^(h+1) = 1/ [lim
((h+1)/h)^h . lim (h+1)/h]= 1/(e.1)=1/e.)

Abraco,
   Ralph

2009/4/16 Henrique Rennó henrique.re...@gmail.com:
 Olá Marcelo,

 Desculpe, mas não entendi sua solução.

 Seria x^[1/(1-x)] = exp(ln x^[1/(1-x)]) como o Leandro citou e não
 exp[ln(x)/(1-x)]?

 O teorema de L'Hôpital seria que para derivadas (que são limites, certo?)
 onde surge uma indeterminação pode-se calcular a derivada da função tantas
 vezes quanto necessário e quando for possível (não existir mais a
 indeterminação) calcula-se o valor da função. Não entendi como você chegou
 em exp[(1/x)/(-1)].

 Eu não mencionei, mas o exercício está num capítulo sobre limites. Assim,
 acredito que a solução não seria através de derivadas ou do Teorema de
 L'Hôpital.

 Obrigado!

 Abraços

 2009/4/15 Marcelo Salhab Brogliato msbro...@gmail.com

 Olá Henrique,

 x^[1/(1-x)] = exp[ln(x)/(1-x)] aplicando L'Hopital: exp[(1/x)/(-1)] =
 exp(-1/x)

 Logo, o limite vale 1/e.

 abraços,
 Salhab





 2009/4/15 Henrique Rennó henrique.re...@gmail.com

 Existe uma solução algébrica para o seguinte limite?

 lim, x-1, x^[1/(1-x)]

 --
 Henrique




 --
 Henrique


=
Instruções para entrar na lista, sair da lista e usar a lista em
http://www.mat.puc-rio.br/~obmlistas/obm-l.html
=


Re: [obm-l] Esfera tocando aresta

2009-04-16 Por tôpico Ralph Teixeira
Reposta curta para o Walter: sim, tem essa esfera tangente às arestas,
uma só, com diâmetro a.raiz(2), onde a é a aresta do cubo.

---///---

Resposta comprida:

EM DIMENSÃO 1 (na reta)
Um cubo de lado 1 é o intervalo [0,1]; só existe uma esfera
interessante, que passa pelos 2 vértices -- é a esfera de centro 1/2
e diâmetro 1.
Tá, isso foi estranho e sem graça; da fato, em dimensão 1, não há
diferença entre cubos e esferas -- ambos são intervalos.

EM DIMENSÃO 2 (no plano)
Um cub isto é, **quadrado** de lado 1 é [0,1]x[0,1]. Há duas
esf... quer dizer, círculos interessantes:
-- Tangente aos 4 lados (círculo inscrito): o diâmetro é 1;
-- Passando pelos 4 vértices (círculo circunscrito): diâmetro é raiz(2).

EM DIMENSÃO 3 (no espaço)
O cubo é [0,1]x[0,1]x[0,1], aresta 1. Esferas, tem 3 interessantes:
-- Tangente às 6 faces (inscrita): diâmetro 1;
-- Tangente às 12 arestas (nem inscrita nem circunscrita...
arestocrita? aristocrata? que eu saiba, não tem nome curto que termine
com crita): diâmetro raiz(2) (que é a distância entre arestas
paralelas opostas, ou seja, a diagonal da face)
-- Tangente aos 8 vértices, quer dizer, passando pelos vértices
(curcunscrita): diâmetro raiz(3).

Por que parar aqui?
EM DIMENSÃO 4 (no... huh... hiperespaço):
O cubo é [0,1]x[0,1]x[0,1]x[0,1], aresta 1. Tem 4 esferas legais:
-- Tangente às 2.C(4,1)=8 hiperfaces (cada uma é um pedaço de espaço
3D): diâmetro 1;
-- Tangente às 4.C(4,2)=24 faces (cada uma 2D): diâmetro raiz(2);
-- Tangente às 8.C(4,3)=32 arestas (que são segmentos 1D): diâmetro raiz(3);
-- Passando pelos 16.C(4,4)=16 vértices: diâmetro raiz(4)=2.

Por que parar aqui?
DIMENSÃO n:
Cubo=[0,1]^n
A super-esfera tangente às 2^k.C(n,k) faces (cada um com dimensão
n-k) tem diâmetro raiz(k), onde k=1,2,3,...,n. São n super-esferas
legais.
(Fica de exercício o trabalho de explicar donde veio esse 2^k.C(n,k)...)

---///---

Agora vou contar um exercício cuja resposta, por muito tempo, me
deixou pasmado. Acho que foi o Nicolau que me contou isso há muito
tempo atrás.

Exercício 1. Leia os exercícios a seguir e descubra o enunciado e a
resposta do exercício 1.

Exercício 2. Num quadrado de lado 1, dá para inscrever 4
circunferenciazinhas de diâmetro 1/2, uma perto de cada vértice,
cada uma tocando 2 outras e 2 lados, do jeito simétrico natural. Bom,
sobra um espacinho no meio, entre as 4, né? Ponha uma
circunferenciazinha tangente às 4 originais, naquele espacinho do
meio. Qual o raio dela?

Exercício 3. Num cubo de lado 1, dá para inscrever 8 esferazinhas de
diâmetro 1/2, uma perto de cada vértice, cada uma tocando 3 outras e
3 faces, do jeito simétrico natural. Bom, sobra um espacinho no meio,
entre as 8, né? Ponha uma esferazinha tangente às 8 originais, naquele
espacinho do meio. Qual o raio dela?

...

Exercício 9. Num supercubo de dimensão 9 e lado 1, dá para inscrever
2^9=512 superesferazinhas de diâmetro 1/2, cada uma tocando 9 outras
e 9 faces, do jeito simétrico natural. Bom, sobra um espacinho no
meio, entre as 512, né? Ponha uma superesferazinha tangente a todas
as 512 outras, naquele espacinho do meio. Qual o raio dela?

Respostas: Ex. 2: (raiz(2)-1)/4; Ex. 3: (raiz(3)-1)/4; Ex. 9: (raiz(9)-1)/4=1/2

Conclusão: Em dimensão 9, aquele espacinho é tão imenso que a tal da
esferinha que você põe lá dentro TANGENCIA AS FACES DO HIPERCUBO
(pois ela tem diâmetro 1!). Que espacinho que nada, em dimensão 9,
fica um tremendo rombo lá entre aquelas esferas todas! E se aumentar
ainda mais a dimensão, a tal da esferazinha do meio aumenta ainda
mais (raio=(raiz(n)-1)/4 em dimensão n) e começa a ter pedaços FORA do
supercubo!

Ok, isto deve provocar uma discussão legal... :) :) :)

Abraço,
Ralph

2009/4/16 Walter Tadeu Nogueira da Silveira wtade...@gmail.com:
 Amigos da lista

 Mais uma vez solicito um esclarecimento. Na inscrição da esfera em um cubo,
 a aresta do cubo vale o diâmetro da esfera. Logicamente o raio da mesma é a
 metade da aresta. No caso da esfera circunscrever o cubo, será o diãmetro a
 diagonal do mesmo.
 Bom...é possível a esfera tocar as arestas de um cubo uma única vez? Qual
 seria o raio?
 Confesso que não idealizei o desenho. Também há dois casos, inscrição e
 circunscrição?
 Abraços

 --
 Walter Tadeu Nogueira da Silveira

=
Instruções para entrar na lista, sair da lista e usar a lista em
http://www.mat.puc-rio.br/~obmlistas/obm-l.html
=


Re: [obm-l] Limite

2009-04-16 Por tôpico Ralph Teixeira
Eu tomara (tomara!) y=1/(x-1), não y=1/(1-x). É um sinalzinho de diferença.

O limite era de x^(1/(1-x)), não era? Aposto que você estava colocando
x^(1/(x-1)) no Excel -- assim dá e, daquele jeito dá 1/e.

Abraço,
  Ralph

2009/4/16 Henrique Rennó henrique.re...@gmail.com:
 Olá Ralph e Marcelo,

 2009/4/16 Ralph Teixeira ralp...@gmail.com

 O que o Marcelo fez funciona (ele tirou a exponencicao do ln, que fica
 entao multiplicando do lado de fora do ln; depois ele se preocupou soh
 com o limite dentro de exp, tem que esquecer o exp por um instante --
 esse limite de dentro eh que foi feito por L'Hopital) e eu acho que eh
 o jeito mais rapido. Mas, se ainda nao sabemos L'Hopital, temos a
 seguinte opcao: vamos fazer os limites laterais trocando variaveis.

 Pela direita, quando x - 1, vou tomar y=1/(x-1). Note que x - 1+
 sse  y - +Inf. Assim
 lim(x - 1+) x^[1/(1-x)] = lim(y - +Inf) (1+1/y)^(-y) = 1 / lim (y -
 +Inf) (1+1/y)^y=1/e

 Não sei se estou errado, mas seria lim(x - 1+) x^[1/(1-x)] = lim(y - +Inf)
 (1+1/y)^(y) e não lim(y - +Inf) (1+1/y)^(-y), já que y = 1/(1-x). Assim, a
 resposta seria e e não 1/e.

 Coloquei a fórmula no excel e para x-1, x^[1/(1-x)] tende a e.


 (imagino que este limite fundamental do denominador tenha sido feito
 previamente, talvez ateh como a definicao de e, senao temos que
 trabalhar mais)

 Para x - 1-, vou tomar z=1/(1-x). Note que, de novo, z - +Inf (eu
 mudei a variavel porque odeio trabalhar com -Inf, eu sempre me
 enrolo). Entao x=1-1/z, e:
 lim(x - 1-) x^[1/(1-x)] = lim(z - +Inf) (1-1/z)^z = 1/e
 (Este tambem jah deve ter sido feito... senao, faca agora z=h+1 e escreva:
 lim (h - +inf) (1-1/(h+1))^(h+1) = lim (h/(h+1))^(h+1) = 1/ [lim
 ((h+1)/h)^h . lim (h+1)/h]= 1/(e.1)=1/e.)

 Abraco,
       Ralph

 2009/4/16 Henrique Rennó henrique.re...@gmail.com:
  Olá Marcelo,
 
  Desculpe, mas não entendi sua solução.
 
  Seria x^[1/(1-x)] = exp(ln x^[1/(1-x)]) como o Leandro citou e não
  exp[ln(x)/(1-x)]?
 
  O teorema de L'Hôpital seria que para derivadas (que são limites,
  certo?)
  onde surge uma indeterminação pode-se calcular a derivada da função
  tantas
  vezes quanto necessário e quando for possível (não existir mais a
  indeterminação) calcula-se o valor da função. Não entendi como você
  chegou
  em exp[(1/x)/(-1)].
 
  Eu não mencionei, mas o exercício está num capítulo sobre limites.
  Assim,
  acredito que a solução não seria através de derivadas ou do Teorema de
  L'Hôpital.
 
  Obrigado!
 
  Abraços
 
  2009/4/15 Marcelo Salhab Brogliato msbro...@gmail.com
 
  Olá Henrique,
 
  x^[1/(1-x)] = exp[ln(x)/(1-x)] aplicando L'Hopital: exp[(1/x)/(-1)] =
  exp(-1/x)
 
  Logo, o limite vale 1/e.
 
  abraços,
  Salhab
 
 
 
 
 
  2009/4/15 Henrique Rennó henrique.re...@gmail.com
 
  Existe uma solução algébrica para o seguinte limite?
 
  lim, x-1, x^[1/(1-x)]
 
  --
  Henrique
 
 
 
 
  --
  Henrique
 

 =
 Instruções para entrar na lista, sair da lista e usar a lista em
 http://www.mat.puc-rio.br/~obmlistas/obm-l.html
 =



 --
 Henrique


=
Instruções para entrar na lista, sair da lista e usar a lista em
http://www.mat.puc-rio.br/~obmlistas/obm-l.html
=


Re: [obm-l] Matrizes

2009-04-15 Por tôpico Ralph Teixeira
Talvez o dilema seja o seguinte: aplicar o metodo de Gauss EM QUEM?

Eh possivel aplicar o metodo de Gauss aa matriz A-lambda.I; acho que
era isso que o professor da Unicamp tava pensando... Com isso, voce
calcula o determinante de A-lambda.I e, portanto, o polinomio
caracteristico, cujas raizes sao os autovalores. Note-se que, como tem
uma variavel lambda no meio da matriz, voce tem que tomar cuidado --
antes de dividir por qualquer coisa que tenha lambda no meio, voce
frequentemente tem de pensar se essa coisa pode ser zero.

Agora, o Bruno tem razao: se voce aplicar o metodo de Gauss direto na
matriz A, as operacoes de linha MUDAM os autovalores, entao nao
adianta nada, como nos exemplos que ele mostrou.

Abraco,
  Ralph


2009/4/11 Bernardo Freitas Paulo da Costa bernardo...@gmail.com:
 Resposta rapida, estou meio sem tempo :

 Hum, tem uma coisa que o processo de Gauss permite calcular
 facilmente, que é o modulo do determinante da matriz ! Porque se você
 disser pro computador nao multiplicar nenhuma linha (sem adicionar a
 uma outra, isso pode, sem problemas), como operaçoes que levam esta
 linha em outra conservam o determinante por multilinearidade e
 anti-simetria (uma matriz com duas linhas iguais é de det = 0, e três
 matrizes com uma linha de uma que é a soma da mesma linha das outras
 duas, e o resto igual, tem det = soma dos dois dets) no final do
 processo você tera o sinal do determinante. Se você prestar atençao
 nas matrizes de permutaçao que você usar (ou seja, calcular o
 determinante delas) você pode inclusive descobrir o sinal do
 determinante. Repare que nessa bagunça toda, você pode ter perdido os
 autovalores, que eles podem mudar bastante no processo. Mas isso nao
 importa, o determinante é conservado. E é por isso que é importante de
 estudar Algebra linear, porque muitas das demonstraçoes vêm junto com
 duas coisas :
 1) Idéias interessantes de invariantes
 2) Algoritmos

 E, se você gosta disso, pode se interessar também pela questao da
 estabilidade numérica do algoritmo, e é por isso que muitas vezes se
 faz uma normalizaçao para evitar numeros muito grandes ou muito
 pequenos. E nisso, você inclui mais uma coisa a prestar atençao na
 hora de calcular o determinante (tem que pensar nao soh nas matrizes
 de permutaçao, mas também nas matrizes de normalizaçao).

 Um grande abraço,
 --
 Bernardo Freitas Paulo da Costa


 2009/4/11 Albert Bouskela bousk...@ymail.com:
 Chi! O Bruno ficou zangado... Acho até que podia, mas é 6ª feira santa (se
 bem que eu não conheço nenhuma 6ª feira pagã), então esta deve ser mais
 santa do que as outras. Quando você, Bruno, inexoravelmente, chegar à minha
 idade, vai ver que é sempre melhor manter o bom humor e rir um pouco
 daqueles e para aqueles que involuntariamente nos irritam.



 Bem, deixa eu tentar esclarecer algumas coisas:



 O método de eliminação (ou de triangulação) de Gauss serve, basicamente,
 para resolver sistemas de equações lineares do tipo Kx = F [1], onde K é uma
 matriz nxn (quadrada), x é o vetor de incógnitas e F é o vetor independente.
 Repare que através do método de Gauss chega-se a uma matriz triangular, cuja
 diagonal principal é igual a 1 ( r(i, i) = 1). Daí: 1*x(n) = F’(n) -- x(n)
 = F’(n) e por retro-substituição se calcula x(n-1), x(n-2) ... x(1).



 Vantagens do método de Gauss:

 É o mais eficiente (seu algoritmo tem o menor números de passos ou linhas);

 Para matrizes positivo-definidas [2] é numericamente estável (isto é
 importantíssimo para as aplicações práticas);

 Caso o sistema seja indeterminado (é o caso da matriz K apresentar 2 ou mais
 linhas LD), vai aparecer um (ou mais) zero(s) na diagonal principal. Isto é
 muito útil quando estamos lidando com matrizes muito grandes, p.ex.,
 1000x1000 e não sabemos se o sistema é LI ou LD.



 Desvantagens:

 Não permite o cálculo dos auto-valores da matriz K [3].



 Caso seja necessário conhecer os auto-valores e auto-vetores [4], então
 devemos empregar outros métodos, p.ex., o de Cholesky. A desvantagem do
 método de Cholesky, em relação ao de Gauss, é que o algoritmo correspondente
 requer o cálculo de n raízes quadradas a mais em relação ao método de Gauss.



 Observações:

 [1] De propósito, coloquei o exemplo da Lei de Hooke generalizada, onde K é
 a matriz de rigidez, x é vetor de deslocamentos (que se quer encontrar) e F
 é o vetor das forças atuantes. É assim que na Engenharia Civil é feito o
 cálculo (dimensionamento) das estruturas (p.ex., edifícios).



 [2] São matrizes nas quais a diagonal principal é numericamente
 preponderante: r(i, i)^2  r(i, j)*r(j, i) . Na maioria dos casos práticos,
 a matriz é simétrica – uma matriz de rigidez é SEMPRE simétrica (o que é
 facilmente demonstrável pela reciprocidade ação vs. deslocamento).



 [3] O que disse acima só é válido para estruturas que têm comportamento
 estático (as forças atuantes pouco variam com o tempo, p.ex., o peso
 próprio). No caso de estruturas dinâmicas (sujeitas a ações 

Re: [obm-l] Como diagonalizar uma matriz?

2009-04-07 Por tôpico Ralph Teixeira
Oi, Fernando.

Esta matriz não é diagonalizável! Ela só tem 3 autovetores L.I., e não
4. São eles:
Autovalor 0: (-1,7,1,6)
Autovalor -4: (-5,-2,8,4)
Autovalor 3: (-2,2,-1,3)
(3 é raiz dupla do pol. carac., mas não há outro autovetor asssociado ao 3)
Então o melhor que você consegue é colocá-la na forma de Jordan:

0  0 0 0
0 -4 0 0
0  0 3 0
0  0 1 3

Note aquele 1 abaixo do primeiro 3 -- você não vai conseguir se livrar dele.

Tanto quanto eu sei, não há diagonalização de matrizes no Excel, pelo
menos não nativamente nas versões que eu conheço.

Abraço,
 Ralph

2009/4/7 Fernando Lima Gama Junior fgam...@gmail.com:
 Pessoal, sei que a pergunta parece fácil, mas não estou conseguindo
 diagonalizar a seguinte matriz:

  2 -1 -3 1
 -2 -1 -1 1
  4  0 -2 1
  0 -2 -4 3

 Alguém poderia me ajudar? Há como diagonalizar matrizes no Excel?
 --
 Fernando Gama

=
Instruções para entrar na lista, sair da lista e usar a lista em
http://www.mat.puc-rio.br/~obmlistas/obm-l.html
=


Re: [obm-l] Como diagonalizar uma matriz?

2009-04-07 Por tôpico Ralph Teixeira
Fiz de cabeça... :) :) :)

Tá, usei o computador de novo (não o Excel, mas o tal do Scientific
Workplace). Mas se eu fizesse o polinômio de 4o grau, ele seria
divisível por x (daí o autovalor 0), e aí sobraria um polinômio que é
fatorável como (x-3)^2.(x+4) (daí o autovalor duplo 3, e o -4).
Então, se eu tivesse feito isso, neste caso teria funcionado (pois eu
teria tido sorte) e eu teria achado as raízes.

Em geral, concordo que achar os 4 autovalores de uma matriz 4x4 pode
ser BEM complicado, se a equação que aparecer for nojenta.

Abraço,
 Ralph

2009/4/7 Fernando Lima Gama Junior fgam...@gmail.com:
 Oi Ralph, obrigado pelas respostas. Mas, não sendo diagonalizável, como
 conseguiu achar os autovalores? Fez no braço mesmo? Pq se fizer, vai gerar
 um polinomio de 4º grau de dificil solução algébrica...

 Abcs,

 2009/4/7 Ralph Teixeira ralp...@gmail.com

 Oi, Fernando.

 Esta matriz não é diagonalizável! Ela só tem 3 autovetores L.I., e não
 4. São eles:
 Autovalor 0: (-1,7,1,6)
 Autovalor -4: (-5,-2,8,4)
 Autovalor 3: (-2,2,-1,3)
 (3 é raiz dupla do pol. carac., mas não há outro autovetor asssociado ao
 3)
 Então o melhor que você consegue é colocá-la na forma de Jordan:

 0  0 0 0
 0 -4 0 0
 0  0 3 0
 0  0 1 3

 Note aquele 1 abaixo do primeiro 3 -- você não vai conseguir se livrar
 dele.

 Tanto quanto eu sei, não há diagonalização de matrizes no Excel, pelo
 menos não nativamente nas versões que eu conheço.

 Abraço,
     Ralph

 2009/4/7 Fernando Lima Gama Junior fgam...@gmail.com:
  Pessoal, sei que a pergunta parece fácil, mas não estou conseguindo
  diagonalizar a seguinte matriz:
 
   2 -1 -3 1
  -2 -1 -1 1
   4  0 -2 1
   0 -2 -4 3
 
  Alguém poderia me ajudar? Há como diagonalizar matrizes no Excel?
  --
  Fernando Gama

 =
 Instruções para entrar na lista, sair da lista e usar a lista em
 http://www.mat.puc-rio.br/~obmlistas/obm-l.html
 =



 --
 Fernando Gama



=
Instruções para entrar na lista, sair da lista e usar a lista em
http://www.mat.puc-rio.br/~obmlistas/obm-l.html
=


[obm-l] Re: [obm-l] RES: [obm-l] Re: [obm-l] teoria dos núm eros

2009-03-30 Por tôpico Ralph Teixeira
Pois é, a gente perde os amigos mas não perde as piadas... Foi mal. O
fato é que eu não sei fazer o problema não (exceto de cabeça como eu
fiz).

Agora, um problema mais clássico é o seguinte: seja S a soma dos
algarismos de 50^50. A soma dos algarismos de S é T, e a soma dos
algarismos de T é U. Calcule U.

Não seria este o enunciado? Este eu sei fazer sem ser de cabeça.
Afinal, como o Salhab disse, basta olhar para 5^50. Agora preciso de
uma estimativa do número de algarismos de 5^50. Se eu não tiver log5
(base 10) na mão, eu faço assim:

a) Decorei que 2^10=102410^3 (é bom para estimar potências de 2)
Então: 5^10=(10^10)/(2^10)10^7
Elevo à quinta: 5^5010^35,
Isto é, 5^50 tem, no máximo, 35 algarismos. Assim, S é no máximo
9*35=315. A soma dos algarismos de S seria, no máximo, 2+9+9=20. Em
outras palavras, T=20.

b) Agora, 5^3 = 125 deixa resto 8 (ou -1 se você for mais liberal) na
divisão por 9;
assim, 5^6 deixa resto 8^2=64 oops-quero-dizer 1 na divisão por 9.
Então 5^48 deixa resto 1 na divisão por 9
e enfim 5^50 deixa resto 5^2=25 oops-quero-dizer 7 na divisão por 9.

c) Como a soma dos algarismos de um número deixa o mesmo resto que o
próprio número na divisão por 9, tanto S como T deixam resto 7 na
divisão por 9. Então T só pode ser 7 ou 16 (25 já é demais, pois vimos
que T=20).

d) Se T=7 então U=7; se T=16 então U=7. Resposta: U=7.

Abraço,
 Ralph

P.S.: De cabeça, eu sei que
5^50=88817841970012523233890533447265625, então S=151 e T=7.
P.S.2: Cabeça = Meu Desktop. :) ;) :) ;)

=
Instruções para entrar na lista, sair da lista e usar a lista em
http://www.mat.puc-rio.br/~obmlistas/obm-l.html
=


[obm-l] Re: [obm-l] Re: [obm-l] Re: [obm-l] Re: [obm-l] teor ia dos números

2009-03-30 Por tôpico Ralph Teixeira
2009/3/29 Simão Pedro sp.eur...@gmail.com:

 Desculpe minha ignorância!
 Mas não entendi esse 5^50 é muito fácil!
 Como se calcula 5^50?


 Abraços!
 Simão Pedro.

É fácil: você imagina um campo de futebol bem grande, do tamanho do
planeta Saturno, com 5^50 bodes. Agora conte o número de patas, some 1
(um bode é manco depois que prendeu a pata num asteróide) e divida por
4. ;) ;) ;) ;)

...

...

Tá, eu confesso: comprei o Scientific Workplace, que faz estas contas
na boa. Tenho certeza que há outros pacotes matemáticos grátis por aí
que também fazem estas contas grandes.

Abraço,
Ralph

=
Instruções para entrar na lista, sair da lista e usar a lista em
http://www.mat.puc-rio.br/~obmlistas/obm-l.html
=


[obm-l] Re: [obm-l] teoria dos números

2009-03-29 Por tôpico Ralph Teixeira
Deixa eu ver aqui... de cabeca... 50^50 dah... isso mesmo deixa eu
somar tudo 151.

;) ;) ;)

2009/3/29 fabio bernardo prof_fabioberna...@yahoo.com.br:
 Será q alguém pode ajudar com esse



 Qual a soma dos algarismos de 50^50?





=
Instruções para entrar na lista, sair da lista e usar a lista em
http://www.mat.puc-rio.br/~obmlistas/obm-l.html
=


[obm-l] Re: [obm-l] Re: [obm-l] Re: [obm-l] Integral de exp( x^-2), por que é impossível?

2009-03-24 Por tôpico Ralph Teixeira
Eh, mas esta eh a integral da nota de aula eh DEFINIDA, de -Inf a
+Inf. Esta dah para calcular passando por integrais duplas e
coordenadas polares (este calculo eh belissimo, neh?).

A integral INDEFINIDA (ou a integral definida F(x)=Int (0 a x)
exp(-t^2) dt ) eh impossivel... bom, no sentido que o Leandro falou
ali em cima: nao dah para escreve-la usando apenas as chamadas
funcoes elementares (sin, cos, ln, exponenciais... esqueci alguma?)
e somas, subtracoes, multiplicacoes, divisoes e raizes (FINITAS). Acho
que o Cesar queria ver a demonstracao deste fato; infelizmente, eu nao
a conheco... alias, nao tenho ideia de como seja esta demonstracao.

Agora, se usarmos funcoes nao elementares, dah para escrever sim (por
exemplo, usando a funcao erf, como disse o Bouskela, que por sua vez
eh uma outra integral destas impossiveis, com aspas). Outra
possibilidade para resolve-la (talvez o verbo correto aqui fosse
re-escreve-la...) eh por serie de potencias.

exp(-x^2)=1-x^2+x^4/2!-x^6/3!+x^8/4!-x^10/5!+...+(-1)^n . x^(2n)/n!+...
F(x)=x-x^3/3+x^5/10-x^7/42+x^9/212-x^11/1320+...+(-1)^n.x^(2n+1)/((2n+1).n!)+...

Reforcando de novo o que o Leandro disse, esta divisao entre funcoes
elementares e nao-elementares eh um tanto arbitraria; quase dah
para argumentar que a funcao F(x)=Int (0 a x) exp(-t^2) dt eh tao
elementar quanto o seno, e tao dificil de calcular quanto o seno.
Pense bem: como calcular F(1), e como calcular sin(1)? Eh mais uma
questao de costume -- a gente mexe com o seno frequentemente, mas
raramente com esta F que nem nome ganhou.

Abraco,
   Ralph
=
Instruções para entrar na lista, sair da lista e usar a lista em
http://www.mat.puc-rio.br/~obmlistas/obm-l.html
=


[obm-l] Re: [obm-l] RE: [obm-l] Re: [obm-l] Re: [obm-l] Re: [obm-l] Integral de exp(x^-2), por que é impossível?

2009-03-24 Por tôpico Ralph Teixeira
Oi, Bouskela. Você tem uma certa razão... Mas, sinceramente, o que
diabos é e^x? Mais espcificamente, o que é e^pi, por exemplo? Dá para
definir por limites usando números racionais, mas dá um certo
trabalhinho...

Então tem um pessoal que prefere DEFINIR o logaritmo pela integral, e
DEFINIR a função e^x como sendo a inversa do ln (assim, este tal de
e por definição seria o número cujo ln é 1!). Neste novo universo,
as coisas se encaixam elegantemente (e, de bônus, responde-se a
pergunta do meu parágrafo anterior). Mais detalhadamente, a ordem
lógica desse pessoal é:

*Alerta! Texto DENSO a seguir! Nada difícil, mas está denso!*

0. Não sabemos o que é ln, nunca ouvimos falar de e, não temos a
mínima idéia do que seja a^x quando x não é racional.
1. Para x em (0,+Inf), definimos ln(x)=Int(1 a x) 1/t dt. Assim,
d(lnx)/dx=1/x e ln1=0.
2. Conclua que ln(x) é crescente (pois a derivada é +).
(2a. Em particular, note que ln2ln1=0.)
3. Mostre que ln(x^r)=r.ln(x) (pelo menos para r racional)
-- Um jeito é: tome h(x)=ln(x^r)-rln(x); derivando, usando a Regra da
Cadeia e (1), vem h'(x)=0. Então h(x)=h(1)=0.
4. Usando 2a e 3, temos que ln(2^r)=r.ln2 pode ser tão grande quanto
quisermos se r for grande, e tão negativo quanto quisermos se r for
bem negativo. Assim, a imagem desta misteriosa ln(x) é o intervalo
(-Inf, +Inf).
5. Por (2), lnx é monótona, então invertível; vamos chamar sua inversa
de exp(x):(-Inf, +Inf) - (0,+Inf) (estes intervalos vêm de (4) e
(1)).
6. Mostre que exp(rx)=exp(x)^r (pelo menos para r racional)
-- Um jeito: use que exp(rx)=exp(rln(exp(x)))=exp(ln(exp(x)^r))=exp(x)^r.
7. DEFINIÇÃO: e=exp(1). Assim, exp(x)=exp(x.1)=exp(1)^x=e^x (pelo
menos para x racional)
8. Agora é o contrário: a gente vai MOSTRAR que e=lim ln(1+1/x)^x
quando x vai para Inf. Como? Use L´Hôpital nesta indeterminação do
tipo 1^(+Inf).
9. DEFINIÇÃO: x^y=exp(y.lnx) sempre que x0, para qualquer y,
inclusive y irracional.

Parece que dá MUITO mais trabalho (poxa, são uns 10 teoreminhas
encadeados)... Mas, no processo, a gente prova elegantemente todas as
propriedades dos logaritmos e das exponenciais (bom, tem algumas que
eu não pus aqui, mas que saem de maneira similar).

Quando a gente dá Cálculo 1 *para a matemática* aqui na UFF, a gente
reserva uma aula de 2 horas para falar disso. Eu começo a aula fazendo
o passo 0. Aí eu pergunto: quanto é ln(a.b)? Quanto é ln(10^6)? Quanto
vale e? Respostas corretas (antes do passo 1): não tenho ideia, nunca
vi mais gordoo, é, que é? :) :) :)

Abraço a todos,
Ralph

=
Instruções para entrar na lista, sair da lista e usar a lista em
http://www.mat.puc-rio.br/~obmlistas/obm-l.html
=


Re: [obm-l] Probabilidade - Joao e Maria

2009-03-19 Por tôpico Ralph Teixeira
Olá a todos.

Notação: x significa um número diferente de 6; 6 significa 6
mesmo. Vou denotar a seqüências de lances de Maria e João, na ordem.
Assim, se eu escrevo xx xx xx x6, isto significa que Maria e João se
alternaram 3 vezes lançando números que não são 6, então Maria lançou
outro número diferente de 6 e, finalmente, João ganhou tirando 6.

Uma maneira de descrever o espaço amostral deste jogo é:

U={6,x6,xx6,xxx6,6,...}

cujas probabilidades são, respectivamente, 1/6, 5/6.1/6=5/36,
5/6.5/6.1/6=25/216, e assim por diante (estou supondo dado justo e
lançamentos independentes, que é a hipótese mais razoável já que
ninguém disse nada a este respeito; note que estes números formam uma
PG de razão 5/6 e soma 1 -- a soma ser 1 é um bom sinal!).

Seja J o evento João ganhou e J2 o evento João ganhou na 2a rodada
(de fato, note que J2={x6}). A pergunta é uma probabilidade
condicional: quanto vale Pr(J2|J)?

Bom, Pr(J2|J)=Pr(J2 e J)/Pr(J)=Pr(J2)/Pr(J) (pois J2 está contido em
J, então J2 e J é o mesmo que J2).

Agora Pr(J2)=Pr({x6})=5/36,
enquanto Pr(J)=Pr(x2)+Pr(xxx2)+Pr(x2)+...=
=5/36+5/36.25/36+5/36.25/36.25/36+...=5/36.1/(1-25/36) =5/11 (usei que
isto é a soma dos termos de uma PG infinita, de razão 25/36).

Assim, Pr(J2|J)=(5/36)/(5/11)=11/36. Esta é a resposta. Bom, eu acho
-- vou deixar a galera ver se eu errei alguma bobagem no meio do
caminho.

Abraço,
   Ralph

2009/3/18 Filipe Junqueira filipejunque...@msn.com:
 Eis o aqui o jogo.



 Joao e Maria jogam um jogo de dados. Ganha quem tirar um “6” primeiro.

 Exemplo:



 Maria joga: tira 4

 Joao joga: tira 3

 Conclusao ninguém ganha

 Maria joga: tira 6

 Conclusao: Ganhou



 Com um dado apenas. Sabendo que Maria comeca jogando o dado na primeira
 rodada e que João ganha o jogo.

 Qual a probabilidade de João ter tirado um 6 na segunda rodada!?





 Obrigado pessoal!



 Obs: Eu só sei que a resposta não é 5/36 !







=
Instruções para entrar na lista, sair da lista e usar a lista em
http://www.mat.puc-rio.br/~obmlistas/obm-l.html
=


Re: [obm-l] Probabilidade - Joao e Maria

2009-03-19 Por tôpico Ralph Teixeira
Eh verdade, acho que a sua interpretacao de rodada eh mais razoavel
do que a minha... Com a sua interpretacao:

Pr(J2)=5^3/6^4
Pr(J)=5/11 (esta nao muda)

Entao Pr(J2|J)=11.5^2/6^4=275/1296, que nem voce disse.

Abraco,
Ralph

2009/3/19 Pedro Cardoso pedrolaz...@hotmail.com:
 Oi, Ralph.

 Eu acho que uma rodada consiste em Maria e João jogarem.
 Logo, Pr(J2) = Pr({xx x6}) = 5/6^4.

 No resto, nossas respostas estão iguais.

 Abraços,

 Pedro.

 Date: Thu, 19 Mar 2009 19:36:41 -0300
 Subject: Re: [obm-l] Probabilidade - Joao e Maria
 From: ralp...@gmail.com
 To: obm-l@mat.puc-rio.br

 Olá a todos.

 Notação: x significa um número diferente de 6; 6 significa 6
 mesmo. Vou denotar a seqüências de lances de Maria e João, na ordem.
 Assim, se eu escrevo xx xx xx x6, isto significa que Maria e João se
 alternaram 3 vezes lançando números que não são 6, então Maria lançou
 outro número diferente de 6 e, finalmente, João ganhou tirando 6.

 Uma maneira de descrever o espaço amostral deste jogo é:

 U={6,x6,xx6,xxx6,6,...}

 cujas probabilidades são, respectivamente, 1/6, 5/6.1/6=5/36,
 5/6.5/6.1/6=25/216, e assim por diante (estou supondo dado justo e
 lançamentos independentes, que é a hipótese mais razoável já que
 ninguém disse nada a este respeito; note que estes números formam uma
 PG de razão 5/6 e soma 1 -- a soma ser 1 é um bom sinal!).

 Seja J o evento João ganhou e J2 o evento João ganhou na 2a rodada
 (de fato, note que J2={x6}). A pergunta é uma probabilidade
 condicional: quanto vale Pr(J2|J)?

 Bom, Pr(J2|J)=Pr(J2 e J)/Pr(J)=Pr(J2)/Pr(J) (pois J2 está contido em
 J, então J2 e J é o mesmo que J2).

 Agora Pr(J2)=Pr({x6})=5/36,
 enquanto Pr(J)=Pr(x2)+Pr(xxx2)+Pr(x2)+...=
 =5/36+5/36.25/36+5/36.25/36.25/36+...=5/36.1/(1-25/36) =5/11 (usei que
 isto é a soma dos termos de uma PG infinita, de razão 25/36).

 Assim, Pr(J2|J)=(5/36)/(5/11)=11/36. Esta é a resposta. Bom, eu acho
 -- vou deixar a galera ver se eu errei alguma bobagem no meio do
 caminho.

 Abraço,
 Ralph

 2009/3/18 Filipe Junqueira filipejunque...@msn.com:
  Eis o aqui o jogo.
 
 
 
  Joao e Maria jogam um jogo de dados. Ganha quem tirar um “6” primeiro.
 
  Exemplo:
 
 
 
  Maria joga: tira 4
 
  Joao joga: tira 3
 
  Conclusao ninguém ganha
 
  Maria joga: tira 6
 
  Conclusao: Ganhou
 
 
 
  Com um dado apenas. Sabendo que Maria comeca jogando o dado na primeira
  rodada e que João ganha o jogo.
 
  Qual a probabilidade de João ter tirado um 6 na segunda rodada!?
 
 
 
 
 
  Obrigado pessoal!
 
 
 
  Obs: Eu só sei que a resposta não é 5/36 !
 
 
 
 
 
 

 =
 Instruções para entrar na lista, sair da lista e usar a lista em
 http://www.mat.puc-rio.br/~obmlistas/obm-l.html
 =

 
 Diversão em dobro: compartilhe fotos enquanto conversa usando o Windows Live
 Messenger.

=
Instruções para entrar na lista, sair da lista e usar a lista em
http://www.mat.puc-rio.br/~obmlistas/obm-l.html
=


[obm-l] Re: [obm-l] RES: [obm-l] Demonstração Geom Plana

2009-03-14 Por tôpico Ralph Teixeira
Concordo com o Osmundo: pense num triangulo beeem degenerado, com A
praaaticamente no meio do segmento BC. Os lados deste triangulo sao
quaaase a=2x, b=c=x, portanto o perimetro eh quaaase 4x.

Bom, as medianas sao praaaticamente 3x/2, 3x/2 e 0, com soma 3x ,que
eh 3/4 do perimetro. Entao 3/4 eh a melhor cota possivel.

Abrco,
 Rlph

2009/3/14 Osmundo Caboclo barz...@dglnet.com.br:
 Caro Thelio a desigualdade triangular se presta para essa demonstração. Seja
 ABC um triângulo e seja G seu baricentro. Olhemos para o triângulo BGC,
 podemos escrever 2/3xm_b + 2/3xm_c  a. Fazendo o mesmo para os triângulos
 AGC e AGB e somando as desigualdades ( elas são coerentes para essa soma )
 sai o resultado que você quer.

 Uma boa pergunta é: seria ¾  a melhor cota possível para comparar esses
 elementos ( soma das medianas com o perímetro ) no conjunto de todos os
 triângulos euclidianos ? Eu chutaria que sim, mas não sei responder.

 Um abraço

 Osmundo Caboclo



 

 De: owner-ob...@mat.puc-rio.br [mailto:owner-ob...@mat.puc-rio.br] Em nome
 de Thelio Gama
 Enviada em: sexta-feira, 13 de março de 2009 21:23
 Para: obm-l@mat.puc-rio.br
 Assunto: [obm-l] Demonstração Geom Plana



 Caros professores



 gostaria de uma ajuda na seguinte demonstração:



 Mostre que a soma das três medianas de um triângulo é maior do que os 3/4
 do perímetro



 Tentei resolver por desigualdade triangular, mas não consegui.



 Obrigado



 Thelio

=
Instruções para entrar na lista, sair da lista e usar a lista em
http://www.mat.puc-rio.br/~obmlistas/obm-l.html
=


[obm-l] Re: [obm-l] Conjuntos: Notação das Partes de (A)

2009-03-12 Por tôpico Ralph Teixeira
Ambas estao corretas. Vazio estah contido em qualquer conjunto,
inclusive P(A). Vazio nao pertence a *qualquer* conjunto... mas, como
voce disse, vazio *pertence* a P(A). Ambas corretas, mas significam
coisas distintas.

Abraco,
   Ralph

2009/3/11 Marcelo Rodrigues ge...@ibest.com.br:
 Olá pessoal boa noite, solucionem para mim uma pequena dúvida em conjuntos,
 por favor:

 Temos o conjunto A = {1,2}

 Partes de A = P(A) = vazio, {1},{2},{1,2}, até aí ok.

 O conjunto P(A) possui 4 subconjuntos.

 Agora vem a dúvida:
 A afirmação : vazio está contido em P(A) , está correta ? Não seria: vazio
 pertence a P(A) ?

 Quem tiver um tempinho para dar uma explicaçãozinha nisso, agradeço, muito.

 Abraços, Marcelo.


=
Instruções para entrar na lista, sair da lista e usar a lista em
http://www.mat.puc-rio.br/~obmlistas/obm-l.html
=


[obm-l] Re: [obm-l] Re: [obm-l] Dúvida em questão de Racio cínio

2009-02-03 Por tôpico Ralph Teixeira
Fiz como o João Luís falou, deu certo: faça um diagrama de Venn,
preencha de dentro para fora. Chamando os conjuntos de A, B e C,
(chamo e de interseção, ou de união):

A e B e C = (A e B) e (A e C) = {Cão} (usando II e IV)
A e B e (não C) = (A e B) - (A e B e C) = {Boi} (II)
A e (não B) e C = (A e C) - (A e B e C) = {Ove} (IV)
(não A) e B e C = (B e C) - (A e B e C) = {Cav} (III)
(não A) e B e (não C) = tudo - (A ou C) = {Por} (I e V)
(não A) e (não B) e C = tudo - (A ou B) = {Chi, Coe} (I e VI)
A e (não B) e (não C) = o que sobrou = {Gat, Gal}

Resposta (c). Tá certo.

Abraço,
  Ralph

2009/2/3 João Luís joaolui...@uol.com.br:
 Não fiz ainda, mas creio que é uma simples questão de considerar cada
 fazenda como sendo um conjunto cujos elementos são o tipo de animal que é
 criado lá. Então afirmativas como os animais comuns às fazendas Alfa e Beta
 são somente cães e bois ,  quer dizer que a intersecção dos conjuntos Alfa
 e Beta é o conjunto {cães, bois}.

 Monte o diagrama de Venn para 3 conjuntos, vá preenchendo as regiões com as
 informações de I a VI, que eu acho que sai fácil.

 Uma pergunta:de onde é essa questão? vestibular? concurso? que instituição?
 e que ano?

 Um abraço,

 João Luís.

 - Original Message -
 From: Marcos Xavier
 To: OBM
 Sent: Monday, February 02, 2009 4:31 PM
 Subject: [obm-l] Dúvida em questão de Raciocínio

 !--[endif]--Amigos, estou com dificuldade na seguinte questão:

 Em um país, há três fazendas: Alfa, Beta e Gama. Sabe-se que nessas fazendas
 criam-se somente animais, e também que,

 I.se reunirmos os animais das três fazendas, teremos porcos,
 galinhas, cães, gatos, bois, ovelhas, cavalos, chinchilas e coelhos;

 II.   os animais comuns às fazendas Alfa e Beta são somente cães e bois;

 III.  os animais comuns às fazendas Beta e Gama são somente cães e
 cavalos;

 IV. os animais comuns às fazendas Alfa e Gama são somente cães e
 ovelhas;

 V.  se reunirmos os animais presentes nas fazendas Alfa e Gama, então
 ficaremos com galinhas, cães, gatos, bois, ovelhas, cavalos, chinchilas e
 coelhos; e

 VI. reunirmos os animais presentes nas fazendas Alfa e Beta, então
 teremos porcos, cães, gatos, bois, ovelhas, cavalos e galinhas.

 Logo, pode-se afirmar que:

 A)  a fazenda Alfa abriga apenas bois, cães e. ovelhas;

 B)  a fazenda Beta abriga apenas bois, cães e cavalos;

 C)  a fazenda Beta abriga bois, cães, porcos e cavalos;

 D)  as outras fazendas juntas abrigam um menor número de espécies de
 animais que a fazenda Alfa;

 E)  bois e galinhas vivem apenas na fazenda Alfa.

 A resposta é Letra (C).

 Agradeço a todos.

 Marcos.

 
 Receba GRÁTIS as mensagens do Messenger no seu celular quando você estiver
 offline. Conheça o MSN Mobile! Crie já o seu!

=
Instruções para entrar na lista, sair da lista e usar a lista em
http://www.mat.puc-rio.br/~obmlistas/obm-l.html
=


[obm-l] Re: [obm-l] Colégio Naval

2009-01-24 Por tôpico Ralph Teixeira
Multiplicando tudo por 2yz:

y^3+z^3+8-6yz=0
(y+z+2)(y^2+z^2+2^2-yz-2y-2z)=0
(y+z+2)((y-2)^2+(z-2)^2+(y-z)^2)/2=0

(Usei aqui a conhecida fatoração
x^3+y^3+z^3-3xyz=(x+y+z)(x^2+y^2+z^2-xy-xz-yz))

Então y+z=-2 ou y=z=2; então y+z=-2 ou y+z=4.

Abraço,
   Ralph

2009/1/24 Rauryson Alves raury...@yahoo.com.br:
 Um amigo me mostrou essa questão do colégio naval e eu repasso a vocês para
 tentarmos achar uma solução:

 Sejam y e z número reais não nulos tal que

 (4/yz)+(y^2/2z)+(z^2/2y)=3

 Qual o valor de y+z?

 
 Veja quais são os assuntos do momento no Yahoo! + Buscados: Top 10 -
 Celebridades - Música - Esportes

=
Instruções para entrar na lista, sair da lista e usar a lista em
http://www.mat.puc-rio.br/~obmlistas/obm-l.html
=


[obm-l] Re: [obm-l] Re: [obm-l] Re: [obm-l] Questão de Prob abilidade - CESPE

2009-01-23 Por tôpico Ralph Teixeira
Concordo com o Fábio. Não são equiprováveis é a pedra no sapato de
98% dos problemas de probabilidade que dão errado... :)

Então, usando o raciocínio do Marcelo, temos as seguintes opções (dada
a primeira derrota de A, daqui para a frente):
 0 (neste caso B é campeao) com probabilidade 1/2
 11 (neste caso A é campeao) com probabilidade 1/2.1/2=1/4
 100 (neste caso B é campeao) com probabilidade 1/2.1/2.1/2=1/8
 1010 (neste caso B é campeao) com probabilidade 1/2.1/2.1/2.1/2=1/16
 1011 (neste caso A é campeao) com probabilidade 1/2.1/2.1/2.1/2=1/16

Note que eu fiz as seguintes hipóteses que o enunciado não deixa
claras, mas que creio serem as mais razoáveis possíveis:
i) A e B têm 50% de chance cada de vencer cada partida (que o próprio
Fábio já havia destacado que não estava explícito no enunciado, e, na
minha opinião, devia)
ii) Partidas distintas são independentes entre si -- ou seja, que não
interessa se A está para ser campeão ou B está desesperado, os times
continuam com 50% de chance cada em cada partida, independentemente da
história passada. Por isso que eu posso usar que p(1011) é o produto
1/2.1/2.1/2.1/2.

Então, p(A ser campeão)=p(11)+p(1011)=5/16. Tô com o Fábio (que,
afinal, é da família Teixeira, então **não podia** estar enganado :)
).

Abraço,
   Ralph


2009/1/23 fabio henrique teixeira de souza fabiodja...@ig.com.br:
 Salhab, agradeço a ajuda. No entanto, acho que vc cometeu um engano. Não
 podemos calcular a chance de A ser campeão por 2 casos favoráveis em um
 total de 5, posto que os 5 não são equiprováveis. Estou errado?
 Abraço.

 2009/1/23 Marcelo Salhab Brogliato msbro...@gmail.com

 Olá Fábio,
 dado que A perdeu o primeiro jogo, temos as seguinte possiveis
 configuracoes para os demais jogos:
 vamos convencionar: 1 = A venceu, 0 = B venceu (ou A perdeu).
 0 (neste caso B é campeao)
 11 (neste caso A é campeao)
 100 (neste caso B é campeao)
 1010 (neste caso B é campeao)
 1011 (neste caso A é campeao)
 logo, temos 2/5 = 40% de chance de A ser campeão.

 Outra maneira seria calcular: P(A ser campeao | A perdeu a primeira
 partida) = P(A ser campeao e A perder a primeira partida)/P(A perder a
 primeira partida)

 P(A perder a primeira partida) = 1/2
 P(A ser campeao e A perder a primeira partida) = 2/10 [basta ver que temos
 2 casos favoraveis em 10, pois temos que considerar os casos em que A ganhou
 a primeira partida, que sao analogos aos acima]

 assim: P = (2/10)/(1/2) = 2/5 = 40%

 acho que é isso!!
 abraços,
 Salhab




 2009/1/22 fabio henrique teixeira de souza fabiodja...@ig.com.br

 Senhores, ao fazer a questão abaixo, encontrei como probabilidade 5/16.
 De saída, o problema não deixa claro que as probabilidades de vitória de A e
 de B são iguais. Considerei-as iguais (a 1/2). No entanto, o gabarito
 oficial diz que a declaração é CERTA. Gostaria que alguém também a fizesse
 para que eu possa ter idéia se estou errando ou acertando. Aguardo o
 retorno.




 Analise em certo ou errado:

 A etapa final de um torneio de futebol será disputada entre os

 times A e B, e o campeão será o time que vencer duas partidas

 seguidas ou um total de três partidas. Considerando que os jogos

 que terminarem empatados serão decididos nos pênaltis, de forma

 que sempre haja um vencedor, julgue os itens que se seguem.

 109

 A chance de o time A ser campeão tendo perdido o primeiro

 jogo é de 20%.



=
Instruções para entrar na lista, sair da lista e usar a lista em
http://www.mat.puc-rio.br/~obmlistas/obm-l.html
=


[obm-l] Re: [obm-l] Questão do MENSA ( triangulo equilatero )

2009-01-23 Por tôpico Ralph Teixeira
Oi, Vitório.

Realmente, este não é o menor caminho. Faça um desenho cuidadoso com
A, B, C, P e Q e um caminho *qualquer* PXYQ onde X está em BC e Y em
AC.

Agora seja P' o simétrico de P com relação a BC e seja Q' o simétrico
de Q com relação a AC.

Como PX=P´X e QY=Q'Y, afirmo que os comprimentos de PXYQ e de P'XYQ'
são iguais, certo? Então achar um caminho que minimize o comprimento
PXYQ (tocando BC em X e AC em Y) é o mesmo que achar um caminho que
minimize o comprimento P'XYQ' (tocando BC em X e AC em Y).

Mas note que QUALQUER CAMINHO DE P' A Q' TOCA AS RETAS BC E AC. Em
suma, basta minimizar o caminho de P' a Q', sem se preocupar em tocar
os lados, pois eles serão tocados automaticamente (bom, tecnicamente,
toca **as retas** que suportam os lados, mas isso serve).

Então agora pergunto: como achar o menor caminho que liga os dois
pontos P' e Q' ? Pois é, o comprimento x desejado é simplesmente a
distância de P' a Q' -- você pode calculá-la usando a lei dos cossenos
no triângulo P'AQ', por exemplo, acho que sai rápido assim. Acho que
dá x^2=1300.

Abraço,
  Ralph

2009/1/23  vitorioga...@uol.com.br:

 Um garoto está no ponto P de um jardim cujo formato é o de um triângulo
 equilátero. Ele deve encostar em duas cercas desse jardim e depois chegar a
 um ponto Q. Supondo que o triângulo equilátero tem vértices A, B e C, que o
 ponto P é o baricentro do triângulo ABC e que o ponto Q é o ponto médio
 entre P e A, o menor percurso que o garoto pode fazer para sair de P,
 encostar na cerca BC, depois na cerca AC e chegar ao ponto Q tem comprimento
 x.
 Qual o valor do quadrado de x, se a distância entre P e Q mede 10 metros?

 Olha a minha solução:
 PA = 2/3 de AM, onde M é o ponto médio do lado BC.
 Então, como PQ = QA , PA = 20 cm.
 AM = (3*20)/2 = 30 cm. AM é mediana, altura e bissetriz interna de ABC.
 Assim, PM = 10 cm. Temos a primeira medida, pois ele parte de P e toca o
 lado BC.
 Seja N o ponto médio do lado AC. a segunda medida será enontrada pelo valor
 de MN. Ora, MN é o lado do triângulo órtico , que também é equilátero e
 cujos vértices são MNR, onde R é o ponto médio do lado AB.
 Por outro lado, NR é base média de ABC e lado de MNR.
 valor do lado de ABC :
 (L*R[3])/2 = 30 -- L=20R[3] cm
 valor do lado de MNR :
 L´ = L/2 = 10R[3] cm = MN
 Então, já temos duas medidas cuja soma é:
 (10+10R[3]) cm
 A terceira medida será a hipotenusa QN do triângulo NQS, onde S é o ponto
 médio de AM, e também a intersecção entre AM e NR.
 SN = L´/2 = 10R[3]/2= 5R[3] cm
 Ora, QN = 5 cm
 Por Pitágoras em NQS:
 QN^2 = SN^2 + NQ^2
 QN^2 = (5R[3])^2 + 5^2
 QN^2 = 75 + 25 = 100
 QN = 10 cm
 Então, D, a distância pedida é:
 D= (10+10+10R[3]) cm
 D = 20 + 10R[3]
 Foi pedido D^2:
 D^2 = (20 + 10R[3])^2
 D^2 = 400 + 400R[3] + 300
 D^2 = 700+400R[3]
 D^2 = 700+693 = 1393 cm

 .porém fiz no CABRI e achei medidas menores...esse menor
 percursosei n ão

 Grato pela ajuda

 =
 Instru�ões para entrar na lista, sair da lista e usar a lista em
 http://www.mat.puc-rio.br/~obmlistas/obm-l.html
 =

=
Instru��es para entrar na lista, sair da lista e usar a lista em
http://www.mat.puc-rio.br/~obmlistas/obm-l.html
=


[obm-l] Re: [obm-l] soluções desta equação

2009-01-19 Por tôpico Ralph Teixeira
Oi, Vitório (?).

Para encontrar os valores de x onde as funções se cortam, você se
deparou com a equação

x^n=2x-x^2

x=0 é uma raiz; se x0, dividimos por x e rearrumamos

x^(n-1)+x=2

É fácil verificar que x=1 serve; note também que a função
g(x)=x^(n-1)+x é crescente em x para x positivo, então ela só pode ser
igual a 2 em um ponto (que é x=1 mesmo). Isto mostra que x=0 e x=1 são
as únicas soluções não-negativas da equação original.

(Ou então: tome p(x)=x^(n-1)+x-2; como 1 é uma raiz, podemos dividir
por x-1 -- use Briot-Ruffini ou seu método predileto -- e encontrar
q(x)=p(x)/(x-1)=x^(n-2)+x^(n-3)+x^(n-4)+...+x^2+x+2; como todos os
coeficientes de q(x) são positivos, fica claro que nenhum valor
positivo de x vai zerar isso).

Agora é fazer do jeito que você fez -- eu também só sei fazer com a
integral, não sei o que eles queriam que fosse feito... Deu n=5 pra
mim também.

Abraço,
 Ralph

2009/1/18  vitorioga...@uol.com.br:



 Olá colegas.

 Estava resolvendo a prova da cesgranrio, se não me engano para uma área
 técnica de química.

 Pois bem, deparei-me com a seguinte questão:

 (Cesgranrio) Considere as funções f e g, ambas de IR+ em IR, e cujas leis
 correspondem a f(x) = 2x -x^2 e g(x) = x^n, em que n é um número inteiro
 positivo. A área da região delimitada, acima pelo gráfico de f e abaixo pelo
 gráfico de g, vale 1/2. É correto afirmar que n é um divisor de
 .
 A) 30
 B) 28
 C) 27
 D) 26
 E) 24

 Pensei em colocar como uma equação do segundo  grau, cujas raízes serão 1+
 raiz(1-x^n) e 1-raiz(1-x^n)

 Ora , 1 - x^n =0 se e só se x^n =1. Contudo x0, então 0 x =1.

 Podemos afirmar que 0 e 1 são raízes??

 porém não são as únicas?

 Mas fazendo a integral (0--1) encontramos n =5, divisor de30.

 Grato.

 =
 Instru�ões para entrar na lista, sair da lista e usar a lista em
 http://www.mat.puc-rio.br/~obmlistas/obm-l.html
 =

=
Instru��es para entrar na lista, sair da lista e usar a lista em
http://www.mat.puc-rio.br/~obmlistas/obm-l.html
=


Re: [obm-l] representação de pares ordenados

2009-01-15 Por tôpico Ralph Teixeira
Oi, Henrique.

Resposta curta:
1. Sim, ha varias opcoes -- mas nao eh **uma** definicao de par
ordenado que lhe dah varias opcoes de conjunto! Sao varias opcoes PARA
A DEFINICAO que voce vai usar. Escolha uma definicao, use-a, mas fique
soh com ela, ateh o final.
Por exemplo: tem gente que define os numeros naturais contendo o 0 (eu
gosto assim, acho que o Nicolau tambem), tem gente que define
comecando pelo 1 (o Elon, por exemplo, nos seus livros). Como nao ha
um consenso sobre qual destas definicoes eh a *correta*, a gente tem
que dizer qual definicao estah usando sempre que necessario. A unica
coisa que eu nao posso eh usar as DUAS DEFINICOES ao mesmo tempo -- ai
teriamos que 0 eh natural e nao eh ao mesmo tempo, e toda a minha
matematica vai por agua abaixo.
2. Para as triplas, voce escolhe a definicao que voce quiser, ou ateh
inventa uma nova. Mas eh muito importante que valha o seguinte:

(a,b,c)=(d,e,f) se e somente se a=d, b=e e c=f.

Se voce inventar uma definicao que nao satisfaca isso (uma onde
(1,2,3)=(1,3,2) ou algo assim), bom, suas triplas ordenadas nao terao
as mesmas propriedades de todas as triplas ordenadas que todo mundo
usa. Isso que eu quis dizer com nao serve.

---

Resposta comprida:


Como definir um par ordenado (e, consequentemente, definir AxB, isto
eh, A cartesiano B)? O que *queremos* eh o seguinte:

DEFINICAO 1. Um par ordenado (a,b) eh um objeto que tem a seguinte
propriedade: (a1,b1)=(a2,b2) se e somente se a1=a2 e b1=b2.

Se voce parar para pensar, esta eh realmente a *unica* propriedade que
precisamos dos pares ordenados -- soh sao iguais quando ambas as
componentes sao RESPECTIVAMENTE iguais. Agora, isto eh um pouco
estranho. Afinal, quando voce define um objeto por uma propriedade,
quem garante que existe ALGUM objeto no mundo que a satisfaz?

...

Bom, tem um pessoal que prefere a seguinte definicao:

DEFINICAO 2. Um par ordenado (a,b) eh o conjunto {{a,b},b}.

A vantagem desta eh que ela eh construtiva (bom, ela soh faz uso da
Teoria dos Conjuntos). Agora, tem gente que usa outras pequenas
variacoes, como (a,b)={{a,b},a} ou outras coisas parecidas. Que eu
saiba, nao existe um consenso (e um dos motivos de nao haver um padrao
eh que ninguem usa pra valer a definicao 2 ou suas variantes -- todo
mundo soh usa a propriedade dentro da DEFINICAO 1, que eh mais simples
e, no final, eh o que interessa).

O que fica faltando aqui eh mostrar que o objeto definido pela DEF2
tem de fato a propriedade lah da DEF1 (que, lembre-se, eh o que
queremos de fato). Entao, para sermos rigorosamente logicos,
precisamos provar:

PROPOSICAO: {{a1,b1},b1}={{a2,b2},b2} se e somente se a1=b1 e a2=b2.

Se a1, b1, a2, b2 forem objetos quaisquer, demonstrar isto eh
surpreendentemente dificil. A unica demonstracao que eu tenho precisa
usar um CANHAO de lema que eu nem sei se eh consenso entre os
matematicos: a ideia de que, na Teoria dos Conjuntos, eh proibido ter
um conjunto que pertenca a si mesmo ou a qualquer de seus elementos...

Se a1, b1, a2, b2 forem restritos a numeros, eu sei fazer: nao pode
ser b1={a2,b2} nem b2={a1,b1} (jah que o da esquerda eh um numero e o
da direita eh um conjunto de numeros); entao b1=b2 e {a1,b1}={a2,b2}.
Mas entao {a1,b1}={a2,b1}. Ou estes conjuntos sao ambos unitarios
(entao a1=b1 e a2=b1, donde vem a1=a2), ou ambos tem dois elementos
(entao para os conjuntos serem iguais devemos ter a1=a2).

Isto cuida da IDA do se e somente se. A volta eh imediata. CQD.


Em suma, mostramos que o objeto da DEFINICAO 2 tem a propriedade que
estah na DEFINICAO 1. O engracado eh que, agora, podemos voltar a usar
a definicao 1 sem problema algum -- a definicao 2 junto com a
PROPOSICAO mostram que ha, de fato, objetos que satisfazem a DEFINICAO
1, entao o grande defeito da DEFINICAO 1 acaba de sumir!

Puxa, eu sou prolixo demais. Serah que alguem se deu ao trabalho de
ler tudo isso? :)

Abraco,
  Ralph


2009/1/15 Henrique Rennó henrique.re...@gmail.com:
 Realmente fiquei confuso. Você utilizou que (a, b) = { {a, b}, b } e em uma
 mensagem anterior o Marcelo colocou que (a, b) = { {a}, {a, b} }. Assim, {
 {a, b}, b } = { {a}, {a, b} }, o que não é verdade. Abaixo você escreve uma
 das opções da definição de par ordenado, ou seja, poderiam haver diversos
 conjuntos representando o mesmo par ordenado?

 { {a}, {a, b}, {a, c} } seria (a, b, c) ?
 Nao serve pois teriamos (a,b,c)=(a,c,b), mesmo que bc.

 { {a}, {a, b}, {b, c} } seria (a, b, c) ?
 Nao serve pois teriamos (a,b,a)=(a,a,b), mesmo que ab.

 Não entendi essas observações.

 Desculpe se estou parecendo chato persistindo nas explicações, mas esse
 conceito não parece simples de definir e estou curioso procurando
 entendê-lo.

 2009/1/14 Ralph Teixeira ralp...@gmail.com

 Como alguem jah disse, essas definicoes sao interessantes sob o ponto
 de vista formal, mas pra mim sacais demais para usar de verdade. Mas
 vamos lah: vou usar:

 (a,b)={{a,b},b}

 Se voce realmente quiser generalizar para n-plas ordenadas, uma

Re: [obm-l] representação de pares ordenados

2009-01-14 Por tôpico Ralph Teixeira
Como alguem jah disse, essas definicoes sao interessantes sob o ponto
de vista formal, mas pra mim sacais demais para usar de verdade. Mas
vamos lah: vou usar:

(a,b)={{a,b},b}

Se voce realmente quiser generalizar para n-plas ordenadas, uma opcao
eh definir recursivamente:

(a1, a2,...,an)=((a1,a2,...,an-1),an)

ou seja, uma n-pla de objetos eh um par ordenado, cujo primeiro termo
eh uma (n-1)-pla ordenada e o segundo termo eh o ultimo objeto.
Usando isto e (a,b)={{a,b},b} (que eh uma das opcoes da definicao de
par ordenado), ficaria:

(a,b,c)=((a,b),c)={ {(a,b),c} ,c}={ { {{a,b},b} ,c},c}

Horrivel! Nao tenho nem coragem para ver se esta multitude de chaves
estah correta :)

Como voce mesmo colocou, aquelas tentativas de definicao de tripla
ordenada nao servem. Em resumo:

 { {a}, {a, b}, {a, c} } seria (a, b, c) ?
Nao serve pois teriamos (a,b,c)=(a,c,b), mesmo que bc.

 { {a}, {a, b}, {b, c} } seria (a, b, c) ?
Nao serve pois teriamos (a,b,a)=(a,a,b), mesmo que ab.

Abraco,
   Ralph

2009/1/14 Henrique Rennó henrique.re...@gmail.com:
 Alguém poderia explicar as dúvidas que coloquei? Estaria errada a forma como
 pensei os exemplos?

 On Fri, Jan 9, 2009 at 4:36 PM, Henrique Rennó henrique.re...@gmail.com
 wrote:

 E os seguintes casos?

 1:
 { {a}, {a, b}, {a, c} } seria (a, b, c) ?
 { {a}, {a, b}, {b, c} } seria (a, b, c) ?

 Conjuntos diferentes correspondendo ao mesmo par ordenado.

 2:
 { {a}, {a, b}, {b} } seria (a, b, ?) ou (a, b, b)?
 { {b}, {a, b}, {a} } seria (b, a, ?) ou (b, a, a)?

 Conjuntos iguais correspondendo a pares ordenados diferentes.

 O número de elementos no conjunto (sejam outros conjuntos ou não) é que
 especifica quantos elementos haverá no par ordenado? No caso 2, como a e b
 já foram usados, qual seria o terceiro elemento do par ordenado?

 Estou pegando o conceito errado?

 --
 Henrique



 --
 Henrique


=
Instruções para entrar na lista, sair da lista e usar a lista em
http://www.mat.puc-rio.br/~obmlistas/obm-l.html
=


Re: [obm-l] Indução

2009-01-13 Por tôpico Ralph Teixeira
Oi, Tarso.

Para ser mais exato, o que tem que ser provado eh:
i) s(1) eh V
ii) Para todo k natural, s(k) implica s(k+1). (este eh o PASSO DE INDUCAO)
Pois eh, como voce disse, este TODO k natural eh importante. Seja
lah qual for o raciocinio que voce fizer para provar que s(k) implica
s(k+1), ele tem que valer para TODO k natural (bom, aas vezes a partir
de 1, aas vezes do ponto inicial que voce precisa, mas isso eh outro
detalhe), inclusive 432. Entao, em principio, concordo contigo que
este passo de inducao tem que dar um friozinho na barriga -- serah que
o raciocinio que a gente fez estah suficientemente geral?

Por outro lado, nao eh tao cabeludo fazer um raciocinio que vale para
todos os naturais ao mesmo tempo -- toda vez que a gente usa uma
variavel, a gente estah fazendo um raciocinio para varios numeros ao
mesmo tempo, e a gente acaba, com experiencia, sabendo quais sao os
casos problematicos para os quais tem que ficar de olho. Alias, como
voce disse, a maioria das inducoes que furam dao problemas nos
primeiros passos... bom, pelo menos na minha experiencia. Entao uma
boa ideia eh sempre seguir o argumento do passo de inducao que voce
fez com seu k generico e segui-lo tintim por tintim com os valores
pequenos de k, para verificar se ele funciona mesmo...

Abraco,
   Ralph

2009/1/13 Tarso de Moura Leitão barz...@dglnet.com.br:
 Como sempre a explicação do Prof. Rauph são excelentes e esclarecedoras.
 Examinando-a ocorreu-me uma dúvida.
 Na demonstração por indução devemos estabelecer a veracidade de dois
 enunciados;
 (1) s(1) é verdadeira ( ou s(2), etc. e
 (2) s(k) acarreta em s(k+1).
 No exemplo analisado a implicação s(1) não implica em s(2)[mais; mas o
 argumento não funciona para mostrar que s(1) implica s(2).] ( copiado do
 e-mail resposta do Rauph ).
 Aqui é que a porca torce o rabo 
 Pergunto, não poderia existir uma situação na qual o problema estaria em,
 digamos, s(432) acarreta s(433) ?
 Em todas as falácias de indução que já vi o problema é sempre esse: s(1) não
 acarreta s(2), aí acaba-se descobrindo  o que está errado na seguinte prova
 por indução  .
 Quando vou explicar esse tipo de situação acabo ficando com uma sensação de
 que, afinal, a indução não se faz em dois passos.
 Caro Prof. Rauph, se puder comentar antecipadamente nós ( e muitos alunos
 afinal ) agradecemos.
 Um abraço
 Tarso de Moura Leitão

=
Instruções para entrar na lista, sair da lista e usar a lista em
http://www.mat.puc-rio.br/~obmlistas/obm-l.html
=


Re: [obm-l] indução

2009-01-09 Por tôpico Ralph Teixeira
Resposta curta: o problema eh que o passo de inducao nao funciona de
k=1 para k=2.

Resposta comprida: para provar que uma sentenca s(n) vale para todo n
natural, por inducao, precisamos provar que:
i) s(1) eh V
ii) Para todo k natural, s(k) implica s(k+1)

No nosso caso, s(n) eh: Todo conjunto com n bolas que tenha pelo
menos uma bola azul soh tem bolas azuis.

Como voce disse, s(1) eh V; corretissimo.

Agora, o resto do argumento mostra que s(k) implica s(k+1) para k=2 ou
mais; mas o argumento nao funciona para mostrar que s(1) implica s(2).
De fato, siga a sua demonstracao devagarzinho fingindo que n=1. Quando
voce chegar na frase retire uma bola deste conjunto e reponha a bola
tirada inicialmente, voce nao pode aplicar s(1) a este conjunto para
concluir que esta bola tirada inicialmente eh azul -- afinal, a
hipotese tem pelo menos uma bola azul nao vale para este conjunto de
uma bola (que pode ser de qualquer cor).

Agora, para realmente entender inducao, note que, se s(2) valesse (por
algum motivo estranho), entao seu raciocinio estaria 100% correto e
teriamos que s(n) vale para todo n sim senhor! Traducao: suponha que
voce estah num mundo com n bolas, onde qualquer conjunto de duas
bolas, sendo uma azul, tem que ter duas bolas azuis. Neste mundo, se
ha uma bola azul, todas sao azuis.

E se voce quiser ver se estah MUITO craque: suponha que num mundo com
infinitas bolas, qualquer conjunto de duas bolas com uma azul tem que
ter duas bolas azuis. A inducao sozinha NAO PROVA que todas as bolas
deste mundo sao azuis. Em outras palavras, inducao prova s(n) para
todo n natural -- mas nao prova s(n) quando n=infinito.

Abraco,
Ralph

2009/1/9 Murilo Krell murilo.kr...@gmail.com:
 Pessoal, alguém poderia dar uma ajudinha?

 já quebrei a cabeça, mas não consigo achar

 Explique, com palavras, o erro da seguinte indução:

 Afirmação: Dado um conjunto de n bolas, se uma delas é azul, então todas são
 azuis.
 Demonstração: para n=1, como pelo menos uma bola é azul e há apenas um
 elemento, então todas as bolas são azuis. Suponha a afirmação válida para um
 dado n. Tome um conjunto de n + 1 bolas, onde pelo menos uma é azul. Tire um
 elemento do conjunto que não seja esta bola azul fixada. Pela hipótese de
 indução, todas as bolas desse conjunto com n elementos são azuis. Retire uma
 bola desse conjunto e reponha a bola tirada inicialmente. Novamente pela
 hipótese de indução temos que todas as n + 1 bolas são azuis.

 []'s

 Murilo


=
Instruções para entrar na lista, sair da lista e usar a lista em
http://www.mat.puc-rio.br/~obmlistas/obm-l.html
=


Re: [obm-l] Equação

2008-12-21 Por tôpico Ralph Teixeira
Rearrumando as coisas e fatorando:

(y-3x+2)(y-2)=1

Entao y-2=1 ou y-2=-1... Uma delas nao presta, a outra presta, entao sim,
esta equacao soh tem uma solução no inteiros.

Abraço,
   Ralph

2008/12/21 Eder Albuquerque eder_...@yahoo.com.br

   Olá a todos,

 Alguém poderia me confirmar se a equação abaixo tem mais de uma solução nos
 inteiros:

 y^2 - 3 = x(3y - 6)


 Cheguei facilmente a uma solução, mas não sei se pára aí.

 Obrigado.





  --
 Veja quais são os assuntos do momento no Yahoo! + Buscados: Top 
 10http://br.rd.yahoo.com/mail/taglines/mail/*http://br.maisbuscados.yahoo.com/-
 Celebridadeshttp://br.rd.yahoo.com/mail/taglines/mail/*http://br.maisbuscados.yahoo.com/celebridades/-
 Músicahttp://br.rd.yahoo.com/mail/taglines/mail/*http://br.maisbuscados.yahoo.com/m%C3%BAsica/-
 Esporteshttp://br.rd.yahoo.com/mail/taglines/mail/*http://br.maisbuscados.yahoo.com/esportes/



Re: [obm-l] Então to doido...

2008-12-15 Por tôpico Ralph Teixeira
Vou contar do jeito do Ponce, supondo que ABCD=BCDA por rotação. Para mim
ficou fácil quando eu desisti de tentar sair em uma linha contando tudo dum
jeito só. Isto é, dividindo em casos distintos, fica mais fácil. Vamos lá:

CASOS: podemos escolher n=2, 3 ou 4 cores distintas para pintar o círculo.
(Já que com uma cor só não dá)

n=2: C(x,2)=x(x-1)/2 escolhas de cor. Escolhidas A e B, há apenas um jeito:
ABAB=BABA (já que AABB não vale).
n=3: C(x,3) escolhas de cor. Escolha a cor que vai repetir, digamos, A -- há
3 opções para tanto. Agora está definido: tem que ser ABAC=ACAB (pois AABC
não vale).
n=4: C(x,4) escolhas de cor. Ponha a cor A em algum lugar para ancorar um
setor, e há 3! maneiras de permutar as outras. Literalmente: ABCD, ABDC,
ACBD, ACDB, ADBC, ADCB.

Total: T=C(x,2)+3C(x,3)+6C(x,4)=x(x-1)(x^2-3x+4)/4

Testemos alguns valores para ver se parece ok:
x=0,1 dá  T=0;
x=2 dá T=1 (ABAB);
x=3 dá T=6 (ABAB, ACAC, BCBC, ABAC, BABC, CACB);
x=4 dá T=24 (C(4,2)=6 com 2 cores, C(4,3).3=12 com 3 cores e aquelas 6 com 4
cores)

Mais ainda, T(x) é o único polinômio de grau 4 que passa por esses 5 pontos,
então deve estar certo. :)

Abraço,
Ralph

P.S.: Se a gente supuser que os setores são distintos (sei lá, numerados ou
algo assim), o mesmo método dá:
2C(x,2)+4.3.C(x,3)+4!.C(x,4)=x(x-1)(x^2-3x+3)
P.S.2: E se o disco for de acrílico transparente, de maneira que não dá para
distinguir seus dois lados (ou se ambos os lados serão pintados com as
mesmas cores), a resposta muda, pois então teríamos ABCD=ADCB se você
**virar** o disco! Aí ficaria:
T=C(x,2)+3C(x,3)+3C(x,4)=...


2008/12/12 Rogerio Ponce abrlw...@gmail.com

 Ola' Ruy e colegas da lista,
 parece-me que todas as respostas ate' o momento estao incorretas...
 Analisando o problema para 2 cores (A e B por exemplo), vemos que
 existe apenas a sequencia ABAB para a pintura do circulo.
 A outra possibilidade BABA e', na verdade, a mesma pintura anterior
 apos girarmos o circulo de 90 graus.
 Observe que, como e' dito que os setores sao iguais, fica muito claro
 que este circulo nao tem qualquer referencia espacial.
 Em outras palavras, uma sequencia nao deve ser obtida a partir de
 outra, atraves da rotacao do circulo.
 Assim, para x=2, a resposta tem que ser 1.
 []'s
 Rogerio Ponce.

 OBS: um outro enunciado possivel seria Imagine um circulo dividido em
 4 setores diferentes entre si


 2008/12/4 ruy de oliveira souza ruymat...@ig.com.br:
  Não concordo com o gabarito desse problema. Imagine um circulo dividido
 em
  quatro partes iguais. Existem x cores diferentes para pintar os quatro
  setores . De quantos modos diferentes esses setores pode ser pintados
 sendo
  que setores com uma linha de fronteira não podem ser da mesma côr.
 Renomado
  cursinho de SP dá como gabarito x(x-1).
 Antecipo os agradecimentos para quem analisar...
Abraços

 =
 Instruções para entrar na lista, sair da lista e usar a lista em
 http://www.mat.puc-rio.br/~obmlistas/obm-l.html
 =



Re: [obm-l] IME

2008-12-01 Por tôpico Ralph Teixeira
Oi.

Entendo que um dos (3,1,1) do Walter é (3,1,2). E tô vendo duas opções a
mais: (4,3,1),(4,4,1). Então, por enquanto, deu 24/125, que é quase
a resposta (c)... Será que a gente ainda está devendo alguma opção?

Abraço,
  Ralph
2008/11/29 Walter Tadeu Nogueira da Silveira [EMAIL PROTECTED]

 Não sei se fiz um caminho longo ou se esqueci algo.
 Precisamos ter b^2 - 4ac=0
 Coloquei as opções na ordem (b,a,c)
 Mas os possíveis ternos foram:
 OBS: b não pode ser 1 pois os coeficientes são todos positivos.

 1)(2,1,1)
 2)(3,2,1);(3,1,1);(3,1,1);
 3)(4,1,1);(4,1,2);(4,1,3);(4,1,4)
(4,2,1);(4,2,2)
 4)(5,1,1);(5,1,2);(5,1,3);(5,1,4);(5,1,5)
(5,2,1);(5,2,2);(5,2,3)
(5,3,1);(5,3,2)
(5,4,1)
(5,5,1)
 Só encontrei 22. Logo a probabilidade seria 22/125, creio.
 Abraços
 Walter
 2008/11/29 arkon [EMAIL PROTECTED]

  *Pessoal alguém sabe o motivo da anulação desta questão?*

 *Uma urna contém cinco bolas numeradas de 1 a 5. Retiram-se, com
 reposição, 3 bolas desta urna, sendo a o número da primeira, b o da segunda
 e c o da terceira. Dada a equação quadrática ax^2+bx+c=0, a alternativa que
 expressa a probabilidade das raízes desta equação serem reais é:*

 *
 (A) 19/25. (B) 23/60. (C) 26/125. (D) 26/60.  (E) 25/60.*
 =
 Instru�ões para entrar na lista, sair da lista e usar a lista em
 http://www.mat.puc-rio.br/~obmlistas/obm-l.html=




 --
 Walter Tadeu Nogueira da Silveira
 http://www.professorwaltertadeu.mat.br




Re: [obm-l] Transformada de Fourier

2008-11-25 Por tôpico Ralph Teixeira
Talvez nao seja exatamente o ideal, mas aqui tem alguma coisa que eu escrevi
voltado para Processamento de Imagens:

http://www.visgraf.impa.br/Courses/eescala/index.html

A preocupacao era mais com aplicacoes do que com teoria, mas estah lah.

Abraco,
 Ralph

On Mon, Nov 24, 2008 at 11:03 PM, Denisson [EMAIL PROTECTED] wrote:

 Alguém poderia citar referências sobre a transformada de fourier?. Serve em
 inglês, portugues ou espanhol.

 --
 Denisson




Re: [obm-l] Contagem

2008-11-22 Por tôpico Ralph Teixeira
O problema eh que eles nao deixam claro o que eh uma possibilidade. Se a
ORDEM importa, entao:

PPP=5.4.3=60
IIP=5.4.5=100

Estah aqui os 160 que eles queriam. O problema eh que a palavra escolha
*sugere* (mas, pra mim, nao define) que a ordem nao importa (porque estamos
acostumadissimos a pensar em combinacoes como numero de maneiras de
ESCOLHER).

Abraco,
  Ralph

2008/11/22 João Luís [EMAIL PROTECTED]

  Esta resposta está esquisitíssima, pois o número total de maneiras de se
 escolher 3 números distintos entre 10 é 120. Então é muito simples mostrar
 que a resposta apresentada está (grosseiramente) errada!

 Quanto á solução, P P P dá soma par  e I I P também, mas I P P, não.  Total
 = 60 somas, o resultado permanece o mesmo.

 Um abraço a todos,

 João Luís.



 - Original Message -

 *From:* Walter Tadeu Nogueira da Silveira [EMAIL PROTECTED]
 *To:* obm-l@mat.puc-rio.br
 *Sent:* Friday, November 21, 2008 8:22 PM
 *Subject:* [obm-l] Contagem

 O problema abaixo foi trazido por um aluno. Eis a solução encontrada pela
 turma:

 O número de possibilidades de escolha de 3 números naturais distintos de 1
 a 10, de modo que sua soma seja sempre par, é:

1. 120
2. 220
3. 150
4. 290
5. 160

 SOLUÇÃO. Supõe-se que são cartões com os números onde:
 Pares: 2, 4, 6, 8 e 10
 Ímpares: 1, 3, 5, 7, 9
 Para que a escolha dos três números dê soma par, deve-se ter: P P P ou I P
 P
 a) P P P temos: C(5,3) = 10
 b) I P P temos: C(5,1) x C(5,2) = 5 x 10 = 50
 Total de 10 + 50 = 60 possibilidades.
 Ficaram felizes, mas a resposta apontava 160. Não consegui mostrar o erro a
 eles. Alguém poderia dar uma ajuda? Grato.


 Walter Tadeu Nogueira da Silveira




Re: [obm-l] PA ( literal ) e aritmética.

2008-11-19 Por tôpico Ralph Teixeira

  2) Se, *x *,*y* e *z *são inteiros positivos , com : xyz + xy + xz + yz +
 x + y + z = 384, quanto vale *xyz* ?
GAB. *240*



Some um dos dois lados e fatore tudo:

(x+1)(y+1)(z+1)=385=5.7.11

Como x,y e z sao inteiros positivos, x+1,y+1,z+1=2. Como aquela ali eh a
fatoracao de 385 em primos, a unica opcao eh que {x+1,y+1,z+1}={5,7,11},
isto eh, x, y e z sao 4, 6 e 10 em alguma ordem. Assim, xyz=240.

Abraco,
   Ralph


Re: [obm-l] Jogo dos 4 bits

2008-11-18 Por tôpico Ralph Teixeira
Se o objetivo eh minimizar o numero **maximo** de palpites... Certamente, eh
possivel adivinhar em um maximo de 5 palpites, usando a seguinte estrategia
de ir trocando um digito de cada vez (Pi=i-esimo palpite, Ri=i-esima
resposta):

P1=
P2=0001
P3=0011
P4=0111

Se a resposta melhorou ao passar de Pi para Pi+1, eh porque aquele digito
que voce trocou estah correto, e vice-versa. Ou seja, apos estes 4 palpites,
voce jah sabe os ultimos 3 digitos com certeza.
Agora basta olhar a resposta a P1 para descobrir se o digito incerto eh 0 ou
1; assim, o 5o palpite serah correto.

Exemplo:
R1=1, R2=2, R3=1 e R4=2.
Como R2R1, o ultimo digito eh 1, isto eh, xxx1 (pois esta eh a unica
diferenca entre P1 e P2);
Como R3R2, xx01.
Como R4R3, x101.
Enfim, como R1=1, soh tem um 0 na resposta, entao 1101 eh a resposta.

Esta estrategia eh facilmente generalizavel: sempre eh possivel adivinhar um
numero de n bits com, no maximo, n+1 palpites (agora, serah que dah com
menos?).

Abraco,
  Ralph

2008/11/18 Douglas Ribeiro Silva [EMAIL PROTECTED]

 O jogo dos 4 bits consiste no computador escolher um número de 4 bits
 e o usuário tentar adivinhar. Para cada palpite do usuário o
 computador retorna quantos bits ele acertou.

 Ex: o computador escolhe 0101

 Usuario: 
 PC:2
 Usuario: 0100
 PC: 3
 Usuario: 
 PC: 2
 Usuario: 0111
 PC: 1
 Usuario: 0101
 PC: 4

 Qual a melhor estratégia para o jogo? O jogador deve sempre trocar a
 quantidade de dígitos que o computador indicar? Qual a quantidade
 máxima que um usuário inteligente gastaria para acertar o numero?

 =
 Instruções para entrar na lista, sair da lista e usar a lista em
 http://www.mat.puc-rio.br/~obmlistas/obm-l.html
 =



Re: [obm-l] exercicio simples de probabilidade

2008-10-31 Por tôpico Ralph Teixeira
Para mim, estao faltando dados... Agora, se voce me disser que:

i) Em cada partida, a chance de A vencer eh p;
ii) As partidas sao independentes entre si;

Entao (ainda nao estah claro qual eh a pergunta, entao apresento duas
respostas):

Pr(A vencer exatamente 4 partidas) = C(6,4).p^4.(1-p)^2
Pr(A vencer 4 ou mais partidas) = C(6,4).p^4.(1-p)^2+C(6,5).p^5.(1-p)+p^6

Em particular, se p=50%, entao:

Pr(A vencer exatamente 4) = 15/64 = 23.4375%
Pr(A vencer pelo menos 4) = 11/32 = 34.375%

Abraco,
Ralph

2008/10/31 Graciliano Antonio Damazo [EMAIL PROTECTED]

   Caros amigos da lista, tenho uma questao simples de probabilidade que
 resultou numa discussao na resolução da mesma numa aula de reforço que eu
 estava estagiando la vai...mas não vale rirrsrs(brincadeira):

 1) Dois times A e B jogam 6 partidas entre si. Qual a probabilidade do time
 A vencer 4 dessas partidas?

 Gostaria de saber como vocês interpretam essa questão. Muito obrigado pela
 atenção desde já.

  --
 Novos endereços, o Yahoo! que você conhece. Crie um email 
 novohttp://br.rd.yahoo.com/mail/taglines/mail/*http://br.new.mail.yahoo.com/addressescom
  a sua cara @
 ymail.com ou @rocketmail.com.



Re: [obm-l] Série de Maclaurin

2008-10-29 Por tôpico Ralph Teixeira
Sim.

On Wed, Oct 29, 2008 at 10:59 AM, Denisson [EMAIL PROTECTED] wrote:

 Se eu escrever a função ln(1+y) como uma série de maclaurin e fizer depois
 y = x^3 eu obtenho uma representação de ln(1+x^3)?

 Obrigado


 --
 Denisson




Re: [obm-l] Série de Maclaurin

2008-10-29 Por tôpico Ralph Teixeira
Desculpa a resposta curta, hoje foi um dia cumprido... :)

Mas basicamente, eh isso que voce falou -- eu vi que voce tomou cuidado e
disse que, por uma substituicao dessas, voce obtem uma REPRESENTACAO de
alguma coisa, nao necessariamente uma outra serie de MacLaurin. Entao tah
certo, respeitado o raio de convergencia na expressao nova. Por exemplo:

ln(1+x)=1-x+x^2/2-x^3/3+x^4/4... desde que |x|1. Se voce botar KOISA no
lugar de x, desde que |KOISA|1, tah valendo! Por exemplo, como voce disse,
trocando x por lnx, vem:

ln(1+lnx)=1-lnx+(lnx)^2/2-(lnx)^3/3... desde que |lnx|1, ou seja, desde que
1/exe.

Abraco,
 Ralph

2008/10/29 Denisson [EMAIL PROTECTED]

 Obrigado Ralph,

 Na verdade eu verifiquei isso na mão por isso perguntei se estaria
 correto, mas em geral qualquer tipo de substituição é válida ou algumas
 condições devem ser satisfeitas? se substituir por x = ln(x) Ou qualquer
 outra função ainda se tornará válido?



 2008/10/29 Ralph Teixeira [EMAIL PROTECTED]

 Sim.


 On Wed, Oct 29, 2008 at 10:59 AM, Denisson [EMAIL PROTECTED] wrote:

 Se eu escrever a função ln(1+y) como uma série de maclaurin e fizer
 depois y = x^3 eu obtenho uma representação de ln(1+x^3)?

 Obrigado


 --
 Denisson





 --
 Denisson




Re: [obm-l] Re: [obm-l] PA com Função do 1 º grau.

2008-10-28 Por tôpico Ralph Teixeira
Eu tambem nao gosto muito da redacao do problema -- o que exatamente eh
C(t)?

Como o enunciado diz que C(t) eh o numero de cigarros vendidos anualmente,
minha interpretacao foi a mesma de voces: C(0) seria o numero de cigarros
vendidos em 1980, C(1) o numero em 1981, etc. Entao a resposta seria 107
trilhoes mesmo.

Mas algo me chamou a atencao: por que eles estao dizendo tao claramente que
t eh medido desde o *INICIO* de 1980? Afinal, com a nossa interpretacao, t
soh faz sentido se for inteiro, e representa o total de cigarros no ano,
entao t eh simplesmente medido em anos desde 1980... Cada ano seria um
bloco indivisivel, entao nao faria sentido tentar falar de inicio, meio
ou fim de um ano...

Entao acho que o que eles queriam eh o seguinte: que C(t) seja a TAXA DE
VENDAS DE CIGARROS, em trilhoes de cigarros POR ANO, NO INSTANTE t (que eh
quase a mesma coisa que numero de cigarros vendidos anualmente no instante
t, mas acho que a palavra TAXA eh essencial; e sim, faz sentido falar em
taxa por ano num instante, o por ano eh soh a unidade de t). Agora faria
sentido tomar t sendo um numero nao inteiro! Se C(t) fosse isso, a resposta
seria:

Integral (t variando de 0 a 20) C(t) dt = Int (0 a 20) 0,1t+4,4 dt =
0,1.t^2/2+4,4t (t de 0 a 20) = 20 + 88 = 108 trilhoes de cigarros.

Veja bem, eu discordo desta segunda interpretacao -- repito, esta
interpretacao poe C(t) como TAXA de vendas em cigarros/ano, nao como NUMERO
de cigarros (que eh o que eles escrevem explicitamente no enunciado).
Pessoalmente, se era isso que eles queriam, acho que o enunciado nao estah
claro o suficiente...

Abraco,
   Ralph

2008/10/28 João Luís [EMAIL PROTECTED]

  Isso. Mas o 2000 não conta, né? é do início de 1980 até o final de 1999,
 então o ano 2000 tá fora.

 Na minha opinião, esse problema deve ser resolvido assim:
   C(0) = 0,1*0 + 4,4 (trilhões de cigarros vendidos em 1980)
 C(1) = 0,1*1 + 4,4 (trilhões de cigarros vendidos em 1981)
 C(2) = 0,1*2 + 4,4 (trilhões de cigarros vendidos em 1982)
 .
 .
 .
 C(19) = 0,1*19 + 4,4 (trilhões de cigarros vendidos em 1999)

 Observe que essa soma será então 0,1*(0+1+2+ ... +18+19) + 20*4,4 = 0,1*190
 + 88 = 107 trilhões de cigarros.

 É ISSO O QUE ESTÁ SENDO PEDIDO NO ENUNCIADO.

 Na minha opinião, esse gabarito está errado.

 Você conhece a origem dessa questão?

 Um abraço,

 João Luís.
 .

  - Original Message -
 *From:* Gustavo Duarte [EMAIL PROTECTED]
 *To:* obm-l@mat.puc-rio.br
   *Sent:* Tuesday, October 28, 2008 12:01 PM
 *Subject:* [obm-l] Re: [obm-l] Re: [obm-l] Re: [obm-l] Re: [obm-l] PA com
 Função do 1 º grau.

 Valeu  João !!! pensei assim: *1980* = C (0) =4,4 ;  *1981* = C(1) =4,5 :
 *1982* = C (2) = 4,6 ;  . ;  *1999 *= c (19) = 6,3 ; *2000* = C ( 20)
 = 6,4. fiz até 2000, pois ele fala no* final* de 1999. favor observar no
 final deste e-mail a fórmula que usei. Até mais

 - Original Message -
 *From:* João Luís [EMAIL PROTECTED]
 *To:* obm-l@mat.puc-rio.br
 *Sent:* Tuesday, October 28, 2008 1:16 PM
 *Subject:* [obm-l] Re: [obm-l] Re: [obm-l] Re: [obm-l] PA com Função do 1
 º grau.

 Veja Gustavo, na verdade eu não resolvi o problema, apenas disse que essa
 PA deve ter 20 termos, pois são 20 anos, concorda? 1980 (inclusive) a 1999
 (inclusive).

 De onde você tirou que an = 6,4? Não entendi. DEsculpe-me se estou deixando
 passar algo óbvio, pois estou analisando as coisas meio na pressa aqui.

 Mais tarde, olho pro problema com toda a calma do mundo e posto uma
 mensagem aqui, ok?

 Um abraço a todos,

 João Luís.

 - Original Message -
 *From:* Gustavo Duarte [EMAIL PROTECTED]
 *To:* obm-l@mat.puc-rio.br
 *Sent:* Tuesday, October 28, 2008 10:11 AM
 *Subject:* [obm-l] Re: [obm-l] Re: [obm-l] PA com Função do 1 º grau.

 João, desculpe eu não ter entendido,para mim ainda não ficou claro... essa
 PA tem 20 termos é isso ?, qual seria o primeiro e qual seraia o último,
 usando na fórmula dá certo ?? onde estou errando ? pensei assim...se a1 =
 4,4  , a razão é 0,1 e an = 6,4 , n = ?? não seria 21 ? porém por outro lado
 vc tem razão ,do início de 1980 ate o final de 1999 realmente são 20 anos, O
 GABARITO TÀ CERTO È 108 MESMO ??  desde ja agradeço pela atenção !!

 - Original Message -
 *From:* João Luís [EMAIL PROTECTED]
 *To:* obm-l@mat.puc-rio.br
 *Sent:* Tuesday, October 28, 2008 7:30 AM
 *Subject:* [obm-l] Re: [obm-l] PA com Função do 1 º grau.

 De 1980 a 1999, são realmente 20 termos: 1999 - 1979 = 20.

 E porque  - 1979? Porque, como o 1980 está incluido nas condições do
 problema, não deve ser subtraído.

 Um abraço a todos,

 João Luís.



 - Original Message -
 *From:* Gustavo Duarte [EMAIL PROTECTED]
 *To:* Olimpíada obm-l@mat.puc-rio.br
 *Sent:* Monday, October 27, 2008 10:59 PM
 *Subject:* [obm-l] PA com Função do 1 º grau.

 O número em trilhões de cigarros vendiddos, anualmente, é dado pela função
 C(t) = 0,1t + 4,4  , com t=0 correspondendo a 1980 ( contado a partir *do
 início de* 1980). Qual o número 

Re: [obm-l] TEORIA DOS JOGOS!

2008-10-27 Por tôpico Ralph Teixeira
mas entao B pensa que, se negociar com A aquele contrato de 6000, talvez
consiga mais de 2000. :)

Nao sei qual eh a resposta, se eh que ha uma. Vou chutar que **um**
equilibrio eh (estamos supondo que A, B e C ou fazem isso ou nada, e o
trabalho nao lhes custa nada):
Contrato 1: A:2 B:4
Contrato 2: A:2 C:6
Contrato 3: B:6 C:4

Se as propostas estiverem assim, ninguem tem incentivo para tentar mudar.
Afinal:
A nao vai conseguir mais de 2 -- se ele tentar, em qualquer contrato, B e C
fecham o contrato 3 com a divisao 6:4, e eh melhor para AMBOS.
B nao vai conseguir mais de 4 -- se ele tentar, em qualquer contrato, A e C
fecham o contrato 2 com a divisao 2:6, e eh melhor para AMBOS.
C nao vai conseguir mais de 6 -- se ele tentar, em qualquer contrato, A e B
fecham o contrato 1 com a divisao 2:4, e eh melhor para AMBOS.

Neste equilibrio, nao vejo como decidir qual dos contratos vencerah, jah que
eles sao equivalentes para A, B, C e tambem para o comprador (afinal, acho
que os valores que ele se dispoe a pagar jah refletem suas preferencias).

Abraco,
Ralph
2008/10/27 Chicao Valadares [EMAIL PROTECTED]

 Como todos sao racionais eles sabem que eles sao competidores entre si e
 devem tentar fechar a melhor proposta o mais rapido possivel e logo percebem
 que B deve conversar com C primeiro. Se  B  nao conseguir negociar com C ai
 B vai ter problemas porque A poderá negociar com C, entao B ficar a ver
 navios. Tanto B quanto C sabem disso por isso B nao deve propor metade do
 valor a C. C fica esperando B fazer uma boa proposta, tal que C não precise
 mais negociar.
 B então oferece a C a possibilidade dele ficar com 8000 e B ficar apenas
 com 2000. Ambos aceitam e sai satisfeitos já que é impossivel C conseguir
 isso com A só se A abrisse mão do seu ganho. Além disso B adota a politica
 do é melhor um passaro na mao que dois voando só deus sabe quando pois é
 melhor ficar com 2000 do que ficar com nada.


 --- Em seg, 20/10/08, Jorge Luis Rodrigues e Silva Luis 
 [EMAIL PROTECTED] escreveu:

  Um agente de teatro escreve a três artistas, informando-os
  de que tem trabalho para dois deles, para quaisquer dois. Os
  três artistas são de forma desigual e, assim, o empregador
  se dispõe a pagar mais por algumas combinações do que por
  outras. Especificamente, A e B podem conseguir $6000; A e C
  podem conseguir $8000 e B e C podem conseguir $1. Os
  dois que obtiverem o trabalho poderão dividir a soma
  recebida da maneira que mais lhe agradem, mas devem decidir
  como dividi-la antes de aceitar o trabalho. Os dois que
  primeiro chegam a um acordo conseguirão o emprego. É
  possível prever que par conseguirá o emprego? Como
  dividirão os lucros?
 
  Nota: Nesse jogo, a primeira reação é a de presumir que
  B e C se associarão. Como dividirão o que ganhem é outra
  questão...
 
  Abraços!
  _
  Confira vídeos com notícias do NY Times, gols direto do
  Lance, videocassetadas e muito mais no MSN Video!
  http://video.msn.com/?mkt=pt-br


  Novos endereços, o Yahoo! que você conhece. Crie um email novo com a
 sua cara @ymail.com ou @rocketmail.com.
 http://br.new.mail.yahoo.com/addresses

 =
 Instruções para entrar na lista, sair da lista e usar a lista em
 http://www.mat.puc-rio.br/~obmlistas/obm-l.html
 =



Re: [obm-l] Probabilidade - Caixa sem Troco

2008-10-21 Por tôpico Ralph Teixeira
Oi, Luiz e Tarso.

Dêem uma olhada em:

http://www.mail-archive.com/obm-l@mat.puc-rio.br/msg17532.html

Abraço,
 Ralph

2008/10/21 luiz silva [EMAIL PROTECTED]

Uma determinada atração custa R$ 5. Temos 2n pessoas em fila, sendo que
 n possuem uma nota de R$ 5 e as outras n possuem uma nota de R$ 10. Qual a
 probabilidade desta fila dar problema ? (ou seja, o caixa ficar sem troco em
 um dado momento)


 Abs
 Felipe.


  --
 Novos endereços, o Yahoo! que você conhece. Crie um email 
 novohttp://br.rd.yahoo.com/mail/taglines/mail/*http://br.new.mail.yahoo.com/addressescom
  a sua cara @
 ymail.com ou @rocketmail.com.



Re: [obm-l] Concurso CMS-2008

2008-10-20 Por tôpico Ralph Teixeira
Eh isso mesmo, o *MENOR* numero de grupos? Que tal 1 grupo soh, com todo
mundo dentro? Se for isso, resposta 264+168=432, questao anulada...

Agora, se for o *MAIOR* numero de grupos, ou o *MENOR* numero de alunos por
turma, entao eu fico com a resposta do Vidal: mdc(264,168)=24 grupos, cada
um com 11+7=18 alunos, letra B.

Abraco,
 Ralph

On Sun, Oct 19, 2008 at 9:21 PM, Ojesed Mirror [EMAIL PROTECTED] wrote:

 Prova do Colégio Militar de Salvador para admissão ao 6o ano do ensino
 fundamental - 2008

 Questão 06
 No colégio MATEMÁGICO existem 264 meninos e 168 meninas. Se grupos forem
 formados de maneira que todos eles fiquem com a mesma quantidade de meninos
 e a mesma quantidade de meninas, a quantidade de alunos (meninos e meninas)
 por grupo, de modo que se tenha o menor número de grupos, é: A-17, B-18,
 C-21, D-24, E-36.

 A resposta do gabarito, publicado hoje é letra B, mas meu filho achou que o
 correto seria a letra D.
 Vou entrar com um recurso para correção do gabarito, mas gostaria de saber
 a opinião dos professores desta lista, se existe alguma interpretação ou
 argumento que sustente a resposta oficial dada.

 Ojesed.
 =
 Instru�ões para entrar na lista, sair da lista e usar a lista em
 http://www.mat.puc-rio.br/~obmlistas/obm-l.html
 =



Re: [obm-l] DESFECHO

2008-10-11 Por tôpico Ralph Teixeira
Suponha que ha n andares. Entao sao C(n,2) pares de andares.

Cada elevador conecta C(6,2)=15 pares de andares; como ha 7 deles, eles
conectam, no maximo, 7.15=105 pares (isto se nao houvesse nenhum par de
elevadores que conectasse os mesmos dois andares). Entao:

C(n,2)=105, isto eh, n(n-1)=210, n=15.

Agora, se fossem 15 andares, fica na conta certa que C(n,2)=105=C(6,2).7;
entao nao poderia haver dois elevadores que conectassem o mesmo par de
andares... Mas isso eh impossivel! Afinal, se o primeiro elevador para nos
andares x1,x2,x3,x4,x5,x6, precisamos de um segundo elevador que conecte x1
a x7; como nenhum par de andares pode se repetir neste segundo elevador,
este segundo elevador para nos andares x1,x7,x8,x9,x10,x11 (onde
x7,x8,x9,x10,x11 sao disjuntos dos x1,x2,x3,x4,x5,x6); mas ainda precisamos
de um terceiro elevador que conecte x1 a x12, e ai temos
x1,x12,x13,x14,x15,x??? (este ultimo vai repetir algum par x1-x? jah servido
anteriormente!).

Entao o numero de andares nao pode ser mais que 14... Agora temos bastante
folga, vejamos se conseguimos uma configuracao dos 7 elevadores servindo
esses 14 andares. Com adivinhacao, chutometro e bom senso, consigo construir
isto  (eu basicamente separei os 14 andares em 7 pares, e consegui agrupar 3
pares em cada elevador de forma satisfatoria; dei um pouco de sorte
tambem...):

ELEVADOR: Lista-dos-andares-que-ele-serve
A: 1-2-3-4-5-6
B: 1-2-7-8-9-10
C: 1-2-11-12-13-14
D: 3-4-7-8-11-12
E: 3-4-9-10-13-14
F: 5-6-7-8-13-14
G: 5-6-9-10-11-12

Ok, esses 7 elevadores conectam todos os pares de andares (confira no braco
mesmo).

Entao o numero maximo de andares eh 14 mesmo.

Abraco
  Ralph

On Sat, Oct 11, 2008 at 1:55 PM, arkon [EMAIL PROTECTED] wrote:

 *Pessoal, peguei esta questão nos arquivos aqui do site, porém, não encontrei 
 o desfecho, ou seja, o resultado.
  Alguém, pode resolver, por favor.

 Num prédio de apartamentos há 7 elevadores que param em não mais que 6 
 andares. **É possível ir de um andar a qualquer outro sem trocar de elevador. 
 Qual é o número máximo de
 andares que esse prédio pode ter?
 **(RPM/IME/USP)






 *

 =
 Instru�ões para entrar na lista, sair da lista e usar a lista em
 http://www.mat.puc-rio.br/~obmlistas/obm-l.html=


Re: [obm-l] Variável de Inequação em Evidência

2008-10-11 Por tôpico Ralph Teixeira
Oi, Henrique.

O grafico de x.e^x tem que ser desenhado para voce ver isso... Deixa eu ver
se consigo coloca-lo em anexo aqui... Ok, acho que foi.

Entao, marque por exemplo y=-0.2. Pelo grafico, x.e^x-0.2 significa que x
estah ENTRE as raizes da equacao x.e^x=-0.2, isto eh, entre W_{-1}(-0.2) e
W(-0.2). Olhe no grafico!

Abraco,
 Ralph
2008/10/11 Henrique Rennó [EMAIL PROTECTED]

 Olá Ralph, bom dia!
 Estou com outra dúvida na sua resolução. Obrigado novamente!

 On Wed, Oct 8, 2008 at 12:26 AM, Ralph Teixeira [EMAIL PROTECTED] wrote:

  Huh, basicamente, nao dah, a menos que voce use a funcao W de Lambert
 (ou nos de mais alguma informacao sobre n).

 A definicao desta funcao W eh mais ou menos assim: seja f(x)=x.e^x (faca o
 grafico dela se puder, ajuda a enxergar o resto). Como f`(x)=(x+1)e^x, a
 funcao f(x) eh crescente para x=-1. Entao a funcao f(x) eh bijetiva de
 [-1,+Inf) em [-1/e,+Inf). Bom, a funcao W eh a inversa desta bijecao. Em
 outras palavras, dado y=-1/e, temos x=W(y) se, e somente se, x.e^x=y (e
 tambem x=-1).

 (Se esta definicao por funcao inversa incomoda, tente definir precisamente
 a funcao raiz quadrada e a funcao logaritmo...)

 Por exemplo, W(0)=0 (pois f(0)=0.e^0=0), W(e)=1 (pois 1.e^1=e), W(2e^2)=2
 e W(-1/e)=-1... Note que, como f eh crescente, entao W eh crescente tambem.

 Note tambem que, para x em (-Inf,-1], a funcao f(x) eh decrescente (e
 f(-Inf)=0). Assim, a equacao x.e^x=y tem uma segunda solucao alem de W(y)
 sempre que -1/ey0; esta segunda solucao eh chamada W_{-1}(y). Em outras
 palavras, quando -1/ey0, W(y) serah a solucao maior que -1, e W_{-1}(y)
 eh a solucao menor que -1.

 Bom, agora eh facil:

 k.2^k  n   =  (k.ln2).e^(k.ln2)  n.ln2

 Entao, ha 3 casos:

 i) Se n.ln2=0, a solucao eh k.ln2  W(n.ln2), isto eh, kW(n.ln2)/ln2
 ii) Se -1/en.ln20, entao a solucao eh W_{-1}(n.ln2)k.ln2W(n.ln2)...


 Não entendi o caso ii. A solução não seria dada por: -1k.ln20, ou seja,
 aplicando W em toda a inequação? Não entendi porque
 seria W_{-1}(n.ln2)k.ln2W(n.ln2).


  iii) Se n.ln2=-1/e, nao ha solucoes para a desigualdade.

 Nao sei se eh uma resposta huh, satisfatoria... mas eh a melhor que
 eu arrumo sem maiores restricoes ao enunciado.

 Abraco,
  Ralph

 On Tue, Oct 7, 2008 at 11:21 AM, Henrique Rennó [EMAIL PROTECTED]
  wrote:

 Bom dia!

 Como k poderia ser colocado em função de n na seguinte inequação?

 k.2^k  n

 Obrigado

 --
 Henrique





 --
 Henrique

attachment: XEX.png

Re: [obm-l] Probabilidade!

2008-10-08 Por tôpico Ralph Teixeira
Argh, escrevi uma besteira! Tem um erro no meu raciocínio, no denominador
daquela probabilidade!

Explico: do jeito que eu estava pensando no problema, **não interessa** de
quantos jeitos a CEF pode sortear as bolinhas -- eu estava fingindo que eles
**já sortearam** as r bolinhas, e a gente tem que descobrir de quantas
maneiras EU posso fazer a minha cartelinha de maneira que EU acerte s
palpites. Bom, EU tenho C(N,p) maneiras de fazer minha cartelinha, então o
denominador é C(N,p); como EU tenho que escolher s das r sorteadas, e p-s
das N-r não sorteadas, temos:

Prob(N,r,p,s) = C(N-r,p-s).C(r,s)/C(N,p) (agora com denominador correto!)

Antes que alguém reclame, existe sim uma segunda maneira de pensar, que é
assim: eu JÁ FIZ a minha cartelinha com p palpites, e agora a CEF é que vai
sortear r bolinhas. ELES é que tem que acertar exatamente s das MINHAS
bolinhas. Bom, se você pensar assim, então, agora sim, eles têm, C(N,r)
possibilidades de sorteio e este é o denominador... mas, para ELES
acertarem exatamente s bolinhas das minhas, ELES tem que sortear s das p da
minha cartelinha, e ELES têm que sortear as outras r-s das N-p que eu não
escolhi. Então a probabilidade é:

Prob = C(p,s).C(N-p,r-s)/C(N,r)

É divertido escrever tudo em forma de fatoriais e notar que ambas as
fórmulas são, de fato, a mesma coisa. Ler ambos os raciocínios é legal, pois
eles mostram que os papéis de p e s são intercambiáveis -- há N=100
bolinhas no total, e você quer acertar s=7; então tanto faz você fazer uma
cartelinha com 15 e eles sortearem 10 ou você fazer uma cartelinha com 10 e
eles sortearem 15, a chance de você acertar 7 é a mesma!

 Para ajudar a lembrar tais fórmulas, note que a soma dos dois primeiros
índices das combinações do numerador dá o primeiro índice da combinação
do denominador, e o mesmo para os segundos índices. Em suma, (N-r)+r=N e
(p-s)+s=p na primeira fórmula, ou p+(N-p)=N e s+(r-s)=r na segunda fórmula.
Quando eu fiz seu problema, eu conferi isso na resposta numérica (que está
correta por acidente, pois p=r no seu problema), mas esqueci de conferir na
minha resposta com letrinhas!

Abraço,
 Ralph
2008/10/7 jose silva [EMAIL PROTECTED]

  Valeu! Parabens! Essa questao era mais interessante do que eu
 imaginava!
 Como eu havia dito: parece dificil, mas nao e facil.
 Muito obrigado!
 jccardosos


 --

 Date: Tue, 7 Oct 2008 20:27:10 -0300
 From: [EMAIL PROTECTED]
 To: obm-l@mat.puc-rio.br
 Subject: Re: [obm-l] Probabilidade!


  A palavra chave para procurar no Google eh distribuicao hipergeometrica
 (hypergometric distribution). Funciona assim:

 Suponha que ha N bolas numeradas numa caixa, das quais r serao sorteadas
 (digamos, pela CEF); voce faz uma escolha de p delas. Qual a chance de
 acertar exatamente s? (Eh, tem 4 variaveis ai: N,p,r,s)

 Resposta: sao C(N,r) possibilidades de sorteio pela CEF. Para voce acertar
 EXATAMENTE s, voce tem que escolher s das r sorteadas E TEM QUE ESCOLHER
 p-s DAS N-r NAO SORTEADAS (eh isto que estah faltando nas outras solucoes).
 Entao a resposta eh:

 Prob(Acertar exatamente s) = C(N-r,p-s).C(r,s) / C(N,r)  (ARGH! NÃO É r
 AQUI EMBAIXO NÃO!)

 Para N=25, r=15, p=15 e s=11, que eh o caso que voce propoe, dah:

 Prob = C(10,4).C(15,11) / C(25,15) = ...

 Abraco,
 Ralph

 2008/10/7 jose silva [EMAIL PROTECTED]

   Concordo com a soluçao do amigo Jose Airton, porem a soluçao nao
 esta batendo, por exemplo de acertar 11 numeros e igual a 1/11,  conforme
 esta disponivel no endereço:
 www1.caixa.gov.br/loterias/loterias/lotofacil/probabilidades.asp .
 jccardosos

 --

 Date: Sat, 4 Oct 2008 12:15:10 -0300
 From: [EMAIL PROTECTED]
 To: obm-l@mat.puc-rio.br
 Subject: Re: [obm-l] Probabilidade!

 Jose Airton e Leandro,

 Foi mal.  Eu, equivocadamente, imagnei que as perguntas fossem qual a
 probabilidade de ALGUM dos alunos e não de UM qualquer dos alunos...
 Bobeira,
 Nehab

 JOSE AIRTON CARNEIRO escreveu:

 José vou te quebrar o galho.
 Para acertar as 15:  P(A) = n(A)/n(U) = C15,15 / C25,15 = 1/3268760.
 Para acertar 14 : P(A) = C15,14 / C25,15.
 Para 13 P(A) = C15,13 /C25,15 e assim por diante ..


 2008/10/2, Carlos Nehab [EMAIL PROTECTED]:

 Oi, Leandro.
 Quantos alunos?
 Nehab

 LEANDRO L RECOVA escreveu:

 Acho que voce resolve isso usando a distribuicao binomial.

 From: jose silva [EMAIL PROTECTED] [EMAIL PROTECTED]
 Reply-To: obm-l@mat.puc-rio.br
 To: obm-l@mat.puc-rio.br obm-l@mat.puc-rio.br
 Subject: [obm-l] Probabilidade!
 Date: Thu, 2 Oct 2008 02:45:49 +

 Em uma escola é feita uma atividade lúdica, envolvendo a aplicação de
 probabilidades. Durante a aula, coloca-se dentro de uma urna, 25 bolas
 marcadas com os números de 1 a 25. Em seguida, são distribuídos entre os
 alunos cartelas contendo estes números, em ordem crescente, ou seja, do
 número 1 ao 25. Após isso, pedem-se aos alunos para marcarem 15 números
 aleatoriamente, nesta cartela. Feito isso, qual a 

Re: [obm-l] Variável de Inequação em Evidência

2008-10-08 Por tôpico Ralph Teixeira
Oi, Henrique.

Bom, antes de mais nada, eu escrevo f(-Inf) por preguiça; o correto seria
limite, quando x tende a -Infinito, de f(x).  Então, enquanto você tem
razão que f(x)=x.e^x dá uma indeterminação do tipo (-Inf).0 quando x tende a
-Inf, re-escrevendo f(x)=x/e^(-x) vejo uma indeterminação do tipo -Inf/Inf,
e agora um L'Hôpital dá:
f(-Inf) = lim 1/(-e^(-x)) quando x- -Inf = 0.

Quanto ao ln2... o problema é que eu quero usar a função W, que foi definida
para resolver xe^x=y, com base e, e não outra. Então, se você me dá k.2^k=n,
eu me ferrei e tenho que tranformar a base, fazendo:

k.e^(k.ln2)=n

Mas agora eu me ferrei de outro jeito, porque a definição da função W era:

y=W(x) == x.e^x=y

E tem que ser o mesmo x multiplicando e em cima do e... Isso a gente
conserta com a pequena mágica de multiplicar tudo por ln2 e fazendo x=k.ln2.
Então fica assim:

k.ln2.e^(k.ln2)=n.ln2, ou seja, xe^x=n.ln2

Agora sim, uma das soluções disso é x=W(n.ln2)...espero que estas linhas
tenham ajudado a explicar o porquê de aparecer o misterioso ln2...

Depois que a gente passou por tudo isso, a parte mais difícil é lidar com a
DESIGUALDADE... Faça um gráfico de x.e^x (desce de (-Inf,0) a (-1,-1/e),
depois sobe para (0,0) e continua subindo para (+Inf,+Inf)), lembre que
W(z) é a maior solução de x.e^x=z e W_{-1}(z) é a menor solução, se houver
(desenhe a reta y=z para alguns valores de z e encontre estas soluções no
gráfico). Então, convença-se que a solução de x.e^xz é:

i)  Se z0, solução é xW(z)
ii) Se -1/ez0, solução é W_{-1}(z)xW(z)
iii) Se z=-1/e, conjunto solução é vazio.

Abraço,
Ralph
2008/10/8 Henrique Rennó [EMAIL PROTECTED]

 Olá Ralph, boa tarde!

 Obrigado pela explicação. Estou com dúvida em duas partes.

  On Tue, Oct 7, 2008 at 11:26 PM, Ralph Teixeira [EMAIL PROTECTED]wrote:

  Huh, basicamente, nao dah, a menos que voce use a funcao W de Lambert
 (ou nos de mais alguma informacao sobre n).

 A definicao desta funcao W eh mais ou menos assim: seja f(x)=x.e^x (faca o
 grafico dela se puder, ajuda a enxergar o resto). Como f`(x)=(x+1)e^x, a
 funcao f(x) eh crescente para x=-1. Entao a funcao f(x) eh bijetiva de
 [-1,+Inf) em [-1/e,+Inf). Bom, a funcao W eh a inversa desta bijecao. Em
 outras palavras, dado y=-1/e, temos x=W(y) se, e somente se, x.e^x=y (e
 tambem x=-1).

 (Se esta definicao por funcao inversa incomoda, tente definir precisamente
 a funcao raiz quadrada e a funcao logaritmo...)

 Por exemplo, W(0)=0 (pois f(0)=0.e^0=0), W(e)=1 (pois 1.e^1=e), W(2e^2)=2
 e W(-1/e)=-1... Note que, como f eh crescente, entao W eh crescente tambem.

 Note tambem que, para x em (-Inf,-1], a funcao f(x) eh decrescente (e
 f(-Inf)=0). Assim, a equacao x.e^x=y tem uma segunda solucao alem de W(y)
 sempre que -1/ey0; esta segunda solucao eh chamada W_{-1}(y). Em outras
 palavras, quando -1/ey0, W(y) serah a solucao maior que -1, e W_{-1}(y)
 eh a solucao menor que -1.


 Onde você diz que a função f(x) é decrescente você coloca que f(-inf) = 0,
 ou seja, -inf.e^(-inf) = -inf/e^(inf) = -inf/inf. Essa fração não seria uma
 indeterminação e não 0 como você mencionou?



 Bom, agora eh facil:

 k.2^k  n   =  (k.ln2).e^(k.ln2)  n.ln2


 Não entendi essa relação acima. Por que ln2 multiplicando o expoente de e?



 Entao, ha 3 casos:

 i) Se n.ln2=0, a solucao eh k.ln2  W(n.ln2), isto eh, kW(n.ln2)/ln2
 ii) Se -1/en.ln20, entao a solucao eh W_{-1}(n.ln2)k.ln2W(n.ln2)...
 iii) Se n.ln2=-1/e, nao ha solucoes para a desigualdade.

 Nao sei se eh uma resposta huh, satisfatoria... mas eh a melhor que
 eu arrumo sem maiores restricoes ao enunciado.

 Abraco,
  Ralph

 On Tue, Oct 7, 2008 at 11:21 AM, Henrique Rennó [EMAIL PROTECTED]
  wrote:

 Bom dia!

 Como k poderia ser colocado em função de n na seguinte inequação?

 k.2^k  n

 Obrigado

 --
 Henrique



 --
 Henrique



Re: [obm-l] Probabilidade!

2008-10-07 Por tôpico Ralph Teixeira
A palavra chave para procurar no Google eh distribuicao hipergeometrica
(hypergometric distribution). Funciona assim:

Suponha que ha N bolas numeradas numa caixa, das quais r serao sorteadas
(digamos, pela CEF); voce faz uma escolha de p delas. Qual a chance de
acertar exatamente s? (Eh, tem 4 variaveis ai: N,p,r,s)

Resposta: sao C(N,r) possibilidades de sorteio pela CEF. Para voce acertar
EXATAMENTE s, voce tem que escolher s das r sorteadas E TEM QUE ESCOLHER
p-s DAS N-r NAO SORTEADAS (eh isto que estah faltando nas outras solucoes).
Entao a resposta eh:

Prob(Acertar exatamente s) = C(N-r,p-s).C(r,s) / C(N,r)

Para N=25, r=15, p=15 e s=11, que eh o caso que voce propoe, dah:

Prob = C(10,4).C(15,11) / C(25,15) = ...

Abraco,
Ralph

2008/10/7 jose silva [EMAIL PROTECTED]

   Concordo com a soluçao do amigo Jose Airton, porem a soluçao nao
 esta batendo, por exemplo de acertar 11 numeros e igual a 1/11,  conforme
 esta disponivel no endereço:
 www1.caixa.gov.br/loterias/loterias/lotofacil/probabilidades.asp .
 jccardosos

 --


 Date: Sat, 4 Oct 2008 12:15:10 -0300
 From: [EMAIL PROTECTED]
 To: obm-l@mat.puc-rio.br
 Subject: Re: [obm-l] Probabilidade!

 Jose Airton e Leandro,

 Foi mal.  Eu, equivocadamente, imagnei que as perguntas fossem qual a
 probabilidade de ALGUM dos alunos e não de UM qualquer dos alunos...
 Bobeira,
 Nehab

 JOSE AIRTON CARNEIRO escreveu:

 José vou te quebrar o galho.
 Para acertar as 15:  P(A) = n(A)/n(U) = C15,15 / C25,15 = 1/3268760.
 Para acertar 14 : P(A) = C15,14 / C25,15.
 Para 13 P(A) = C15,13 /C25,15 e assim por diante ..


 2008/10/2, Carlos Nehab [EMAIL PROTECTED]:

 Oi, Leandro.
 Quantos alunos?
 Nehab

 LEANDRO L RECOVA escreveu:

 Acho que voce resolve isso usando a distribuicao binomial.

 From: jose silva [EMAIL PROTECTED] [EMAIL PROTECTED]
 Reply-To: obm-l@mat.puc-rio.br
 To: obm-l@mat.puc-rio.br obm-l@mat.puc-rio.br
 Subject: [obm-l] Probabilidade!
 Date: Thu, 2 Oct 2008 02:45:49 +

 Em uma escola é feita uma atividade lúdica, envolvendo a aplicação de
 probabilidades. Durante a aula, coloca-se dentro de uma urna, 25 bolas
 marcadas com os números de 1 a 25. Em seguida, são distribuídos entre os
 alunos cartelas contendo estes números, em ordem crescente, ou seja, do
 número 1 ao 25. Após isso, pedem-se aos alunos para marcarem 15 números
 aleatoriamente, nesta cartela. Feito isso, qual a probabilidade de após a
 retirada aleatória e sem reposição, de 15 bolas consecutivas desta urna, de
 um dos estudantes acertar os 15 números? De um dos alunos acertar 14 ou 13
 ou 12 ou  11 números?


 =
 Instruções para entrar na lista, sair da lista e usar a lista em
 http://www.mat.puc-rio.br/~obmlistas/obm-l.html=


 =
 Instruções para entrar na lista, sair da lista e usar a lista em
 http://www.mat.puc-rio.br/~obmlistas/obm-l.html=
 --
 Conheça já o Windows Live Spaces, o site de relacionamentos do Messenger! Crie
 já o seu! http://www.amigosdomessenger.com.br/




Re: [obm-l] Variável de Inequação em Evidência

2008-10-07 Por tôpico Ralph Teixeira
Huh, basicamente, nao dah, a menos que voce use a funcao W de Lambert (ou
nos de mais alguma informacao sobre n).

A definicao desta funcao W eh mais ou menos assim: seja f(x)=x.e^x (faca o
grafico dela se puder, ajuda a enxergar o resto). Como f`(x)=(x+1)e^x, a
funcao f(x) eh crescente para x=-1. Entao a funcao f(x) eh bijetiva de
[-1,+Inf) em [-1/e,+Inf). Bom, a funcao W eh a inversa desta bijecao. Em
outras palavras, dado y=-1/e, temos x=W(y) se, e somente se, x.e^x=y (e
tambem x=-1).

(Se esta definicao por funcao inversa incomoda, tente definir precisamente a
funcao raiz quadrada e a funcao logaritmo...)

Por exemplo, W(0)=0 (pois f(0)=0.e^0=0), W(e)=1 (pois 1.e^1=e), W(2e^2)=2 e
W(-1/e)=-1... Note que, como f eh crescente, entao W eh crescente tambem.

Note tambem que, para x em (-Inf,-1], a funcao f(x) eh decrescente (e
f(-Inf)=0). Assim, a equacao x.e^x=y tem uma segunda solucao alem de W(y)
sempre que -1/ey0; esta segunda solucao eh chamada W_{-1}(y). Em outras
palavras, quando -1/ey0, W(y) serah a solucao maior que -1, e W_{-1}(y)
eh a solucao menor que -1.

Bom, agora eh facil:

k.2^k  n   =  (k.ln2).e^(k.ln2)  n.ln2

Entao, ha 3 casos:

i) Se n.ln2=0, a solucao eh k.ln2  W(n.ln2), isto eh, kW(n.ln2)/ln2
ii) Se -1/en.ln20, entao a solucao eh W_{-1}(n.ln2)k.ln2W(n.ln2)...
iii) Se n.ln2=-1/e, nao ha solucoes para a desigualdade.

Nao sei se eh uma resposta huh, satisfatoria... mas eh a melhor que eu
arrumo sem maiores restricoes ao enunciado.

Abraco,
 Ralph

On Tue, Oct 7, 2008 at 11:21 AM, Henrique Rennó [EMAIL PROTECTED]wrote:

 Bom dia!

 Como k poderia ser colocado em função de n na seguinte inequação?

 k.2^k  n

 Obrigado

 --
 Henrique



Re: [Fwd: [obm-l] geometria...fórmula da mediana????]

2008-10-03 Por tôpico Ralph Teixeira
Pro Nehab:

Sejam os lados a=b=c. A mediana eh b. Viu, eu lembro! ;)



Ah, *aquela* mediana Pera ai que eu vou perguntar pro Wagner :) :P

Quanto ao barril de chopp vou perguntar pro Gugu. ;) :) ;)

Abraco,
 Ralph

P.S.: Po, pior que eu lembro a... como era, Lei de Stewart?... algo que diz
que o comprimento de uma ceviana beta que determina comprimentos m e n sobre
o lado a satisfaz:
b^2/am - beta^2/mn + c^2/an = 1
(se botar as coisas na ordem certa)... Daqui sai mediana e outras
coisas...Ai, Nehab, se for para decorar, decora logo algo assim! ;) ;) ;)

P.S.2: Conferi na Wikipedia, acertei de primeira , ateh o nome do fulano
(viva! bom, eh **Teorema** de Stewart na Wikipedia:
http://en.wikipedia.org/wiki/Stewart's_theorem). Por outro lado, dei aulas
de Geometria de 1987 a 1991, entao tive mais chances de por essas coisas na
cabeca
On Fri, Oct 3, 2008 at 5:07 PM, Carlos Nehab [EMAIL PROTECTED] wrote:

 Não quero fofocar não, mas tão chamando neguinho de meia idade de velho...
 Mas qual a culpa dos referidos hoje senhores se nos livros em que eles
 estudaram, além de zilhões de exercícios ótimos para resolver, adoravam dar
 nomes óbvios às formuletas?
 Alguém por ai, com menos de 20 anos sabe, AGORA, sem colar e de cor,
 formuletas para cálculo das bissetrizes internas e externas de um
 triângulo?  Duvi-dê-ó-dó...  Oh, o vestiba tá rolando.  E como calcular o
 volume de um barril de chopp?
 Então, Olavo, tome de fórmuleta da mediana procê...

 Nehab :-P

  Mensagem original   Assunto: [obm-l] geometria...fórmula
 da mediana Data: Fri, 3 Oct 2008 18:49:02 + De: Antonio Neto
 [EMAIL PROTECTED] [EMAIL PROTECTED] Responder a:
 obm-l@mat.puc-rio.br Para: obm-l@mat.puc-rio.br obm-l@mat.puc-rio.br 
 Referências:
 [EMAIL PROTECTED][EMAIL PROTECTED]

Oi, acontece que as unicas pessoas que sabem essas formulas sao o Wagner
 e o Paulo Cesar. Por acaso sei deduzi-las, mas vamos mais devagar. Seja G o
 baricentro. Ponha GE = x e BC = 2x. Do mesmo modo, GD = y e GB = 2y. Use
 Pitagoras nos triangulos BEG e GCD, some tudo e seja o que Deus quiser. Se
 nao der certo, mas vai dar, eh soh escrever. Amplexos, olavo.

 Antonio *Olavo* da Silva Neto


 --
   Seja ABC um triângulo de lados BC, CA, AB cujas medidas são
 respectivamente iguais a a, b, c. Se D e E são os pontos médios de AC e AB
 respectivamente, mostre que a mediana BD é perpendicular a CE se, e somente
 se, b² + c² = 5a²


 É suficente usar a fórmula que fornece a mediana?

 Grato






 --
 Explore the seven wonders of the world Learn 
 more!http://search.msn.com/results.aspx?q=7+wonders+worldmkt=en-USform=QBRE
 =
 Instruções para entrar na lista, sair da lista e usar a lista em
 http://www.mat.puc-rio.br/~obmlistas/obm-l.html=


Re: [obm-l]

2008-09-29 Por tôpico Ralph Teixeira
Vou supor que as cordas (digamos, A, B e C) são distintas. Como você disse,
ele escolhe a corda e, a partir daí, o tiro está determinado. Então a
escolha total dele é algo como:

AABCCBCA

onde há 3 A´s, 2B´s e 3C´s nesta lista. Então a questão é: quantas
permutações destas 8 letras podemos fazer? A resposta é 8!/(3!2!3!)=560.

Abraço,
   Ralph

2008/9/29 Patricia Ruel [EMAIL PROTECTED]

 Um cabide contém 3 cordas penduradas. Presos à 1a corda, há 3 pratos, um em
 cima do outro; presos à 2a corda, há 2 pratos, também um em cima do outro,
 assim como na terceira corda, que contém os outros 3 pratos restantes. Um
 atirador resolve atirar nos 8 pratos da seguinte forma: escolhe uma corda e
 atira no primeiro prato inteiro de cima para baixo. De quantas formas o
 atirador pode quebrar os 8 pratos? (desconsidere tiros errados)




 --
 Receba GRÁTIS as mensagens do Messenger no seu celular quando você estiver
 offline. Conheça o MSN Mobile! Crie já o 
 seu!http://mobile.live.com/signup/signup2.aspx?lc=pt-br



Re: [obm-l] Integral

2008-09-24 Por tôpico Ralph Teixeira
Enquanto ha varias solucoes, para mim a mais facil eh fazer a substituicao
u=v+1, que simplifica o denominador um bocado, e seguir dai para a frente:

Int ((1-v)/(1+v)^2 dv) = Int ((1-(u-1))/u^2 du)=Int (2/u^2 - 1/u du) = -2/u
- ln |u| + C = -2/(v+1) - ln|v+1| + C.

Abraco,
  Ralph
On Wed, Sep 24, 2008 at 10:18 AM, warley ferreira [EMAIL PROTECTED]wrote:


 Queria saber como resolver essa integral
 Integral de*1- v * dv
(v+1)^2
 Obrigado
 Warley Souza

  --
 Novos endereços, o Yahoo! que você conhece. Crie um email 
 novohttp://br.rd.yahoo.com/mail/taglines/mail/*http://br.new.mail.yahoo.com/addressescom
  a sua cara @
 ymail.com ou @rocketmail.com.



Re: [obm-l] Caro Professor Ralph

2008-09-24 Por tôpico Ralph Teixeira
Estou supondo que os pratos, quando vazios, tem massas iguais.

Basicamente, depois que voce decidir onde coloca os pesos x1, x2 e x3 e
equilibrar a balanca, voce vai descobrir que a massa m eh
m=c1.x1+c2.x2+c3.x3 onde c1, c2, c3 estao no conjunto {-1,0,1} (ci=-1
significa que o peso xi estah no mesmo prato que a massa m, ci=1 significa
peso xi no outro prato, ci=0 significa que nao usamos aquele peso xi).

Note que ha 3^3=27 possiveis escolhas dos coeficientes, mas eu vou
coloca-las aos pares: para cada escolha {c1,c2,c3} ha uma escolha
{-c1,-c2,-c3}. Norte que, com isso, para cada combinacao que dah m, eu
arrumo uma combinacao que dah -m, o que indica que metade das combinacoes
sao inuteis (massas negativas nao existem) -- bom, para ser exato, 13
combinacoes dao massas negativas, jah que c1=c2=c3=0 fica pareada consigo
mesma e dah massa 0. Assim, das 27 combinacoes, (pelo menos) uma dah 0, e ha
no maximo 13 que determinam massas positivas, pois outras 13 determinam
menos os resultados daquelas primeiras 13 (nota: se os pesos forem mal
escolhidos, algumas dessas 13 podem dar 0 e voce perde opcoes!).

Problema 1: Creio que nao ha uma resposta para o primeiro problema! Se fosse
m variando de 1 a 13, eu acreditava que dava, mas 1 a 21, nao vejo como! O
raciocinio acima explica o porque de soh termos no maximo 13 valores
positivos possiveis de m.

Problema 2: O raciocinio acima mostra que m=13. Para m variando de 1 a 13,
podemos usar massas de 1, 3 e 9 gramas. Fica ai como exercicio ver que todo
numero de 1 a 13 pode ser escrito como combinacao de 1, 3 e 9, possivelmente
com sinais negativos. De qualquer forma, o proximo problema eh o caso geral
e inclui n=3.

Problema 3: A generalizacao do raciocinio acima mostra que, com n massas,
ha, no maximo, (3^n-1)/2 massas positivas que podem ser aferidas. Afirmo que
eh possivel aferir todos os numeros de 1 a (3^n-1)/2 usando os seguintes n
pesos: 1, 3, 9, 27, ..., 3^(n-1).

Para tanto, vou mostrar que combinacoes de coeficientes distintas tem que
dar numeros distintos, ou seja, a funcao f:{-1,0,1}^n em Z dada por
f(c1,c2,c3,...,cn)=c1.1+c2.3+c3.9+...+.cn.3^(n-1) eh injetiva. Afinal, se
dois estados da balanca dessem o mesmo peso, teriamos c1.1+c2.3+...+cn.3^n =
d1.1+d2.3+...+dn.3^n, onde cada ci e cada di sao -1, 0 ou 1. Seja m o maior
indice possivel onde cm  dm, ou seja, suponha c(m+1)=d(m+1) e
c(m+2)=d(m+2), etc. Cortando estes caras iguais dos dois lados, fico com:

c1.1+c2.3+...+cm.3^m = d1.1+d2.3+...+dm.3^m
(dm-cm).3^m = (c1-d1).1+(c2-d2).3+...+(cm-dm).3^(m-1)

Agora, o cara do lado esquerdo eh, em modulo, PELO MENOS 3^m, jah que
|cm-dm|=1. O cara do lado direito eh, em modulo, NO MAXIMO,
2.1+2.3+...+2.3^(m-1)=3^m-1. Entao eles nao podem ser iguais! ABSURDO! O
absurdo eh supor que cmdm para algum m; o unico jeito de combinacoes
destes numeros serem iguais eh se cada cm for igual a cada dm!

Entao as 3^n combinacoes distintas dos coeficientes c1, c2, ..., cn levam a
3^n resultados distintos da funcao f. Mas o resultado minimo eh botar
todos os coeficientes cn iguais a -1, que dah -1-3-9-...-3^n=-(3^n-1)/2; o
resultado maximo eh 1+3+9+...+3^n=(3^n-1)/2. Todos os resultados tem que ser
inteiros, e ha exatamente 3^n inteiros entre este minimo e aquele maximo
(incluindo 0, que corresponde a botar todo mundo igual a 0). Conclusao:
todos os inteiros tem de estar na imagem de f, isto eh, todas as massas m
entre -(3^n-1)/2 e (3^n-1)/2 podem ser medidas. Bom, as massas negativas nao
existem (se existissem, a gente as determinava!), mas todas as positivas
ainda podem ser determinadas.

Abraco,
 Ralph

2008/9/24 Tarso Moura Leitão [EMAIL PROTECTED]

  Caro professor Ralph, caso consiga algum tempinho gostaria muito de ver
 algum comentário seu sobre os problemas enviados de Haifa ( Israel ) por um
 professor
 bem velhinho.
 Antecipadamente agradeço.
 Tarso de Moura Leitão

  Todos os problemas se dão em uma balança de braços iguais.
 Primeiro - Num dos pratos da balança foi colocado um objeto cuja massa m é
 um número interiro de gramas, com m entre 1 e 21, inclusive.
 Existem três massas aferidas de x, y e z gramas com as quais ( e com a
 balança ) consegue-se determinar a massa m. Encontre x, y e z.

 Segundo: Considere, novamente, o objeto de massa m, agora m é um dos
 seguintes números inteiros:
 1,2,3,...,M - 1, M.. Qual o valor máximo de M para que ainda seja
 possível determinar o valor de m usando três massas aferidas de x, y e z
 gramas ? Determine também x,y e z e exiba um procedimento para obter o valor
 máximo M.

 Terceiro - É dado que m é um dos números inteiros
 1,2,3,,M - 1, M.  Qual o valor máximo de M para que seja possível
 calcular m usando n massas aferidas ?
 Nos itens anteriores tínhamos apenas três massas aferidas.
 ( Nos enunciado recebido do professor Bloh não há condições impostas sobre
 o número n, parece razoável buscar o menor n.)



Re: [obm-l] Ajuda Equação Diofantina de Segundo Grau

2008-09-23 Por tôpico Ralph Teixeira
Abrindo e simplificando:

2^n(2M+1)=2N-2L

Entao n=1 para que ambos os lados sejam pares. Entao posso escrever:

2^(n-1) (2m+1) = N-L

Daqui voce ve que tem um MONTE de solucoes -- escolha N e L inteiros
quaisquer distintos; o numero N-L sempre pode ser escrito de maneira unica
como potencia de 2 vezes um impar; a potencia eh n-1, o impar eh 2m+1.

Abraco,
   Ralph

P.S.: Para ver o negocio de que todo inteiro A pode ser escrito de maneira
unica como potencia de 2 vezes impar, eh soh fatora-lo em fatores primos:
A=+-2^(p1).3^(p3).5^(p5)... A unica maneira de decompor isto em potencia de
2 vezes impar eh tomar a potencia como 2^(p1) e o impar como
+-3^(p3).5^(p5)



On Tue, Sep 23, 2008 at 3:58 PM, luiz silva [EMAIL PROTECTED]wrote:

   (L+1) 2 + 2n (M + 1)2 + N2 = L2 + 2nM2 + (N+1)2

 Alguém sabe resolver esta equação diofantina ?

 Abs
 Felipe


 --
 Novos endereços, o Yahoo! que você conhece. Crie um email 
 novohttp://br.rd.yahoo.com/mail/taglines/mail/*http://br.new.mail.yahoo.com/addressescom
  a sua cara @
 ymail.com ou @rocketmail.com.



Re: [obm-l] A Lei de Benford e as Loterias

2008-09-22 Por tôpico Ralph Teixeira
Deixa eu ver se desmistifico um pouco a (belíssima!) distribuição de
Benford...

Note que há vários tipos de números randômicos. Se um numero eh sorteado
aleatoriamente em [10,100) (com distribuição uniforme de probabilidade),
então o primeiro digito tem a mesma chance de ser 1, 2 ou 7, com 1/9 para
cada.

Agora, a distribuição de probabilidade dos números que aparecem em vários
eventos não eh aleatória DESTE jeito. Se os números tem uma densidade de
probabilidade do tipo C/x (onde x varia continuamente de 1 a 100, digamos),
então:

a) A Integral de C/x de x=1 a x=100 tem de dar 1; com isto, você descobre
que C=1/(ln(100)-ln(1))=1/ln(100)
b) A probabilidade do primeiro digito ser 1 eh a integral C/x de 1 a 2, mais
a integral de 10 a 20, que dah
(ln(20)-ln(10))/ln(100)+(ln(2)-ln(1))/ln(100)=log(2) (log aqui eh base 10);
analogamente, a probabilidade do primeiro digito ser 6 sera
(ln(7)-ln(6)+ln(70)-ln(60))/ln(100)=log(7)-log(6). E assim por diante, os
dígitos menores são menos comuns, e a distribuição tem as diferenças entre
logs sucessivos.

A mesma distribuição aparece se você supuser que x varia continuamente de
10^m a 10^n com densidade de probabilidade do tipo C/x. Ta, o assunto não eh
só isso (por exemplo, minha escolha de intervalo de 10^m a 10^n eh meio
especifica; mas, se x variar de a10^m10^nb, com varias ordens de grandeza
entre a e b, então os rabos de probabilidade entre a e 10^m e entre 10^n e
b não devem afetar muito as coisas; se o valor mínimo for a=1, as coisas
melhoram muito!), mas acho que o espírito esta contido nisso que eu escrevi.
Em suma, se os números que você esta olhando tem uma distribuicao que faz
números altos serem raros (mais ou menos com densidade proporcional a
1/x), e tem pelo menos algumas ordens de grandeza possíveis nos dados, então
os primeiros dígitos satisfazem a distribuicao de Benford. Eh por isso que
histogramas com comprimentos de rios e populacoes de cidades tem primeiro
digito satisfazendo esta distribuição, mas primeiro digito de números de
loteria não (em loterias, não há preferência por números menores, todos são
igualmente prováveis).

A propósito, há algum tempo atrás, eu fui ao site do IBGE e catei as
populacoes de todos os municípios brasileiros pelo ultimo censo, botei tudo
no Excel para extrair o primeiro digito de cada e botei num histograma. Eh
inacreditável como o histograma encaixa quase perfeitamente numa
distribuição de Benford (assim como as primeiras tantas potencias de vários
números -- 10 não vale -- e outras coisas que eu experimentei onde números
altos são mais raros). Agora, se você declarar que uma cidade eh um
município com 7 a 12 habitantes, a distribuição do primeiro digito
das populacoes das cidades não eh mais Benford -- como você se limitou a
menos de uma ordem de grandeza nos dados, você eliminou completamente a
possibilidade de o primeiro digito ser 2, 3, 4, 5 ou 6, e isso muda toda a
distribuição, eh claro.

Isto dito, ainda não li os papers que justificam a distribuição com mais
rigor -- tenho certeza que meus argumentos acima devem estar cheios de
furos...

Abraço,
  Ralph

2008/9/22 Bouskela [EMAIL PROTECTED]

  Olá José Airton!

 Vi em, uma de suas mensagens posteriores, que você conseguiu concluir que a
 Lei de Benford é válida para a distribuição do 1º dígito de 2^n. Muito bem!

 Posso fazer também um raciocínio lógico para INTUIR a validade desta Lei
 para as cotações de uma Bolsa de Valores genérica: a unidade monetária
 (real, dólar, euro...) é (geralmente) projetada para que os preços se situem
 nas proximidades da própria unidade, ou, então, de seus múltiplos de 10 -
 daí a primazia do número 1 - vá lá...

 Mas o que dizer da distribuição do número de habitantes das cidades de um
 país? E do número de mangas em cada mangueira de um grande pomar? Etc...

 Podemos INTUIR que a nossa base decimal de numeração está em harmonia com o
 Universo (nossa lógica faz parte dele) e, portanto, foi projetada para
 privilegiar o número 1 - um argumento meio na linha do Paulo Coelho... (é
 dose!).

 Bem, ao que tudo indica, a Lei de Benford se aplica a (todas?) as
 distribuições quase-aleatórias. Por que quase? Porque essas tais
 distribuições devem estar associadas a eventos reais (físicos) - pelo
 menos é assim que parece! Uma distribuição de números randômicos, por
 exemplo, não obedece à Lei de Benford.

 Em resumo, concordo com o Tarso: é tudo muito estranho...

 Sds.,
   [EMAIL PROTECTED]
 [EMAIL PROTECTED]

  --
 *De:* [EMAIL PROTECTED] [mailto:[EMAIL PROTECTED] *Em
 nome de *JOSE AIRTON CARNEIRO
 *Enviada em:* domingo, 21 de setembro de 2008 22:14
 *Para:* obm-l@mat.puc-rio.br
 *Assunto:* Re: [obm-l] A Lei de Benford e as Loterias

  Estranha mas verdadeira.Acabei de comprovar com a calculadora do meu
 celular:
 2^0 = 1
 2^4 = 16
 2^7 = 128
 2^10 = 1024
 2^14 = 16384
 2^17 = 131072
 2^20 = 1048576
 2^24 = 16777216
 2^27 = 134217728
 2^30 = 1073741824
 2^34 = 17179869184
 2^37 = 

Re: [obm-l] BOTES

2008-09-06 Por tôpico Ralph Teixeira
A chave eh sacar que voce quer levar os botes Oito e Quatro juntos, porque
sao os mais lentos, e voce jamais quer voltar com qualquer um deles, para
nao perder tempo... Entao, quando voce for levar Quatro e Oito, tem que ter
algum bote rapido do outro lado esperando para voce voltar com ele... Como
conseguir isto?

Abraco,
 Ralph

On Fri, Sep 5, 2008 at 10:19 PM, arkon [EMAIL PROTECTED] wrote:

  Pessoal, qual o macete para essa questão?


 Existem quatro botes numa margem de um rio; seus nomes são Oito, Quatro,
 Dois e Um, porque essas são as quantidades de horas que cada um deles demora
 para cruzar o rio. Pode-se atar um bote a outro, porém não mais de um, e
 então o tempo que demoram em cruzar é igual ao do mais lento dos botes. Um
 só marinheiro deve levar todos os botes até à outra margem do rio. Qual é o
 menor tempo necessário para completar o translado?

 Gabarito: 15 h.
 =
 Instru�ões para entrar na lista, sair da lista e usar a lista em
 http://www.mat.puc-rio.br/~obmlistas/obm-l.html=


Re: [obm-l] Coisas de alunos

2008-08-20 Por tôpico Ralph Teixeira
Se a questao era:

Resolva a equacao 

eu diria que a solucao estah incorreta. Digo isso pois, para mim resolva
significa encontre todas as solucoes e deixe claro que voce achou todas.
Agora, se a questao fosse:

Encontre uma solucao da equacao...

entao eu diria que o raciocinio estah 100% correto.

Isso dito, concordo com os colegas que experimentacao deve ser encorajada, e
eu daria uns pontos parciais pelo que foi feito. Para eles entenderem o que
estah errado, eu colocaria o seguinte raciocinio para eles:

Resolva a equacao x^3-x=0

e eu faria

x^3-x=0^3-0
x=0

o que dah uma solucao, mas nao sao todas. Eu tentaria dizer que a primeira
linha NAO IMPLICA NECESSARIAMENTE na segunda. Que, se x=0, entao x^3-x=0,
mas nao vice-versa.

Abraco,
  Ralph

2008/8/20 Walter Tadeu Nogueira da Silveira [EMAIL PROTECTED]

 Amigos,

 Nossos alunos fazem coisas que imprevisíveis. Uma ajuda nessa correção.

 A questão era de exponencial: 3^(x+2)-3^(x)=24

 Muitos alunos descobriram que 24 = 27 - 3 ou 3^3 - 3^1

 E montaram a equação: x + 2 = 3 então x = 1.

 Como discutir essa correção com eles? Alguma sugestão?

 Grato





Re: [obm-l] Problemas interessantes de coloração

2008-07-25 Por tôpico Ralph Teixeira
Mas nao precisa ser o triangulo todo da mesma cor -- bastam os VERTICES
:)

2008/7/25 Igor Battazza [EMAIL PROTECTED]:

 Tambem nao sei se entendi, pois o problema nao diz nada sobre
 restriçoes a respeito das cores... Se nao tiver restriçoes, na 4),
 acho que posso colorir o plano em listras alternadas com 2 cores, azul
 e vermelho por exemplo, de maneira que a espessura de cada listra seja
 menor do que 1 unidade (1/2 unidade por ex.).

 2008/7/25 Rafael Ando [EMAIL PROTECTED]:
   Nao sei se entendi direito o 3 e o 5, mas o que me impede de fazer o
  seguinte:
 
  Sejam azul e vermelho as duas cores. Seja A um ponto azul. Entao seja d0
 a
  distancia minima de A ate qualquer ponto vermelho. Entao todos o pontos
 da
  circunferencia de centro A e raio rd serao azuis tambem. Um retangulo /
  triangulo equilatero inscrito nessa circunferencia resolveriam o
  problema
 
  2008/7/24 Otávio Menezes [EMAIL PROTECTED]:
 
  1) Pinte o plano com três cores. Prove que há dois pontos com a mesma
 cor
  situados a exatamente 1 unidade um do outro.
  2) Pinte o plano com duas cores. Prove que uma dessas cores contém pares
  de pontos a qualquer distância entre si.
  3) Pinte o plano com duas cores. Prove que existe um triângulo
 equilátero
  com todos os vértices da mesma cor.
  4) Mostre que é possível colorir o plano em duas cores de modo que não
  exista um triâmgulo equilátero de lado 1 com todos os vértices da mesma
 cor.
  5) Pinte o plano em duas cores. Mostre que existe um retângulo com todos
  os vértice da mesma cor.
 
  Os dois primeiros são muito fáceis, os outros são mais complicados.
 
 
 
  --
  Rafael

 =
 Instruções para entrar na lista, sair da lista e usar a lista em
 http://www.mat.puc-rio.br/~obmlistas/obm-l.html
 =



Re: [obm-l] Problemas interessantes de coloração

2008-07-25 Por tôpico Ralph Teixeira
Oi, Rafael -- mas esta distancia minima pode nao existir... Por exemplo, no
plano xy, imagine que pintamos de azul todos os pontos de coordenadas (x,y)
onde ambos x e y sao racionais; todos os outros pontos, onde x ou y sao
irracionais, a gente pinta de vermelho. Entao, escolhido um ponto A azul,
nao existe a distancia minima d0 que voce citou...

Abraco,
Ralph

2008/7/25 Rafael Ando [EMAIL PROTECTED]:

 Nao sei se entendi direito o 3 e o 5, mas o que me impede de fazer o
 seguinte:

 Sejam azul e vermelho as duas cores. Seja A um ponto azul. Entao seja d0 a
 distancia minima de A ate qualquer ponto vermelho. Entao todos o pontos da
 circunferencia de centro A e raio rd serao azuis tambem. Um retangulo /
 triangulo equilatero inscrito nessa circunferencia resolveriam o
 problema

 2008/7/24 Otávio Menezes [EMAIL PROTECTED]:

 1) Pinte o plano com três cores. Prove que há dois pontos com a mesma cor
 situados a exatamente 1 unidade um do outro.
 2) Pinte o plano com duas cores. Prove que uma dessas cores contém pares
 de pontos a qualquer distância entre si.
 3) Pinte o plano com duas cores. Prove que existe um triângulo equilátero
 com todos os vértices da mesma cor.
 4) Mostre que é possível colorir o plano em duas cores de modo que não
 exista um triâmgulo equilátero de lado 1 com todos os vértices da mesma cor.
 5) Pinte o plano em duas cores. Mostre que existe um retângulo com todos
 os vértice da mesma cor.

 Os dois primeiros são muito fáceis, os outros são mais complicados.




 --
 Rafael



Re: [obm-l] Moedas: 2 problemas

2008-07-23 Por tôpico Ralph Teixeira
Rotule as moedas com os numeros de 1 a 15, mas escreva-os em binario com 4
algarismos cada: 0001, 0010, ..., .

Separe as moedas em 4 grupos -- o grupo que tem 1 no primeiro digito, o que
tem 1 no segundo digito, etc. Explictamente, em decimal, os grupos sao:

G1={8,9,10,11,12,13,14,15}
G2={4,5,6,7,12,13,14,15}
G3={2,3,6,7,11,12,14,15}
G4={1,3,5,7,9,11,13,15}

Agora verifique que grupos tem um peso maior que os outros, pois estes
contem a moeda falsa. A sua moeda falsa eh a unica que estah exatamente nos
grupos escolhidos.
Alias, monte um numero d1d2d3d4 fazendo di=1 se o grupo i eh mais pesado que
os outros, e di=0 caso o grupo i tenha peso normal. A moeda falsa eh a
representada por d1d2d3d4 (em binario).

Ah, sim, note que, como HA uma moeda falsa, nao pode ser ; em outras
palavras, se voce der azar e todos os grupos tiverem o mesmo peso, voce
conclui que TODOS tem a moeda falsa, que eh a moeda =15.

Note como deste jeito eh facil generalizar para 2^n-1 moedas e n pesagens!

Abraco,
  Ralph
2008/7/23 Bouskela [EMAIL PROTECTED]:

  Olá!



 1º PROBLEMA:



 Acredito que quase todos vocês já conheçam o problema 12 (ou 13) moedas /
 1 moeda falsa (+ leve OU + pesada) / balança de comparação. Seu enunciado é
 o seguinte:



 Considere uma coleção de 12 (pode, até, ser de 13) moedas – uma delas é
 falsa. A única diferença entre a moeda falsa, em relação às demais, é que o
 seu peso é DIFERENTE, isto é, pode ser MAIOR, ou MENOR, do que o peso das
 moedas verdadeiras.

 Todas as moedas verdadeiras têm o mesmo peso. Todas as moedas – inclusive a
 falsa – são aparentemente iguais.

 Dispondo de uma balança de comparação (balança de dois pratos), pede-se
 determinar qual é a moeda falsa, utilizando a balança, no máximo, 3 vezes.



 Reparem que se dispõe apenas de uma balança de comparação (i.e., uma
 balança de dois pratos).



 Este é um problema simples, cuja solução, entretanto, requer bastante
 inteligência. Aqueles que não o conhecem podem tentar resolvê-lo.



 2º PROBLEMA:



 Outro dia, um aluno me propôs um problema similar e, também, bastante
 interessante: 15 moedas / 1 moeda falsa (+ pesada) / 1 balança eletrônica.
 Segue, abaixo, seu enunciado:



 Considere uma coleção de 15 moedas – uma delas é falsa. A única diferença
 entre a moeda falsa e as demais é que ela é mais pesada.

 As 14 moedas verdadeiras têm o mesmo peso. As 15 moedas são aparentemente
 iguais.

 Dispondo de uma balança eletrônica (destas que fornecem o peso com
 exatidão), pede-se determinar qual é a moeda falsa, utilizando a balança, no
 máximo, 4 vezes.



 Reparem que se dispõe apenas de uma balança eletrônica, isto é, de um único
 prato. Este tipo de balança indica o valor numérico do peso de uma
 determinada massa (no caso n moedas), colocada sobre o seu prato.



 Nota: a divisão (quebra) de qualquer moeda não é permitida.



 Bem, só consegui resolver este problema de uma forma MUITO complicada.
 Resolvi, então, propô-lo a vocês para saber se alguém conhece, ou descobre,
 uma solução mais simples.



 Saudações,

 AB.



Re: [obm-l] Moedas: 2 problemas

2008-07-23 Por tôpico Ralph Teixeira
Ah, droga, errei... troquem por favor o 12 do grupo 3 pelo 10. :)

2008/7/24 Ralph Teixeira [EMAIL PROTECTED]:

  Rotule as moedas com os numeros de 1 a 15, mas escreva-os em binario com
 4 algarismos cada: 0001, 0010, ..., .

 Separe as moedas em 4 grupos -- o grupo que tem 1 no primeiro digito, o que
 tem 1 no segundo digito, etc. Explictamente, em decimal, os grupos sao:

 G1={8,9,10,11,12,13,14,15}
 G2={4,5,6,7,12,13,14,15}
 G3={2,3,6,7,11,12,14,15}
 G4={1,3,5,7,9,11,13,15}

 Agora verifique que grupos tem um peso maior que os outros, pois estes
 contem a moeda falsa. A sua moeda falsa eh a unica que estah exatamente nos
 grupos escolhidos.
 Alias, monte um numero d1d2d3d4 fazendo di=1 se o grupo i eh mais pesado
 que os outros, e di=0 caso o grupo i tenha peso normal. A moeda falsa eh a
 representada por d1d2d3d4 (em binario).

 Ah, sim, note que, como HA uma moeda falsa, nao pode ser ; em outras
 palavras, se voce der azar e todos os grupos tiverem o mesmo peso, voce
 conclui que TODOS tem a moeda falsa, que eh a moeda =15.

 Note como deste jeito eh facil generalizar para 2^n-1 moedas e n pesagens!

 Abraco,
   Ralph
 2008/7/23 Bouskela [EMAIL PROTECTED]:

   Olá!



 1º PROBLEMA:



 Acredito que quase todos vocês já conheçam o problema 12 (ou 13) moedas /
 1 moeda falsa (+ leve OU + pesada) / balança de comparação. Seu enunciado é
 o seguinte:



 Considere uma coleção de 12 (pode, até, ser de 13) moedas – uma delas é
 falsa. A única diferença entre a moeda falsa, em relação às demais, é que o
 seu peso é DIFERENTE, isto é, pode ser MAIOR, ou MENOR, do que o peso das
 moedas verdadeiras.

 Todas as moedas verdadeiras têm o mesmo peso. Todas as moedas – inclusive
 a falsa – são aparentemente iguais.

 Dispondo de uma balança de comparação (balança de dois pratos), pede-se
 determinar qual é a moeda falsa, utilizando a balança, no máximo, 3 vezes.



 Reparem que se dispõe apenas de uma balança de comparação (i.e., uma
 balança de dois pratos).



 Este é um problema simples, cuja solução, entretanto, requer bastante
 inteligência. Aqueles que não o conhecem podem tentar resolvê-lo.



 2º PROBLEMA:



 Outro dia, um aluno me propôs um problema similar e, também, bastante
 interessante: 15 moedas / 1 moeda falsa (+ pesada) / 1 balança eletrônica.
 Segue, abaixo, seu enunciado:



 Considere uma coleção de 15 moedas – uma delas é falsa. A única diferença
 entre a moeda falsa e as demais é que ela é mais pesada.

 As 14 moedas verdadeiras têm o mesmo peso. As 15 moedas são aparentemente
 iguais.

 Dispondo de uma balança eletrônica (destas que fornecem o peso com
 exatidão), pede-se determinar qual é a moeda falsa, utilizando a balança, no
 máximo, 4 vezes.



 Reparem que se dispõe apenas de uma balança eletrônica, isto é, de um
 único prato. Este tipo de balança indica o valor numérico do peso de uma
 determinada massa (no caso n moedas), colocada sobre o seu prato.



 Nota: a divisão (quebra) de qualquer moeda não é permitida.



 Bem, só consegui resolver este problema de uma forma MUITO complicada.
 Resolvi, então, propô-lo a vocês para saber se alguém conhece, ou descobre,
 uma solução mais simples.



 Saudações,

 AB.





Re: [obm-l]

2008-07-22 Por tôpico Ralph Teixeira
Huh, assim, tá errado. Por exemplo, toma A=0, uma matriz nula nxn, e
qualquer matriz B nxn satisfaz esta equação. Deve estar faltando alguma
hipótese no enunciado?

Ah, pera aí... talvez seja mostrar que só há uma matriz B tal que AB=A para
TODA matriz A nxn? Se for isso, então tome A=I (identidade nxn), e vem IB=I,
isto é, B=I é a única que serve.

Abraço,
Ralph

2008/7/22 Samuel Wainer [EMAIL PROTECTED]:

 Olá,

 Estu lendo o livro Física matemática do Butkov e achei um problema
 interessante:

 Seja A uma matriz, prove q a matriz B tal que: AB=A é única.

 Essa unicidade da identidade está quebrando a minha cabeça. Alguém já viu
 algo parecido?

 Desde á agradeço,

 Samuel

 --
 Notícias direto do New York Times, gols do Lance, videocassetadas e muitos
 outros vídeos no MSN Videos! Confira já! http://video.msn.com/?mkt=pt-br



Re: [obm-l] Re: [obm-l] Re: [obm-l] Re: [obm-l] Probabilidades Geométricas: 2 problemas difíceis

2008-07-10 Por tôpico Ralph Teixeira
Este problema eh legal, e jah apareceu um par de vezes na lista. A minha
solucao eh igualzinha aa do Ponce, mas a **MII-NHA** tem uma figuri-inha, a
do Pon-ce **NAO TE-EM!!**. :P
Aqui estah ela, para que todos apreciem meus dotes artisticos:
http://www.mat.puc-rio.br/~nicolau/olimp/obm-l.200706/msg00182.html

 Abraco, Ralph.

P.S.: Eh, por causa destes dotes artisticos eh que eu fui fazer
Matematica :)

2008/7/10 Chicao Valadares [EMAIL PROTECTED]:
 Eu fiz algo parecido e achei 1/12. Depois eu posto aqui na lista.


 O Binômio de Newton é tão belo como a Vênus de Milo.
 O que há é pouca gente para dar por isso... 
 Fernando Pessoa - Poesias de Alvaro Campos

 _
 As informações existentes nessa mensagem e no(s) arquivo(s) anexado(s)
 são
 para uso restrito, sendo seu sigilo protegido por lei. Caso não seja
 destinatário, saiba que leitura, divulgação ou cópia são proibidas.
 Favor
 apagar as informações e notificar o remetente. O uso impróprio será
 tratado
 conforme as normas da empresa e a legislação em vigor. Agradecemos sua
 colaboração.


 The information mentioned in this message and in the archives attached
 are
 of restricted use, and its privacy is protected by law. If you are not
 the
 addressee, be aware that reading, disclosure or copy are forbidden.
 Please
 delete this information and notify the sender. Inappropriate use will
 be
 tracted according to company's rules and valid laws. Thank you for your
 cooperation.


 --- Em seg, 7/7/08, Rogerio Ponce [EMAIL PROTECTED] escreveu:

 De: Rogerio Ponce [EMAIL PROTECTED]
 Assunto: Re: [obm-l] Re: [obm-l] Re: [obm-l] Probabilidades Geométricas:
2 problemas difíceis
 Para: obm-l@mat.puc-rio.br
 Data: Segunda-feira, 7 de Julho de 2008, 20:38
 Ola' Chicao,
 sem perda de generalidade, eu assumi que o segmento
 de reta do
 problema seria o segmento unitario [0 1], de forma que
 x pode ser
 qualquer real no intervalo [0, 1].
 E para cada valor de x, o ponto y
 tambem pode estar em qualquer
 posicao no intervalo [0, 1].
 Assim, usando o espaco cartesiano para plotar todos os
 pares (x,y)
 possiveis, voce obtera' um quadrado de lado unitario.
 Da mesma forma, se voce plotar todos os pares que
 satisfazem 'as
 exigencias do problema, voce obtera'  os dois
 triangulos internos ao
 quadrado unitario, conforme descrito na solucao.

 Repare que os tais dois triangulos sao
 simplesmente o conjunto de
 pares (x,y) capazes de definir um triangulo sobre o
 segmento unitario,
 conforme o enunciado.
 Para isso, e' necessario e suficiente que x
 e y satisfacam 'as
 seguintes condicoes:
 - o menor deles e' menor (ou igual**) que 1/2
 - o maior deles e' maior (ou igual**) que 1/2
 - a diferenca entre eles e' menor (ou igual**) que 1/2

 ** OBS: quando acontece um igual , temos um
 triangulo degenerado
 (com area zero).

 []'s
 Rogerio Ponce.



 2008/7/7 Chicao Valadares
 [EMAIL PROTECTED]:
  Os valores possiveis de x e y equivalem a area
 do quadrado unitario,
   que vale 1.
 
  Nao entendi, seria o produto xy que equivaleria a
 área?
 
 

 =
 Instruções para entrar na lista, sair da lista e usar a
 lista em
 http://www.mat.puc-rio.br/~obmlistas/obm-l.html
 =


  Novos endereços, o Yahoo! que você conhece. Crie um email novo com a
sua cara @ymail.com ou @rocketmail.com.
 http://br.new.mail.yahoo.com/addresses

 =
 Instruções para entrar na lista, sair da lista e usar a lista em
 http://www.mat.puc-rio.br/~obmlistas/obm-l.html
 =



Re: [obm-l] Probabilidade

2008-07-09 Por tôpico Ralph Teixeira
Oi, Anna, e galera.

Vou ser pedante e prolixo, ateh mais do que costumo ser, entao jah peco
desculpas adiantado...

Um problema bem enunciado de probabilidade tem que dizer (i) o que e como
algo serah escolhido, (ii) se ha alguma informacao adicional do resultado
desta escolha, (iii) de que evento deseja-se calcular a probabilidade. A
informacao (ii) ateh pode ser omitida, mas, ainda assim, o minimo devia ser:

i) A gente escolhe blah blah blah assim assim assim PONTO. iii) Qual a
probabilidade de ESTE EVENTO BACANA ACONTECER?

Se voces prestarem bastante atencao a esta estrutura, vao perceber o defeito
do enunciado do jeito que estah: como eles botaram toda a pergunta depois de
qual a probabilidade de..., estah tudo no pedaco (iii), sem dar a minima
indicacao de se e como (i) vai acontecer (e o sendo, gerundio meio solto,
tambem nao ajuda -- ele se refere a escolher-se ou eh um complemento de
duas pessoas de sexos diferentes? Eh algo que se sabe, ou que se quer?).

Entao minha resposta pedante e totalmente tecnica eh:
Eh impossivel determinar esta probabilidade, porque eu nao sei como nem o
que voce vai escolher, o que voce tah afim de escolher Nao eh **soh** um
problema de equiprobabilidade, que naturalmente a gente pressupoe nesses
problemas, eh pior -- por exemplo, se voce for escolher 40 pessoas deste
grupo, a probabilidade de escolher-se duas pessoas de sexos diferentes,
sendo apenas uma delas de oculos, eh altissima; se voce for escolher soh
uma pessoa do grupo, a mesma probabilidade eh 0; se voce escolher o Ralph,
que nao usa mais oculos, e a Anna Luisa, que usa, porque eles sao muito
legais e voce vai pagar cinema para eles, a probabilidade eh 1. Alias, se
voce resolver nao escolher ninguem, a probabilidade eh 0.

Bom, por outro lado, sendo um pouco menos chato, a *impressao* que fica do
enunciado eh que serao escolhidas 2 pessoas do grupo, aleatoriamente. Ainda
assim, como (i) nao estah claro, vejo 3 interpretacoes possiveis; vou
escreve-las aqui do jeito que eu as enunciaria (vou colocar onde comeca cada
pedaco (i), (ii) e (iii) daquela estruturinha):

 A) (i) Escolha aleatoriamente duas pessoas (distintas) deste grupo. (iii)
Qual a probabilidade de elas terem sexos diferentes **e** apenas uma delas
usar oculos? (Acho que eh isso que eles queriam...)
B) (i) Escolha aleatoriamente duas pessoas (distintas) deste grupo. (ii)
Sendo que apenas uma usa oculos (eu de fato escreveria sabendo para nao
deixar duvida), (iii) qual a probabilidade de elas serem de sexos
diferentes? (Acho que eles nao queriam isso, mas, literalmente, esta
interpretacao faz um melhor uso da palavra sendo!)
C) (i) Escolha aleatoriamente duas pessoas deste grupo (ii) de sexos
diferentes. (iii) Qual a probabilidade de apenas uma delas usar oculos?
(Acho esta interpretacao meio forcada, mas ateh valida.)

Vamos responder logo as 3 questoes (como eu disse, se isso caisse numa
prova, eu responderia (A), mas acho que (B) eh tecnicamente melhor)

PARTE COMUM a A, B e C:
Farei M=mulher, H=homem, O=oculos, S=sem oculos. Entao ha:
16 MO (mulheres, mocas com oculos; afinal, todas as mulheres sao
sempre mocas... :) ;) )
20 HO (homens com oculos)
30 HS (homens sem oculos)
24 MS (mulheres sem oculos)

Interpretacao A) Seriam:
C(90,2)=90x89/2 maneiras **equiprovaveis** de escolher um par de pessoas;
MOxHS+MSxHO=16x30+24x20 maneiras favoraveis;
A probabilidade seria entao (16x30+24x20)x2/(90x89)=64/267

Interpretacao B) Seriam:
OxS=36x54 maneiras **equiprovaveis** de escolher um par de pessoas,
sabendo-se com certeza que uma usa oculos e a outra nao;
16x30+24x20 maneiras favoraveis;
A probabilidade seria entao (16x30+24x20)/(36x54)=40/81

Interpretacao C) Seriam:
MxH=40x50 maneiras **equiprovaveis** de escolher duas pessoas de sexos
diferentes;
16x30+24x20 maneiras favoraveis;
A probabilidade seria entao (16x30+24x20)/(40x50)=12/25

Argh, aposto que atrapalhei mais do que ajudei. Notem que essa discussao eh
chata mais importante -- muita gente se aproveita destas ambiguidades para
enganar as pessoas com estatisticas...

Serah que alguem leu esta coisa toda? :)

Abraco,
Ralph

P.S.: Note como sendo eh ambiguo. Imagine as duas seguintes situacoes:
A) Eu chego na sorveteria e digo: Eu quero um sorvete de duas bolas, sendo
uma bola de chocolate. Voce ouve: eu quero 2 bolas *e* eu quero
chocolate. O sendo aqui eh algo que se deseja, eh praticamente um
complemento de sorvete. Esta interpretacao eh analoga a (A) acima.
B) Eu chego na sorveteria e eles me dao um sorvete de uma bola, de
chocolate. Entao eu digo: Sendo uma bola de chocolate, eu quero um sorvete
de duas bolas... Voce ouve: **Como** uma bola eh de chocolate, eu quero
outra. Agora o sendo indica uma informacao que se sabe e ao mesmo
tempo uma hipotese, o Se de um Se...entao. Isto dah interpretacao (B).

2008/7/9 Anna Luisa [EMAIL PROTECTED]:

  Por favor alguém pode me dar uma ajuda.

 1) Em uma reunião há 90 pessoas, 36 das quais usam óculos. Se 16 moças usam
 óculos e 30 

[obm-l] Constante de Artur

2008-06-18 Por tôpico Ralph Teixeira
Caramba! Este numero eh EXATAMENTE o numero que eu precisava para demonstrar
a conjectura de Riemann! Achei uma raiz da funcao zeta cuja distancia aa
reta Re(z)=1/2 eh 1/(Constante de Artur)!

A demonstracao eh notavel, se resume a apenas esta figurinha
que, infelizmente nao cabe na margem destes 20Kb.

Abraco,
   Ralph

P.S.: ;)
2008/6/18 Bruno França dos Reis [EMAIL PROTECTED]:

 Segundo o Google:
 Constante de Artur = ln(1 + arctan(e^2 - 3,79)^pi)) + cosh(pi^e+
 e^(1,21*pi)) = 7.80040173 * 10^28

 Não resisti...
 Bruno


 2008/6/18 Artur Costa Steiner [EMAIL PROTECTED]:

 Acho que nao eh um postulado, mas sim uma definicao. Da mesma forma que,
 por definicao, a^n = a**a (n vezes) para n inteiro positivo. Da mesma
 forma que, por definicao, Gama(x) = Integral (0 a oo) e^(-t) t^(x -1) dx

 Se eu fosse um cara prepotente, poderia definir número de Artur como ln(1
 + arctan(e^2 - 3,79)^pi)) + cosh(pi^e^+ e^(1,21pi. Contrariamente a outras
 cosntantes, nao serve para nada, uma definicao idiota, as seria uma
 definicao, nao um postulado.

 Artur

 -Mensagem original-
 De: [EMAIL PROTECTED] [mailto:[EMAIL PROTECTED]
 nome de Paulo Santa Rita
 Enviada em: quarta-feira, 18 de junho de 2008 13:59
 Para: obm-l@mat.puc-rio.br
 Assunto: Re: [obm-l] FATORIAL DE ZERO


  Ola Jorge e demais colegas
 desta lista ... OBM-L,

 Nao ha o que justificar ...  0! = 1 e um POSTULADO : tao POSTULADO
 quanto o quinto postulado de Euclides. E - assim como o famoso
 postulado euclidiano tambem foi - ele e ainda hoje um dos alicerces da
 nossa maneira de contar, pois, se o negarmos, as consequencias que dai
 advem parecem nao corresponder com a realidade com que estamos
 acostumados a lidar

 Mas nada pode tolher a nossa liberdade de imaginacao.

 Quando o Lobachevski negou o quinto postulado de Euclides e afirmou
 que por um ponto fora de uma reta era possível traçar não uma, mas
 várias retas paralelas a reta inicial dada, ele chamou os
 desenvolvimento desta LOUCA HIPOTESE de GEOMETRIA IMAGINARIA
 simplesmente porque achava que a realidade se conformava com a
 geometria de Euclides, nao com a Geometria que ela estava descobrindo.
 Entretanto, com o passar do tempo, ficamos sabendo que a realidade e
 muito provavelmente NAO-EUCLIDIANA mais provavel que a realidade se

  Jorge Paulino wrote:
   Provavelmente esse tópico já foi criado em algum
  momento. Mesmo assim, como sou novo por aqui, gostaria de alguma
  contribuição.
 
  Sem recorrer à função gama, usando como recurso
  apenas a interpretação através da problemas de contagem, como justificar
 que
  0!=1??
 
  Eu conheço apenas a interpretação vinculada ao
  número de subconjuntos. Como Cn,p é igual ao número de subconjuntos de p
  elementos de um conjunto de n elementos,  então Cn,0 = 1 indica o número
 de subconjuntos de 0 elementos, a saber, o
  vazio.
 
  Porém, se C8,3 indica o número de comissões
  de 3 pessoas num grupo de 8, como aceitar que o número de comissões de
 zero
  pessoas é igual C8,0=1?
 
  Se A5,3 fornece o número de senhas de 3 letras
  distintas a partir de um universo de 5, como aceitar que deste mesmo
 universo é
  possível obter uma senha de zero letras, isto é, A5,0 = 1?
 
   Grato,
   Jorge

 =
 Instruções para entrar na lista, sair da lista e usar a lista em
 http://www.mat.puc-rio.br/~obmlistas/obm-l.html
 =

 =
 Instruções para entrar na lista, sair da lista e usar a lista em
 http://www.mat.puc-rio.br/~obmlistas/obm-l.html
 =




 --
 Bruno FRANÇA DOS REIS

 msn: [EMAIL PROTECTED]
 skype: brunoreis666
 tel: +33 (0)6 28 43 42 16

 e^(pi*i)+1=0


Re: [obm-l] DESAFIO

2008-05-19 Por tôpico Ralph Teixeira
Desculpa, Eduardo, mas eu vou ser muito muito chato e inserir minha fala
probabilística favorita (quem me conhece não me aguenta mais com isso):

Mas os eventos contados são igualmente prováveis?

(Neste caso, não são!!, então sua solução, apesar de muito bela,
infelizmente não funciona.)

---///---

Vamos tentar outra solução... Comprei n caixas. Vou supor que
i) As probabilidades dos brinquedos estão igualmente distribuídos (isto é,
não há, a priori, figurinha difícil); isto significa que a probabilidade
de uma determinada caixa conter o brinquedo 1 é 1/5=0.2, assim como o
brinquedo 2, 3, 4 ou 5.
ii) Caixas distintas são independentes entre si; esta é uma suposição
razoável se, por exemplo, as caixas são bem distribuídas geograficamente, ou
se você compra de vários lugares aleatoriamente, e se o número de caixas que
você compra é bem menor que o produzido... Tem outros jeitos de esta
suposição ser razoável também, então fico com ela.

Então vamos lá: sejam N1, N2, N3, N4 e N5 as probabilidades de você NÃO ter
os brinquedos 1, 2, 3, 4, 5 respectivamente, depois de comprar as n caixas.
Temos (para i, j, k, l em {1,2,3,4,5} distintos dois a dois):

Pr(Ni)=(0.8)^n   ((i) garante o 0.8; (ii) garante o ^n; há 5 termos
deste tipo)
Pr(Ni e Nj)=(0.6)^n (há C(5,2)=10 termos destes)
Pr(Ni e Nj e Nk)=(0.4)^n (C(5,3)=10 termos assim)
Pr(Ni e Nj e Nk e Nl)=(0.2)^n (C(5,4)=5 termos assim)
Pr(N1 e N2 e N3 e N4 e N5)=0^n=0 (se n=1)

O evento que me interessa é N1 ou N2 ou N3 ou N4 ou N5 (este é o evento não
completei a coleção, algum dos brinquedos me faltou). Usando aquelas leis
de De Morgan (argh!):

Pr(Não completar coleção) = Pr(N1 ou N2 ou N3 ou N4 ou N5) =
= Soma(Pr(Ni))-Soma(Pr(Ni e Nj))+Soma(Pr(Ni e Nj e Nk))-Soma(Pr(Ni e Nj e Nk
e Nl)) + Pr(N1 e N2 e ... e N5) =
= 5(0.8)^n - 10(0.6)^n + 10(0.4)^n - 5(0.2)^n

(Deixa eu fazer um reality check: fazendo as contas com esta expressão aí
dá P(1)=P(2)=P(3)=P(4)=1 e P(5)=601/625... Isto reflete que é impossível
completar a coleção com 1,2,3 ou 4 caixas, e a chance de fechar a coleção
com 5 caixas é 5!/5^6=24/625. Ok!)

Eu quero que isso seja menor que 10%, então a equação a resolver é:

P(n)=5(0.8)^n-10(0.6)^n+10(0.4)^n-5(0.2)^n  0.1

Argh, não tenho idéia de que método algébrico usar nesta caca Vou dar um
bicão só com o primeiro termo para obter uma primeira aproximação (na
esperança de que os outros sejam bem menores, afinal, olhe as bases deles!):
5(0.8)^n  0.1
(0.8)^n  0.02
n  ln(0.02)/ln(0.8) = 17.53 (usei uma calculadora; talvez desse para
estimar isso de outro jeito, mas eu vou na calculadora daqui para a frente)

Da natureza do problema, é claro que P(n) é não-crescente nos inteiros
positivos. Vamos experimentar alguns valores por perto do 17.53:

P(17)=5(0.8)^17-10(0.6)^17+19(0.4)^17-5(0.2)^17 ~= 11.090%
P(18)=5(0.8)^18-10(0.6)^18+19(0.4)^18-5(0.2)^18 ~= 8.9057%

Então é isso aí, a resposta é n=18 caixas!

Abraço,
Ralph
2008/5/19 Eduardo Estrada [EMAIL PROTECTED]:

  Olá, Fernando,

 Podemos considerar que a pessoa tenha comprado n caixas do produto, sendo
 que, destas, b1 caixas contendo o brinde 1, b2 caixas contendo o brinde 2, e
 assim por diante, de tal modo que:

 b1 + b2 + b3 + b4 + b5 = n

 O total de compras em que todos os brindes são contemplados corresponde ao
 número de soluções inteiras positivas da equação acima, e o total irrestrito
 de compras corresponde ao número de soluções inteiras não negativas. Esses
 valores são, respectivamente, os binomiais C(n-1,5-1) = C(n-1,4) e
 C(n+5-1,5-1) = C(n+4,4). Para que se cumpra o enunciado, façamos:

 C(n-1,4)/C(n+4,4) = 0,9,

 ou, expandindo,

 (1/240)n^4 - (19/24)n^3 + (7/48)n^2 - (95/24)n + 1/10 = 0

 A equação acima admite uma raiz real próxima de zero, que não convém, pois
 devemos certamente comprar ao menos 5 caixas, e outra em torno de 189,84.
 Logo, basta comprar 190 caixas para se garantir a probabilidade de 90 % de
 se adquirir os cinco brindes.

 Um abraço,
 Eduardo Luis Estrada

 - Mensagem original 
 De: Fernando Lima Gama Junior [EMAIL PROTECTED]
 Para: obm-l@mat.puc-rio.br
 Enviadas: Domingo, 18 de Maio de 2008 23:41:10
 Assunto: [obm-l] DESAFIO

 Suponha que uma indústria alimentícia coloque em seus produtos um brinde
 para incentivar as vendas para crianças. São 5 tipos de brindes possível e a
 idéia é fazer com que a pessoa colecione os brindes, mas será impossível
 descobrir qual brinde tem em uma determinada caixa antes de abrir o produto.
 Nesse caso, um colecionador dos brindes sortudo será aquele que ao comprar 5
 caixas do produto, cada uma com um brinde diferente. Acontece que como ele
 não sabe qual brinde tem dentro de cada caixa ele pode ter que comprar mais
 de 5 caixas para completar a coleção, já que podem vir brindes repetidos.
 Qual seria o número mínimo de caixas que a pessoa teria que comprar para
 assegurar, com 90% de chances, de que ela terá os 5 brindes?

 Fernando



 --
 Abra sua conta no Yahoo! 
 

Re: [obm-l] DESAFIO

2008-05-19 Por tôpico Ralph Teixeira
3 eh primo, 5 eh primo, 7 eh primo, 9 eh primo, 11 eh primo, 13 eh primo,
... ;) ;) ;)

2008/5/19 Maurício Collares [EMAIL PROTECTED]:

 On 5/19/08, Eduardo Estrada [EMAIL PROTECTED] wrote:
  Se eu fosse engenheiro, eu diria:

 O Ralph é formado em Engenharia da Computação, não? Isso não significa
 que ele trabalha como engenheiro hoje em dia, claro, mas...

 --
 Abraços,
 Maurício

 PS: Este email não deve ser levado a sério, foi só uma observação. Eu
 gosto bastante de piadinhas matemáticas sobre engenheiros, pra falar a
 verdade :P

 =
 Instruções para entrar na lista, sair da lista e usar a lista em
 http://www.mat.puc-rio.br/~obmlistas/obm-l.html
 =



Re: [obm-l] Polinômios de variável complexa

2008-05-15 Por tôpico Ralph Teixeira
Olá a todos novamente.

 Oi, J.R.. Por um lado, sua análise final está correta -- o lugar geométrico
é uma união de intervalos na reta real; mas, enquanto a princípio poderia
haver outros intervalos ou curvas no plano complexo (e para cada curva
teria de haver a sua espelhada, exatamente pelo raciocínio que você fez),
neste caso específico eles são vazios. Com a sua notação, eu diria que o seu
k1 é +Infinito, então não tem aquela fase de uma raiz real e duas complexas
conjugadas.

(Aliás, dá para mostrar que quando k tende a +Inf, as raízes se aproximam de
-Inf, -4 e -2, que ainda são reais.)

De fato, aquela minha solução **não** pressupõe que o domínio do polinômio
era só R desde o início não... Bom, eu pressupus que os *coeficientes* do
polinômio eram reais (e são, pois quando se diz que k é positivo
automaticamente k tem de ser real). Daqui sai que p(R) está contido em R
(ainda não estou dizendo nada sobre p(z) para z fora de R). Então a gente
*prova* que basta analisar raízes reais, pois:

i) As primeiras linhas daquela minha solução *mostram* que há 3 raízes
reais; o raciocínio é válido pois p(x) tem coeficientes reais, então
*coincide* com um polinômio de R em R. As 3 raízes reais que a gente acha
quando pensa que o domínio é R não desaparecem quando o domínio é expandido
para C.
ii) Mas o polinômio tem grau 3, então só pode ter 3 raízes complexas (um
polinômio de grau n tem n raízes complexas, contando multiplicidade, mesmo
que os coeficiemtes sejam complexos).
iii) Mas então todas as raízes são reais!

Então, se o problema é onde estão as possíveis raízes, agora sim, basta
analisar as raízes reais -- não há outras!

---///---

Agora vem a parte mais difícil... depois de sacar que as raízes são sempre
reais, a chave do problema é: ao invés de pensar...

Tá aqui um k. Será que eu consigo descobrir as raízes x1, x2 e x3
associadas a este k, para depois colar tudo e responder este problema?
(que é dífícil pra caramba, pois basicamente você teria que resolver uma
equação de 3o grau, ou pelo menos descobrir propriedades destas raízes em
função de um parâmetro k)

...é muito melhor pensar assim, ao contrário:

Tá aqui um número x, candidato a raiz. Será que ele **pode** ser raiz do
polinômio para algum k positivo? Isto é, será que eu arrumo um k que faz
este x ser raiz?
(esta é fácil, o único k que pode funcionar
é k0=-(x+1)(x+3)(x+5)/(x+2)(x+4); se este k0 for positivo, aquele x é raiz
do polinômio, para k=k0)

Abraço,
  Ralph

P.S.: A afirmação da Alane vale para polinômios **com coeficientes reais**
(que é o caso neste problema). Se os coeficientes não forem reais, você pode
ter algo como p(x)=x-i, que tem a raiz i mas não tem a conjugada, ou
quaisquer raízes complexas, de fato.

2008/5/13 J. R. Smolka [EMAIL PROTECTED]:

 Primeiramente obrigado à Alane e ao Ralph pelas sugestões. Vamos por
 partes:

 A Alane lembrou que se z é uma raiz do polinômio, então o conjugado
 complexo de z também será raiz. Não tenho certeza absoluta, mas acho que
 este princípio se mantém para funções polinomiais de C em C.

 O Ralph fez uma análise como se o polinômio fosse função de R em R, que não
 é o caso. Mas me deu algumas idéias sobre como atacar o problema. Até agora
 estou apenas no nível qualitativo. Depois vou tentar resolver a álgebra (a
 menos que alguém me mostre que esta linha de raciocínio não tem futuro :-)).
 O que estou pensando é:

 1) Se k=0, P(x) tem três raízes reais em x=-1, x=-3 e x=-5.

 2) Deve existir uma faixa de valores 0k=k1 para a qual P(x) ainda
 apresenta três raízes reais, que vão excursionar em algum trecho do
 semi-eixo real negativo. A investigar: (a) Qual o valor de k1? (estudo de
 máximos/mínimos/inflexões via P'(x)=0 deve ajudar nisso); (b) qual(is)
 intervalo(s) do semi-eixo real negativo é(são) percorrido(s) pelas raízes?

 3) Se kk1 então deve continuar a existir uma raiz real (que também
 excursiona no semi-eixo real negativo) e um par de raízes complexas
 conjugadas. Sobre a raiz real a pergunta é: qual o seu intervalo de
 excursão? Sobre as raízes complexas o raciocínio é mais longo...

 4) Temos que P(x)=x^3+(k+9)x^2+(6k+23)x+(8k+15). Se z=r.e^(i.a) é raiz de
 P(x), então r^3.e^(i.3a)+(k+9)r^2.e^(i.2a)+(6k+23)r.e^(i.a)+(8k+15)=0. Então
 temos quatro componentes, com argumentos complexos 0 (número real), a, 2a e
 3a. De cara enxergo como candidatos a raiz os números complexos na forma
 z=r.e^(i.pi/2), onde o valor de r depende de k. Desta forma, o componente de
 argumento complexo 2a=2.pi/2=pi  pode anular o componente de argumento
 complexo 0, e o componente de argumento complexo 3a=3.pi/2 pode anular o
 componente de argumento complexo a=pi/2. Se isto realmente for possível
 (tenho que verificar a álgebra), então z excursiona em um intervalo do
 semi-eixo imaginário positivo, com este intervalo limitado em (pelo menos)
 um valor que é função de k1, e o seu conjugado complexo vai ter um
 comportamento espelhado no semi-eixo imaginário negativo.

 Então minha primeira 

Re: [obm-l] Polinômios de variável complexa

2008-05-15 Por tôpico Ralph Teixeira
Oi Smolka.

Talvez a minha última mensagem ainda não tenha chegado... Você tem razão em
prestar atenção ao fato de que a variável é complexa, e nem todos os
Teoremas de variável real valem. Mas, repito, a soluão que eu tinha vale
mesmo que x seja uma variável complexa. Deixe-me dizer tudo da seguinte
forma para ressaltar bem a diferença que você está colocando (e que é
saudável):

i) Eu tenho um polinômio P(x)=(x+1)(x+3)(x+5)+k(x+2)(x+4) de variável
complexa;
ii) Note que P tem grau 3, então tem , no máximo, 3 raízes complexas
(**este** Teorema vale para polinômios de variáveis e coeficientes
complexos).
iii) Considere agora o polinômio Q(x)=(x+1)(x+3)(x+5)+k(x+2)(x+4), onde a
única diferença é que Q tem x como variável **real**. Ou seja, P:C-C, mas
Q:R-R. Eu posso fazer isto pois, se x é real, Q(x) é real.
iv) O raciocínio daquela solução mostra que Q tem 3 raízes reais, digamos,
x1, x2 e x3.
v) Mas, note que, quando x é real, então P(x)=Q(x). Em outras palavras,
P(x1)=P(x2)=P(x3)=0. Ou seja, x1, x2 e x3 também são raízes de P, e são
reais.
vi) Mas, como P só tem no máximo 3 raízes complexas, elas são os mesmos x1,
x2 e x3. Assim, elas são reais!

Então não adianta procurar raízes não reais, elas não existem!

Isto dito os raciocínios que você está fazendo são super legais para
tentar enxergar o que o polinômio faz com o plano de Argand-Gauss. De uma
certa maneira, eu devia ficar quieto e deixar você explorar o assunto, pois
você vai achar um monte de coisas legais assim, mesmo que não
resolvam *ESTE* problema! :)

Abraço,
 Ralph
2008/5/15 J. R. Smolka [EMAIL PROTECTED]:

 Obrigado ao Ojesed pela idéia de fazer uma substituição de variável do tipo
 z=(x+1) para simplificar a análise. Deve ser útil. Mas não dá para aplicar
 Cardano diretamente, porque (repito) este é um polinômio de variável
 complexa. Cardano serve para resolver equações cúbicas de variável real
 (possivelmente válido até se os coeficientes forem complexos), que não é o
 caso aqui.

 Não é a primeira vez que esta confusão acontece. Será porque a variável
 usada é x (que induz a pensar em números reais) em vez de z (como é comum
 para números complexos)? Pensar em x como um vetor de coordenadas
 cartesianas (a,b) ou polares (|x|,arg(x)) ajuda o raciocínio.

 Para os que (ainda) se interessarem no problema, lembro que uma função de C
 em C tem como domínio todo o plano de Argand, e a imagem será pelo menos um
 subconjunto (não necessariamente contínuo) de todo o plano de Argand.

 Neste caso, como a função é um polinômio de grau 3, cada ponto x do plano
 domínio é mapeado para um ponto do plano imagem através das translações e
 rotações provocadas pela  potenciação de x e pela multiplicação de x por
 números reais.

 A questão inicial, então, é descobrir que região do plano de Argand pode
 possuir raízes de P(x)=0. Depois determinar a localização destes pontos
 nesta região (em função de k, que é um número real). E, finalmente, analisar
 a figura geométrica descrita pelo deslocamento destes pontos no plano de
 argand quando k varia entre 0 e +inf.

 Exemplo do raciocínio da primeira parte: não existe x tal que P(x)=0 na
 região do plano de Argand definida por 0=arg(x)pi/4 porque neste caso
 im(x)0, im(x^2)0 e im(x^3)0, o que torna impossível que im(P(z))=0.

 Como disse antes, consigo enxergar as regiões do plano de Argand definidas
 por arg(z)=pi/2 (o semi-eixo imaginário positivo, excluída a origem) e por
 arg(z)=pi (o semi-eixo real negativo, também excluída a origem) como
 candidatas a hospedeiras das raízes de P(x)=0. Mas será que a minha visão
 geométrica está correta e completa?

 Ainda não desenvolvi a álgebra destes casos para verificar se um, outro ou
 ambos são compatíveis com P(x) (afinal de contas, estou fazendo isto por
 puro diletantismo, e o tempo livre para raciocinar livremente anda meio
 curto ;-)). Mas continuo interessado em idéias a respeito.

 [ ]'s

 Esta questão foi da prova de álgebra do IME 1976/1977. Vou transliterar um
 pouco o enunciado.

 Seja P(x)=(x+1)(x+3)(x+5)+k(x+2)(x+4), com x complexo e k real positivo.
 Desenhar no plano complexo o lugar geométrico  das raízes de P(x)=0 para
 todos os valores possíveis de k.

 Tentei o seguinte: se z=a+bi é raiz de P(x), então P(z)=0, o que implica
 que Re[P(z)]=0 e Im[P(z)]=0, então daria para obter expressões em função de
 a e b que descrevessem o lugar geométrico procurado. Só que as expressões
 parecem intratáveis.

 Alguma outra idéia?

 *J. R. Smolka*



Re: [obm-l] FUVEST-93

2008-05-14 Por tôpico Ralph Teixeira
Aaaahh... levei um tempo para achar algum erro, acho que entendi:

a) Se a=1, b=-2 e c=0, temos x^2-2|x|=0, que tem as raízes x=0, x=-2 e x=2.
Então (A) é FALSA.
b) Supondo que x é real, então temos ax^2+bx+c=0 ou ax^2-bx+c=0. Assim, x
teria de ser uma das 4 raízes destas 2 quadráticas... ah, mas pera aí, quem
disse que são quadráticas? Poderia ser a=b=c=0, e então teríamos infinitas
raízes Talvez esta seja a razão da anulação: (B) é FALSA. (Se eles
dissessem que a0, (B) seria verdadeira)
c) Se a=1, b=0 e c=1, teríamos x^2+1=0, que não tem raízes reais. Então (C)
é FALSA.
d) Por outro lado, o exemplo de (a) mostra que (D) é FALSA.
e) Se a=1, b=c=0, temos x^2=0, que não tem raízes distintas. (E) é FALSA.

Abraço,
 Ralph
2008/5/14 arkon [EMAIL PROTECTED]:

  *Pessoal essa questão foi anulada pela Universidade, poderiam me explicar
 qual o motivo da anulação?*

 * *

 *(FUVEST-93) Quaisquer que sejam os números reais a, b e c pode-se afirmar
 que a equação ax^2 + b|x| + c = 0:*

 * *

 *a) tem, no máximo, duas raízes reais distintas.*

 *b) tem, no máximo, quatro raízes reais distintas.*

 *c) tem pelo menos uma raiz real.*

 *d) não possui raízes reais.*

 *e) tem sempre raízes distintas.*

 * *

 *DESDE JÁ AGRADEÇO*



Re: [obm-l] GUARDA-CHUVAS

2008-05-09 Por tôpico Ralph Teixeira
A resposta curta eh: 5! maneiras possiveis de distribuir, e C(5,3) maneiras
favoraveis de exatamente 3 receberem corretamente. Entao p=C(5,3)/5!=1/12.
Mas, para justificar isso direitinho, serei prolixo, porque, com
combinatoria e probabilidade, todo cuidado eh pouco. :)

Uma distribuicao de guarda-chuvas eh basicamente uma permutacao dos numeros
de 1 a 5. Algumas possiveis distribuicoes seriam:

12345 (Todo mundo recebe o guarda-chuva que lhe pertencia)
12534 (Apenas os visitantes 1 e 2 recebem seus guarda-chuvas; a pessoa 3
recebe o guarda-chuva da 5, a pessoa 4 recebe o da 3, e a pessoa 5 recebe o
da 4)
15342 (1, 3 e 4 recebem seus guarda-chuvas, os outros dois estao trocados)

Entao temos 5!=120 maneiras de distribuir os guarda-chuvas, *todas
igualmente provaveis* (assim eh que interpreto o ao acaso do enunciado).

Agora, em quantas destas temos *exatamente* 3 pessoas recebendo seus
guarda-chuvas originais? Para descobrir uma destas
possibilidades, precisamos (e BASTA!) escolher 3 pessoas para receberem os
guarda-chuvas originais (as outras 2 terao que receber os guarda-chuvas
trocados) -- note que eh soh isso mesmo:

i) Escolhidas 3 pessoas, soh tem um jeito de distribuir os 5 guarda-chuvas
de maneira favoravel (isto eh, para que soh estas 3 recebam o certo;
afinal, as outras 2 tem de receber guarda-chuvas trocados entre si, e soh
tem um jeito disso acontecer); por exemplo, se escolhermos 1, 3 e 4 temos
apenas a distribuicao 15342, e nenhuma outra;
ii) Distribuidos os 5 guarda-chuvas do jeito do enunciado (com apenas 3
recebendo corretamente), ha apenas uma maneira de determinar as 3 que
receberam certo; por exemplo, de 15342, temos a escolha 1, 3 e 4, e
nenhuma outra.

Entao eu conto C(5,3)=5!/(3!2!) maneiras *igualmente provaveis* do nosso
evento acontecer, dentre 5! totais igualmente provaveis. A probabilidade eh
1/(3!2!) = 1/12.

Abraco,
 Ralph

P.S.: Por este mesmo raciocinio, se fossem N guarda-chuvas, a probabilidade
de exatamente m receberem os guarda-chuvas corretos eh C(N,m).D(N-m) / N! =
D(N-m)/(m!(N-m)!), onde D(x) eh o numero de desarrumacoes (permutacoes
caoticas) de x elementos. Entao o Pedro Barboza tem razao... mas, no
problema do Arkon, N-m=2, e eh facil ver que D(2)=1 (soh tem um jeito de
desarrumar dois elementos), entao nao foi necessario entrar a fundo nas
permutacoes caoticas.

2008/5/9 pedro barboza [EMAIL PROTECTED]:


 a solução eh por permutação caótica





 --
 Date: Thu, 8 May 2008 11:05:20 -0300
 Subject: [obm-l] GUARDA-CHUVAS
 From: [EMAIL PROTECTED]
 To: obm-l@mat.puc-rio.br

  *Pessoal, alguém se habilita*

 * *

 *(ENAD) Ao entrar em casa de amigos, cinco pessoas deixam seus
 guarda-chuvas com a dona da casa. Quando as pessoas resolvem pedi-los de
 volta para sair, a dona da casa constata que todos eles são aparentemente
 iguais, e resolve distribuí-los ao acaso. Qual a probabilidade de que
 exatamente três pessoas recebam cada uma o seu próprio guarda-chuva?*

 * *

 *(A) 1/12.(B) 1/6.  (C) 1/4.  (D) 1/3.(E)
 5/12.*

 **

 *GABARITO: LETRA A*


 --
 Conheça já o Windows Live Spaces, o site de relacionamentos do Messenger! Crie
 já o seu! http://www.amigosdomessenger.com.br/



Re: [obm-l] Polinômios de variável complexa

2008-05-09 Por tôpico Ralph Teixeira
Acho que a primeira coisa a fazer eh notar que as 3 raizes sao reais! De
fato:

i) Polinomio de 3o grau, termo principal = 1.x^3: P(-Inf)=-Inf e
P(+Inf)=+Inf;
ii) P(-4)=30 e P(-2)=-30

Assim, ha uma raiz real em (-Inf,-4), outra em (-4,-2) e a terceira em
(-2,+Inf). Isto dah as 3 raizes reais, entao a gente nao precisa se
preocupar com os complexos!

Agora a gente em que ver que valores destes intervalos podem, de fato, ser
raiz da equacao polinomial P(x)=0.

Para tanto, perguntamos -- para que valores de x tem-se
(x+1)(x+3)(x+5)/((x+2)(x+4))0 ? Dada um solucao x=a desta inequacao, tem um
k que faz a ser raiz daquele polinomio (qual?); e vice-versa, se tem um k
positivo tal que a eh raiz do polinomio, entao a satisfaz esta inequacao!

Resposta final: (-Inf,-5) U (-4,-3) U (-2,-1).

2008/5/9 J. R. Smolka [EMAIL PROTECTED]:

 Esta questão foi da prova de álgebra do IME 1976/1977. Vou transliterar um
 pouco o enunciado.

 Seja P(x)=(x+1)(x+3)(x+5)+k(x+2)(x+4), com x complexo e k real positivo.
 Desenhar no plano complexo o lugar geométrico  das raízes de P(x)=0 para
 todos os valores possíveis de k.

 Tentei o seguinte: se z=a+bi é raiz de P(x), então P(z)=0, o que implica
 que Re[P(z)]=0 e Im[P(z)]=0, então daria para obter expressões em função de
 a e b que descrevessem o lugar geométrico procurado. Só que as expressões
 parecem intratáveis.

 Alguma outra idéia?

 [ ]'s

 *J. R. Smolka*



Re: [obm-l] Número

2008-04-30 Por tôpico Ralph Teixeira
Acho que dá para acelerar um tiquinho assim:

i) Caso c=9.
Então N=c^3=729; daqui a7, e a^3=7^3=343. Portanto, N=a^3+c^31000,
absurdo.

ii) Caso c9. Aí:

N=100a+10b+c=a^3+b^3+c^3
N+1=100a+10b+(c+1)=a^3+b^3+(c+1)^3 (pois c+1 é o último dígito, sim)

Subtraindo uma da outra, sai c=0 (pois c=-1 não presta). Então a gente tem
que resolver:

100a+10b=a^3+b^3 ou seja a(100-a^2)=b(b^2-10). Em particular, note que o
lado esquerdo é positivo, então devemos ter b=4. Agora eu ia na força bruta
(fazendo 15 ao invés de 100 contas): os possíveis valores da primeira
expressão são (fazendo a=1,2,...,9): 99, 192, 273, 336, 375, 384, 357, 288 e
171. Os possíveis valores da segunda (fazendo b=4,5,6,7,8,9): 24, 75, 156,
273, 432, 639. O único elemento comum é 273, então a=3 e b=7, que nem o
pessoal achou.

Abraço,
 Ralph



2008/4/30 Henrique Rennó [EMAIL PROTECTED]:

 Ola Joao Gabriel,

 Acredito que o numero N seja 370 e portanto a resposta e' 8, pois seus
 divisores sao 1, 2, 5, 10, 37, 74, 185, 370.

 Seja N = abc, tem-se abc = a^3 + b^3 + c^3

 1) Se 0 = c = 8 entao ab(c+1) = a^3 + b^3 + (c+1)^3

 2) Se c = 9 e 0 = b = 8 entao a(b+1)0 = a^3 + (b+1)^3

 3) Se c = 9 e b = 9 e 1 = a = 8 entao (a+1)00 = (a+1)^3

 4) Se c = 9 e b = 9 e a = 9 entao 1000 = 1^3

 Na condicao 1) tem-se:

 ab(c+1) - abc = a^3 + b^3 + (c+1)^3 - (a^3 + b^3 + c^3)
 1 = a^3 + b^3 + c^3 + 3c^2 + 3c + 1 - a^3 - b^3 - c^3
 3c^2 + 3c = 0
 c(c+1) = 0

 c = -1 nao e' possivel

 Se c = 0 deve-se testar as possibilidades. Como N termina em 0 tem-se que
 a^3 e b^3 terminam em digitos que somados resulta em 10, ou seja, 1+9 ou 2+8
 ou 3+7 ou 4+6 ou 5+5.

 1^3 = 1
 2^3 = 8
 3^3 = 27
 4^3 = 64
 5^3 = 125
 6^3 = 216
 7^3 = 343
 8^3 = 512
 9^3 = 729

 Testando os valores de a^3 + b^3, onde os ultimos digitos de a^3 e b^3
 somados resulta em 10:

 O simbolo != significa diferente.

 ultimos digitos: 1 + 9 == a = 1 e b = 9 == a^3 + b^3 = 1^3 + 9^3 = 1 +
 729 = 780 != 190
 ultimos digitos: 2 + 8 == a = 8 e b = 2 == a^3 + b^3 = 8^3 + 2^3 = 512 +
 8 = 520 != 820
 ultimos digitos: 3 + 7 == a = 7 e b = 3 == a^3 + b^3 = 7^3 + 3^3 = 343 +
 27 = 370 != 730
 ultimos digitos: 4 + 6 == a = 4 e b = 6 == a^3 + b^3 = 4^3 + 6^3 = 64 +
 216 = 280 != 460
 ultimos digitos: 5 + 5 == a = 5 e b = 5 == a^3 + b^3 = 5^3 + 5^3 = 125 +
 125 = 250 != 550
 ultimos digitos: 6 + 4 == a = 6 e b = 4 == a^3 + b^3 = 6^3 + 4^3 = 216 +
 64 = 280 != 640
 ultimos digitos: 7 + 3 == a = 3 e b = 7 == a^3 + b^3 = 3^3 + 7^3 = 27 +
 343 = 370 = 370 OK!
 ultimos digitos: 8 + 2 == a = 2 e b = 8 == a^3 + b^3 = 2^3 + 8^3 = 8 +
 512 = 520 != 280
 ultimos digitos: 9 + 1 == a = 9 e b = 1 == a^3 + b^3 = 9^3 + 1^3 = 729 +
 1 = 730 != 910

 Como encontramos uma resposta para o caso 1 nao seria necessario verificar
 os casos 2, 3 e 4.

 Assim, N = 370.

 2008/4/30 João Gabriel Preturlan [EMAIL PROTECTED]:

   Preciso de ajuda:
 
 
 
  Um número natural N de três algarismos é igual a soma dos cubos dos seus
  dígitos. Um número N+1 tem a mesma propriedade. Qual é o número de divisores
  inteiros de N?
 
 
 
  a)4
 
  b)6
 
  c)8
 
  d)12
 
  e)16
 
 
 
  Desde já agradeço.
 
 
 
  JG.
 
  No virus found in this outgoing message.
  Checked by AVG.
  Version: 7.5.524 / Virus Database: 269.23.6/1404 - Release Date:
  29/04/2008 18:27
 



 --
 Henrique


Re: [obm-l] equação

2008-04-25 Por tôpico Ralph Teixeira
A ideia fundamental eh notar que a espressao do lado esquerdo eh NO MINIMO
1/64, e isto soh ocorre quando tanx=1. Vejamos como mostrar isto

SOLUCAO I (COM CALCULO II):

Considere o problema de minimizar f(x,y)=x^14+y^14 sujeito aa restricao
x^2+y^2=1. Use Lagrange, o minimo satisfaz:

14x^13=2kx
14y^13=2ky
(ou 2x=2y=0, o que claramente nao serve)

Casos:
i) Se x=0, entao y=+-1, entao f=1.
ii) Se y=0, entao x=+-1, entao f=1.
ii) Se x,y0 entao 14x^12=14y^12=2k; assim, x=+-y, isto eh, x e y sao ambos
+-raiz(2)/2, e f(x,y)=1/64.

Assim, o minimo de f(x,y) eh 1/64, que soh ocorre quando x=+-raiz(2)/2 e
y=+-raiz(2)/2.

Mas x=sint, y=cost satisfaz a restricao! Assim, a expressao do lado esquerdo
eh sempre maior ou igual a 1/64, e a igualdade soh ocorre quando
sinx=+-raiz2/2 e cosx=+-raiz2/2. Assim, x=kpi/2+pi/4 eh a solucao da sua
equacao, k inteiro.

SOLUCAO II (COM CALCULO I):
Seja f(x)=(sinx)^14+(cosx)^14. Vamos encontrar o minimo de f(x), digamos, no
intervalo [0,2pi], que eh um periodo de f. Note que
f`(x)=14sinxcosx((sinx)^12-(cosx)^12). Fazendo f'(x)=0, temos 3 casos:
i) sinx=0 implica f(x)=1
ii) cosx=0 implica f(x)=1
iii) (sinx)^12=(cosx)^12 implica tanx=+-1, entao sinx e cosx sao +-
raiz(2)/2, entao f(x)=1/64.
Como f tem de ter um minimo no intervalo fechado [0,2pi], ele eh 1/64, que
soh ocorre em x=pi/4,3pi/4, 5pi/4 r 7pi/4. Como f eh periodica, este minimo
soh poderah ocorrer nos pontos que correspondem a estes (modulo um periodo).
Assim, as unicas solucoes sao x=kpi/2+pi/4, k inteiro.

SOLUCAO III (SEM CALCULO, MAS COM MUITO BRACO):

i) Pode ser cosx=0? Entao seria sinx=1, o que nao dah certo.
ii) Entao divida tudo por (cosx)^14. Usarei t=(tanx)^2 para economizar bits:

(tanx)^14 + 1 = (secx)^14 / 64
64(t^7 + 1) = (t+1)^7
Abre tudo (coragem e binomio de Newton!):
63t^7-7t^6-21t^5-35t^4-35t^3-21t^2-7t+63=0
Note que t=1 eh raiz, e dupla. Elimine-a via Briot-Ruffini (coragem!):
(t-1)^2.(63t^5+119t^4+154t^3+154t^2+119t+63)=0

Soh que o polinomio da direita nao tem raizes positivas nem zero (se
tivesse, como eh que aquele bando de numero positivo somado dava zero?). Por
outro lado, soh interessam as raizes positivas (lembra que t eh a tangente
AO QUADRADO; t negativo nao dah solucao alguma!). Entao soh pode ser t=1,
tanx=+-1, e entao x=kpi/2+pi/4 com k inteiro.

---///---

Tenho certeza que ha outra solucao sem calculo (eu jah vi este problema em
algum lugar, acho ateh que caiu em alguma prova quando eu estava na Turma
IME/ITA, e eu acho que resolvi do jeito III, e todo mundo ficou embasbacado
com a completa **feiura** da minha solucao bracal).

Abraco bracal,
 Ralph


On Thu, Nov 1, 2001 at 12:12 AM, Pedro [EMAIL PROTECTED] wrote:

  Amigos da lista , me dê um idéia para essa equação:

 --
 [image: sin^{14}{x} + cos^{14}{x} = \frac {1}{64}]

2181a1a59ab195dd3341a5c7802bbd4efacbba7e.gif

Re: [obm-l] função

2008-04-17 Por tôpico Ralph Teixeira
Bom, vou supor que f(cx+d)=ax+b para todo x real.

Entao, defina y=cx+d. Se c0, entao x=(y-d)/c, entao:

f(y)=ax+b=a(y-d)/c+b para todo y real (isto porque enquanto x percorre a
reta real, y=cx+d tambem percorre a reta toda).

isto eh

f(y)=(a/c)*y+(bc-ad)/c

Esta frase aqui garante que f eh uma funcao de primeiro grau desde que
tambem tenhamos a0.

Em suma:

(1) SE a,c0, entao f eh do primeiro grau.

Agora vamos aos outros casos:

(2) Se c0 e a=0, entao f(y)=b para todo y mostra que f eh constante...
Bom, na minha nomencalatura, neste caso, f NAO EH de primeiro grau (mas
ainda eh uma funcao afim, cujo grafico ainda eh uma reta).

(3) Se c=0 e a0, entao a informacao dada diz que f(d)=ax+b para todo x
real, o que eh absurdo -- f(d) nao poderia assumir varios valores
simultaneamente.

(4) Enfim, se c=a=0, entao o dado eh f(d)=b, o que soh da informacao sobre
um ponto do grafico da funcao. Entao f pode ser de primeiro grau, mas pode
nao ser tambem.

Em suma, acho que o que voce quer eh a condicao (1) -- se a0 e c0, entao
f(cx+d)=ax+b para todo x real implica que f eh de primeiro grau -- mais
exatamente, que f(x)=(a/c)*x+(bc-ad)/c.

Abraco,
 Ralph

2008/4/17 João Gabriel Preturlan [EMAIL PROTECTED]:

  Eu estava resolvendo um exercício sobre funções compostas e surgiu a
 seguinte dúvida que não consigo resolver:



 Considerando a função f(cx + d) = ax + b :



 Em que casos que f(x) é um função do primeiro grau?



 Não sei se eu fui claro: quando que a partir daquela função composta eu
 posso garantir que, antes mesmo da resolução da função, f(x) é certamente do
 primeiro grau.



 É possível provar isso matematicamente?



 Desde já agradeço muito.



 JG.

 No virus found in this outgoing message.
 Checked by AVG.
 Version: 7.5.524 / Virus Database: 269.23.1/1384 - Release Date:
 17/04/2008 15:47



Re: [obm-l] Convolução

2008-04-16 Por tôpico Ralph Teixeira
O pessoal da John Hopkins tem uns applets legais para entender melhor o que
eh convolucao (e Fourier, e coisas afins):

www.jhu.edu/signals/index.html

Quanto a aplicacoes, ha quilos delas em processamento de sinais (pois
qualquer processamento linear e invariante por translacao tem que ser uma
convolucao). Eu tenho um livrinho que fala de algumas aplicacoes a
processamento de imagens (foi do 23o coloquio do IMPA, de 2001, e o IMPA
vendia a R$10 cada, mas acho que estah esgotado)... Um site onde voce pode
encontra-lo eh o de uma disciplina que eu dei ano passado:

www.uff.br/FMID (clique em Materiais e procure o arquivo do dia 28/03 --
eh o livro todo, em PDF)

Abraco,
Ralph

2008/4/16 Douglas Ribeiro Silva [EMAIL PROTECTED]:

 Olá pessoal!

 Alguém tem indicação de algum site ou bom livro que dê maiores
 detalhes sobre aplicações da integral de convolução? Agradeço qualquer
 ajuda

 Abraços!

 =
 Instruções para entrar na lista, sair da lista e usar a lista em
 http://www.mat.puc-rio.br/~obmlistas/obm-l.html
 =



Re: [obm-l] Onde está o erro???

2008-04-13 Por tôpico Ralph Teixeira
Eh, eh um problema de notacao -- frequentemente, a literatura confunde (para
economizar linguagem) periodo com periodo fundamental.

Entao, se ele quer dizer que as funcoes f+g e f.g TEM periodo P, estah
correto. Elas tem periodo P sim. Por exemplo, cosx.sin5x tem periodo 2pi
(dentre outros, inclusive pi, que eh o fundamental).

Agora, este periodo nao eh necessariamente o FUNDAMENTAL, como voce mesmo
destacou.

(Um exemplo mais drastico ainda eh tomar f=qualquer coisa periodica e g=-f.
Entao f+g=0 tem qualquer periodo, nao soh o original de f.)

Abraco,
   Ralph



2008/4/13 [EMAIL PROTECTED]:

 VI NO LIVRO DO AREF E NO SITE RUMO AO ITA O TEOREMA ABAIXO.
 Sejam f e g duas funções periódicas, definidas for y=f(x) e y=g(x), cujos
 períodos são, respectivamente, p1 e p2, com p1 diferente de p2. Se
 p1/p2=m/n,onde
 m e n são inteiros positivos e primos entre si, então as funções definidas
 por f+g e f.g são periódicas e seu período(P) é P=n.p1=n.p2.

 POLÊMICA

 O período da função f(x)=cosx.sen5x aplicando o teorema acima é
 período=2pi.No
 entanto, temos a seguinte definição: Se f é periódica, então f(x+p)=f(x),
 para todo x pertencente no domínio de f, onde o menor p positivo que
 satisfaz
 a sentença anterior chamaremos de período  principal ou primitivo de f.
 Então,
 resolvendo pela definição, encontramos período igual a pi.

 E agora o que faço? Existem restrições? O teorema tá errado? Agradeço
  antecipadamente
 pelos esclarecimentos.



 =
 Instruções para entrar na lista, sair da lista e usar a lista em
 http://www.mat.puc-rio.br/~obmlistas/obm-l.html
 =



Re: [obm-l] Sobre a Soma!!!

2008-04-09 Por tôpico Ralph Teixeira
Para 8a serie, acho que o jeito eh o seguinte:

a) Calcule S = 1 + 11 + 111 +  + ... + (111...111)
Note que 10S =10 + 110 + 1110 +  + (111...110) + (111...1110)
(Marque com chave por baixo que aqueles termos finais tem n-1 e n digitos 1,
respectivamente)

Subtraindo a segunda menos a primeira, fica:
9S = -1 -1 -1 -1... -1 +(111...1110) = 111...1110 - n
Agora 111...1110 = 999...9990/9 = (10^(n+1)-10)/9

Assim, 9S = (10^(n+1)-10)/9 - n, e entao S = (10^(n+1)-10-9n)/81.

b) Calcule A= 1 + 22 + 333 +  + ... +n(111111)
Mesmo truque: 10A =  10 + 220 + 3330 + ... +(n-1)(111110) +
n(111...1110)

Subtraindo: 9A = -1 -12 -113 -1114 -5... -(1110+n) +
n(111...1110)
Agora eh mais dificil -- a gente tem que separar os 1s do resto:

9A = n(111...1110) - (10+110+...+(111...0)) - (1+2+3+...+n)   (aquele -1
inicial veio parar no segundo termo)
O primeiro termo eh n.(10^(n+1)-10) / 9, de novo;
O segundo termo eh 10 vezes a soma S do item anterior;
O terceiro termo eh n(n+1)/2 (use soma dos termos da P.A., ou use o truque
de Gauss para mostrar isto para eles)
Substitua tudo, agora o resto eh contalhada feiosa.

Abraco,
  Ralph

2008/4/9 Pedro Júnior [EMAIL PROTECTED]:

 Olá Saulo...

 Rapaz vc não sabe oque aconteceu sobre tal continha...

 Estava resolvendo algumas questões de somas e produtos com alguns alunos,
 na minha lista de exercícios tem o seguinte problema:

 encontre a soma: 1 + 11 + 111 +  + ... + (111...1)
 onde (111...1) tem exatamente n dígitos, probleminha clássico encontrado
 nos livros da mir e outros mais, bom só que na minha lista havia colocado
 tudo quanto é questão que encontrei na net a respeito de somas e produtos, e
 um deles é exatamente esse

 1 + 22 + 333 +  + ... + n(111...1) = ?

 só que lista que apanhei na net não tem a resolução e inicialmente achei
 muito simples. Rapaz quando comecei a ver de perto o que acabara de colocar
 no quadro, meu amigo, nada de chegar a um resultado fechado, assim como no
 problema anterior, então fiquei e ainda estou, vi sua resolução, mas teve
 uma passagem que falou em derivada, e estou ministrando aula para crianças
 de 8º ano, 1º ano  e 2º anos, daí mais uma dificuldade.

 bom vou continuar tentando, espero ter sorte de encontrar uma expressão
 mais simples que seja equivalente.

 Abraços e muito obrigado.

 Pedro Jr



Re: [obm-l] Conjuntos numéricos na Reta...

2008-03-27 Por tôpico Ralph Teixeira
Bom, a resposta à sua pergunta depende do que se entende por números
na reta. Se não há definição precisa de reta numérica, não dá para
discutir se todos os números dela (ela? que ela?) estão nos reais ou
não.

Uma solução rápida, limpa, simples e sem graça é **DEFINIR** a reta
numérica como o conjunto dos números reais. Rápido, limpo e simples
(mas sem graça). :) :) A boa notícia é que deste jeito a reta numérica
herda todas as propriedades dos números reais, inclusive aquela de que
todo conjunto numérico limitado tem um supremo.

Assim, a resposta usual é garantimos que não há número, na reta, que
não se enquadre no conjunto dos reais pois definimos a reta numérica
como o conjunto dos reais. :P

Abraço,
   Ralph

P.S.: Por outro lado, dá para definir números surreais (veja
http://en.wikipedia.org/wiki/Surreal_numbers). Para trabalhar com
eles, algumas propriedades antigas têm de ser descartadas (por
exemplo, neles a=b e b=a não implica a=b), mas eles são bem bacanas,
são uma extensão dos reais que não tem nada a ver com os complexos e o
pessoal tenta pensar neles ainda numa espécie de reta, usando
nuvens ao invés de pontos, incluindo uns pontos no infinito e
fazendo várias outras barbáries. Por incrível que pareça, no final,
tudo funciona, as construções são justificadas formalmente e estes
objetos têm aplicações.

2008/3/27 Paulo - Uniredes [EMAIL PROTECTED]:
 Como é que sabemos que os conjuntos já conhecidos são suficientes para
 representar números da Reta Real ? Existe alguma prova de que eles são
 necessários e sufucientes ?

 Explico:

 Temos os naturais
 Depois estendemos o conceito para os inteiros...
 Depois os racionais...
 Depois os irracionais...

 Bom, que me garante que não há número, na reta, que não se enquadre em
 qualquer desses conjuntos ?

 Há algum teorema mágico que diga isso, como existe o maravilhoso Teorema
 de Gödel sobre a inconsistência da lógica ?


 []s

 ---
 Paulo C. Santos (PC)
 e-mail: [EMAIL PROTECTED]
 Homepage: http://uniredes.org
 Tel.: (21) 2510.8783 - Cel.: (21) 8753-0729
 
 MS-Messenger: [EMAIL PROTECTED]


 =
 Instruções para entrar na lista, sair da lista e usar a lista em
 http://www.mat.puc-rio.br/~obmlistas/obm-l.html
 =


=
Instruções para entrar na lista, sair da lista e usar a lista em
http://www.mat.puc-rio.br/~obmlistas/obm-l.html
=


Re: [obm-l] LADOS DO TRIÂNGULO

2008-03-26 Por tôpico Ralph Teixeira
Os lados sao c, b=c+1 e a=c+2 (pois A=2C eh o maior angulo, e C eh o
menor; o outro eh B=180-3C).

Entao, pela lei dos senos:

(c+2)/sin2C=c/sinC

Daqui, tiramos cosC=(c+2)/2c. Agora eu vou logo na Lei dos Cossenos:

c^2=a^2+b^2-2abcosC
c^2=(c+2)^2+(c+1)^2-2(c+1)(c+2)(c+2)/2c
(c+1)(c+2)^2=c(c^2+6c+5)
(Que bom que o termo em c^3 vai cortar, entao teremos uma quadratica
em c. Vai sair!)
5c^2+8c+4=6c^2+5c
c^2-3c+4=0
c=4 ou c=-1

Entao c=4, b=5 e a=6 eh a unica solucao possivel... (Para multipla
escolha, tah bom aqui, mas para o problema ficar correto mesmo, agora
a gente teria que checar se, no triangulo de lados 4, 5 e 6, o angulo
A eh o dobro de C mesmo; mas dah para calcular cosC=3/4 e cosA=1/8,
isto eh, dah para ver que cosA=cos2C, donde sai que A=2C).

Abraco,
Ralph


2008/3/26 arkon [EMAIL PROTECTED]:



 Pessoal alguém pode me enviar, por favor, a resolução dessa questão



 (EN-84) As medidas dos lados de um triângulo ABC são 3 números inteiros e
 consecutivos e o ângulo maior A é o dobro do menor C. Os lados deste
 triângulo são:



 a) 2, 3 e 4.b) 3, 4 e 5.c) 8, 9 e 10.d) 4, 5 e 6.e) 5, 6 e
 7.



 DESDE JÁ MUITO OBRIGADO

=
Instruções para entrar na lista, sair da lista e usar a lista em
http://www.mat.puc-rio.br/~obmlistas/obm-l.html
=


Re: [obm-l] Livros de Geomtria Analitica

2008-03-25 Por tôpico Ralph Teixeira
Para responder o PS, o Elon escreveu sim um livro Geometria Analitica
e Algebra Linear, publicado pela Colecao Matematica Universitaria do
IMPA, cuja intencao era exatamente ensinar Geometria Analitica usando
bastante vetores para alunos no comeco da Graduacao, com um olho num
proximo segundo curso mais pesado de Algebra Linear. Ele estah aqui
neste link:

http://www.impa.br/opencms/pt/publicacoes/colecao_matematica_universitaria/livro_geometria_analitica_e_algebra_Linear/index.html

Voce tem que gostar do estilo do Elon para aprecia-lo (na minha
modesta opiniao, isto significa: correto, elegante e sucinto; mas para
ler voce tem que se concentrar e destrinchar devagar cada pagina com
papel e caneta do lado o tempo todo). Bom, eu gosto do estilo do Elon,
mas minha opiniao eh parcial, jah que eu estudava com ele jah ha mais
de 20 anos atras... Vale lembrar que o livro eh bem baratinho, entao
custa pouco experimentar e ver... :)

Abraco,
   Ralph

2008/3/25 Daniel Madeira [EMAIL PROTECTED]:
 Caros amigos da lista,

Estou estudando na universidade a disciplina: Geometria Analítica. Estive
 navegando na internet em busca de livros para e comprar e estudar essa
 disciplina. Em particular me interessa um livro que aborde problemas sobre
 parte de vetores como semi-retas (a parte geometrica), a parte referente a
 vetores Linearmente depedentes e independentes e exercícios de demonstração
 envolvendo propriedades de operações com vetores. Estive vendo alguns
 livros. O que acham desses livros ?

 - Introdução à geomtria analítica no espaço
   autor: Paulo Boulos e Ivan de Camargo
   edt: Makron Books

 - Álgebra Linear e geometria vectorial
  autor: Rios, Sixto
  edit: Litexa

 - Vetores e Geometria Analitica, teoria e exercicios
  autor: Ana Célia da Costa Loreto e Armando
  editora: LCTE

 - Vetores e uma iniciação à geometria análitica
  autor: Dorival A. de Mello e Renate G. Watanabe
 editora: Penn

 - Exercícios de geometria analitica (vale a pena esse livro ?)
  autor: Blasi
 editora: Papirus

 Já tenho os livros: álgebra linear e aplicações do callioli, geometria
 analítica - um tratamento vetorial do Paulo Boulos e geometria analítica do
 Steinbruch.

   Peço desculpas pelo e-mail extenso. Certo da compreensão de vocês e de sua
 ajuda, agradeço desde já.

  Cordialmente,

Daniel.

 ps:  algum livro do elon lages envolvendo essa parte de geometria analitica
 ? ele vale a pena ?



 
 Abra sua conta no Yahoo! Mail, o único sem limite de espaço para
 armazenamento!



=
Instruções para entrar na lista, sair da lista e usar a lista em
http://www.mat.puc-rio.br/~obmlistas/obm-l.html
=


Re: [obm-l] Informativo sobre evento.

2008-03-24 Por tôpico Ralph Teixeira
Ah, muito obrigado pela divulgacao. Melhor ainda, o endereco

http://www.uff.br/semanadamatematica/

poe um menuzinho simpatico do lado esquerdo para facilitar a
navegacao. Destaque para o cartaz, que NAO EH UM FILME.

Abraco,
 Ralph (um dos membros da comissao organizadora)

2008/3/24 Olimpiada Brasileira de Matematica [EMAIL PROTECTED]:
 Caros(as) amigos(as) da OBM,

 A seguir enviamos, informações sobre a IV Semana da Matemática da UFF.
 outras informações e o programa de atividades no site:
 http://www.uff.br/semanadamatematica/home.html

 Cordialmente,

 Secretaria da OBM.
 ==
 IV Semana de Matemática da UFF
 13 a 17 de maio de 2008
 Niterói http://www.niteroi.com.br/turismo.htm - Rio de Janeiro

 *APRESENTAÇÃO
 *
 A IV Semana de Matemática da UFF será realizada no Campus do Valonguinho
 da Universidade Federal Fluminense, no período de 13 a 17 de maio de
 2008. O objetivo principal é o de se criar um ambiente onde
 pesquisadores, educadores e a comunidade em geral possam interagir,
 divulgando experiências, inovações e diagnosticando novas áreas de
 atuação em Matemática nas suas várias manifestações.
 Público-alvo: alunos e professores do curso de matemática da UFF, alunos
 e professores das escolas de ensino fundamental e médio da região.
 Atividades previstas: palestras palestras.php, minicursos e oficinas
 minicursos.php, mesa redonda mes-01.php, grupos de pesquisa,
 comunicações e o museu interativo para o ensino da matemática museu.html.
 Atividades culturais: sessões de cinema.

 Secretária Regina Simplício de Souza
 Telefone: (021) 2629-2005
 FAX: (021) 2629-2114
 E-mail: [EMAIL PROTECTED] mailto:[EMAIL PROTECTED]
 Horário de atendimento: das 14 h às 18 h.

 Endereço:
 Universidade Federal Fluminense
 Instituto de Matemática
 Rua Mário Santos Braga, S/N, Campus do Valonguinho
 CEP 24020-140, Niterói, RJ, Brasil

 *COMISSÃO ORGANIZADORA
 *Dirce Uesu Pesco (Coordenadora),
 Ana Maria Lima de Farias (Vice-Coordenadora),
 Anne Michelle Dysman Gomes, Gabriel Calsamiglia Mendlewicz, Humberto
 José Bortolossi,
 José Roberto Linhares de Mattos, Mario Olivero Marques da Silva, Ralph
 Costa Teixeira,
 Solimá Gomes Pimentel, Wanderley Moura Rezende.








 

 =
 Instruções para entrar na lista, sair da lista e usar a lista em
 http://www.mat.puc-rio.br/~obmlistas/obm-l.html
 =


=
Instruções para entrar na lista, sair da lista e usar a lista em
http://www.mat.puc-rio.br/~obmlistas/obm-l.html
=


Re: [obm-l] QUESTÃO ANTIGA

2008-03-19 Por tôpico Ralph Teixeira
Acho mais acessível resolver esta assim: não há mal algum em supor que
abc, já que eles são diferentes entre si.

Agora, não pode ser a=1 (pois 1/b+1/c=0 com b,c0 não dá). Também não
pode ser a=3, pois então 1/a+1/b+1/c  1/3+1/3+1/3 =1.

Conclusão: a=2.

Agora, fica 1/b+1/c=1/2. Por um raciocínio análogo, não pode ser b=4,
pois aí teríamos 1/b+1/c  1/4+1/4=1/2. Como ba, só pode ser b=3.

Agora jogue a=2 e b=3 na equação, e descubra que c=6.

Abraço,
  Ralph

On Tue, Mar 18, 2008 at 4:50 PM, gugolplexj [EMAIL PROTECTED] wrote:

 Olá,

 Creio q esse problema já transitou por aqui há algum tempo.

 Quais os números naturais a, b, c diferentes entre sí, tais que
 1/a +1/b + 1/c = 1?

 Grato,
 Jorge.

=
Instruções para entrar na lista, sair da lista e usar a lista em
http://www.mat.puc-rio.br/~obmlistas/obm-l.html
=


Re: [obm-l] Solido

2008-03-17 Por tôpico Ralph Teixeira
Nos limites para a integral em x, aquele x=9-y^2 está misterioso,
acho que é ali o problema...

Bom, vamos lá. Se a gente realmente quer dx dy, temos que encontrar
a projeção do sólido no plano xy. Eu fiz um desenho aqui com um certo
cuidado, e me parece que esta projeção é a região entre x=1+y^2 (pois
a curva de interseção entre o cilindro parabólico z=1-y^2 e o plano
z=2-x se projeta em xOy sobre 2-x=1-y^2, isto é, x=1+y^2) e x=2. O
resto, eu concordo contigo. Então a integral correta seria:

Int (y=-1 a y=1) Int (x=1+y^2 a x=2) (1-y^2)-(2-x) dx dy

Vejamos... conta... conta deu 8/15, confere com o gabarito que você tem.

Abraço,
Ralph

P.S.: Eu acho que eu prefiro fazer esta aí usando dy dz... Deixa eu
ver... Ficaria
Int (y=-1 a y=1) Int (z=0 a z=1-y^2) 2-(2-z) dz dy. A vantagem disto é
que a gente não precisa dissecar aquela interseção entre x+z=2 e
z=1-y^2, mas eu concordo que não está MUITO mais fácil não.

2008/3/15 Klaus Ferraz [EMAIL PROTECTED]:

 Determine o volume do solido limitado pelas superficies z=1-y^2 , x+z=2 e
 x=2 e para z=0.
 v=8/15.
 Eu só queria que montasse a integral dupla. Porque a que estou achando tah
 dando errado.
 Estou achando $_(-1,1)$(-y^2+9,y^2+1)[1-y^2 - (2-x)]dxdy. Não sei qual meu
 erro.

 Grato.

 
 Abra sua conta no Yahoo! Mail, o único sem limite de espaço para
 armazenamento!

=
Instruções para entrar na lista, sair da lista e usar a lista em
http://www.mat.puc-rio.br/~obmlistas/obm-l.html
=


Re: [obm-l] Teoria dos Números

2008-03-09 Por tôpico Ralph Teixeira
p^1994+p^1995=p^1994(p+1)

Como p^1994 jah eh um quadrado perfeito (de p^997), a condicao pedida
eh equivalente a p+1 ser quadrado perfeito. Mas entao:

p+1=k^2 (com k inteiro)
p=k^2-1=(k+1)(k-1)

Mas se p eh primo, como eh que vai ser o produto de dois inteiros? O
unico jeito eh se um deles for 1 e o outro for p; como nao pode ser
k+1=1 (pois entao p=0, o que nao serve), tem de ser

k-1=1, entao k=2, entao p=3.

Assim, ha apenas um numero primo satisfazendo a dita condicao, que eh
p=3. De fato:

3^1994+3^1995=3^1994.4=(3^997.2)^2.

Abraco,
Ralph

On Sun, Mar 9, 2008 at 7:01 PM, Pedro Júnior
[EMAIL PROTECTED] wrote:
 Determine a quantidade de números primos p, para que a expressão p^1994 +
 p^1995 seja um quadrado perfeito.
 Desde já muito agradecido.
 Pedro Jr


=
Instruções para entrar na lista, sair da lista e usar a lista em
http://www.mat.puc-rio.br/~obmlistas/obm-l.html
=


Re: [obm-l] Derivada errada?

2008-03-03 Por tôpico Ralph Teixeira
O problema desta demonstração é que ela não prova que h é derivável. A
Regra do Produto diz que:

SE h e g forem diferenciáveis num ponto x=a, então hg também é e (hg)'=h'g+hg'

Então, quando você passa de f=hg para f'=h'g+hg', você está USANDO que
h é derivável, fato que, teoricamente, ainda não está provado.

Em outras palavras, o que a gente demonstrou com este argumento (que
eu acho bacana!) foi apenas:

Sejam f e g duas funções diferenciáveis no ponto x=a; seja
h(x)=f(x)/g(x). **SE** h(x) for diferenciável em x=a, então
h'=(f'g-fg')/g^2 neste ponto.

Abraço,
 Ralph

2008/3/1 Rhilbert Rivera [EMAIL PROTECTED]:


 Um famoso livro de cálculo, demonstra a regra da derivada do quociente da
 seguinte maneira:



 Sejam f e g  duas funções e seja h = f/g, definida onde g diferente de zero.
 Então f = hg, aplicando a regra do produto á função f, temos que:

 f'  = h'g + hg'

 Daí, obtemos

 h'=(f' –hg')g.

 Substituindo o valor de h nesta última expressão, vem:

   h'= (f'g – fg')/g^2



 Um outro livro diz que  essa demonstração está errada, mas não diz onde.

 Ágüem poderia me dá uma dica, porque esse procedimento não demonstra a regra
 do quociente?
 Obrigado!
 
 Encontre o que você procura com mais eficiência! Instale já a Barra de
 Ferramentas com Windows Desktop Search! É GRÁTIS!

=
Instruções para entrar na lista, sair da lista e usar a lista em
http://www.mat.puc-rio.br/~obmlistas/obm-l.html
=


Re: [obm-l] VARETAS

2008-02-29 Por tôpico Ralph Teixeira
1) De cada uma de três varetas de comprimento L quebra-se um pedaço.
Calcule a probabilidade de que com esses três pedaços, seja possível
se construir um triângulo.

Bom, o problema nao explicita como a vareta eh quebrada, mas acho
razoavel supor que a distribuicao de probabilidade de cada pedaco eh
uniforme em [0,L] (em outras palavras, vou pressupor que, para cada
pedaco, a probabilidade deste pedaco ter tamanho menor que a eh a/L).
Tambem eh super razoavel supor que as 3 quebras sao independentes umas
das outras.

Neste caso, voce pode identificar o processo de quebrar as 3 varetas
com a escolha de um ponto (x,y,z) no cubo [0,L]x[0,L]x[0,L] (isto eh,
vertice na origem, faces paralelas aos eixos, outro vertice em
(L,L,L)). De acordo com esta identficacao (e mediante as hipoteses que
eu fiz acima), a probabilidade de voce escolher um ponto que esteja
dentro de um solido S contido no cubo eh Volume(S)/L^3.

Deixa eu esclarecer esta afirmacao com um exemplo. Suponha que voce
quer a probabilidade de termos xa e yb e zc para a,b,c fixos entre
0 e L. Bom, estas 3 equacoes definem um paralelepipedozinho de volume
abc no espaco x-y-z, entao teriamos Pr(xa e yb e zc)=abc/L^3.

No caso do problema, queremos xy+z, yx+z e zx+y. Desenhe isso -- eh
o solido dentro do cubo e dentro da superficie composta por estes 3
planos -- analiticamente, eh o tetraedro regular de vertices
(0,0,0),(L,L,0),(L,0,L),(0,L,L) UNIAO com o tetraedro retangulo de
vertices (L,L,0),(L,0,L),(0,L,L),(L,L,L). Na figura anexa, eh o
tetraedro colorido MAIS o tetraedro transparente no canto superior
direito da figura.

Agora eh soh achar o volume deste solido por analitica, geometria ou
sei lah. O volume do tetraedro colorido eh (Lraiz(2))^3 raiz(2)/12 =
L^3/3, enquanto o do cantinho superior eh L^3/6. Somando e dividindo
por L^3, voce acha a resposta, que eh
1/2=50%.

2) Quebra-se uma vareta em três pedaços. Calcule a probabilidade de
que se possa formar um triângulo com esses pedaços.

Esta jah apareceu aqui na lista... Deixa eu ver Aqui:
http://www.mat.puc-rio.br/~nicolau/olimp/obm-l.200706/msg00182.html

Note que a solucao que eu escrevi lah faz uma hipotese de que voce
escolhe os pontos de quebra ao acaso e independentemente, depois
quebra a vareta. Nao creio que eh isso que uma pessoa normalmente faz
quando quebra uma vareta em 3 pedacos; alias, quando uma pessoa quebra
uma vareta em *dois* pedacos, nao costuma ser com a probabilidade do
problema anterior tampouco -- a gente tende a quebrar mais perto do
meio, raramente nas pontas.

Abraco,
Ralph
attachment: Menor.png

Re: [obm-l] desigualdade

2008-02-01 Por tôpico Ralph Teixeira
Vejamos Lagrange:

Caso i) Grad(x^2+y^2+z^2)=0 dah x=y=z=0 que nao serve.

Caso ii) Grad(x^3+y^3+z^3-3xyz)=(3a/2).grad(x^2+y^2+z^2)
(Chamei a constante lambda de 3a/2 para facilitar o que vem a seguir)

O sistema eh:
i) x^2-yz=ax
ii) y^2-xz=ay
iii) z^2-xy=az
iv) x^3+y^3+z^3-3xyz=1

(Se x=0, vem yz=0. Suponha y=0, e vem z^3=1, isto eh, (x,y,z)=(0,0,1) com
a=1 eh solucao.
Analogamente, temos as possibilidades (0,1,0) e (1,0,0) para (x,y,z).
Qualquer uma delas satisfaz a restricao e dah x^2+y^2+z^2=1... serah que
este eh o minimo?)

Agora, meu professor Secco me fez decorar a fatoracao:
v) x^3+y^3+z^3-3xyz=(x+y+z)(x^2+y^2+z^2-xy-xz-yz)=S.(T-DP)
onde S=x+y+z; T=x^2+y^2+z^2 e DP=xy+xz+yz.

Entao temos:
vi) 1=S(T-DP)
(i)+(ii)+(iii)=vii): T-DP=aS
x(i)+y(ii)+z(iii)=viii): 1=aT
Identidade ix): S^2=T+2DP

Estas 4 equacoes sao mais faceis de resolver:
vii) em vi) dah 1=aS^2 (em particular, a,S0)
com viii) 1=aS^2=aT, entao S^2=T e, por (ix),  DP=0
Isto em vii): entao T=aS
Isto em aS^2=aT dah a=S, entao (como aS^2=1) a=S=1 e, enfim, T=1.

Em suma:
i) Como x^2+y^2+z^2=0 e a superficie x^3+y+3+z^3-3xyz=1 eh fechada, tem de
haver um minimo;
ii) Por Lagrange (funcoes suaves) se houver algum minimo, ele deve
satisfazer T=1;

Ou seja, achamos o minimo T=1, mesmo sem ter resolvido completamente o
sistema.
De quebra, encontramos no chute tres pontos onde o minimo eh alcancado (pode
haver outros).

Abraco,
  Ralph

P.S.: E note o problema com aquele sistema; T=1 eh uma esfera e S=1 eh um
plano. Entao S=T=1 eh um circulo (onde automaticamente DP=0).
Pela identidade v), em TODOS os pontos deste circulo tem-se
x^3+y^3+z^3-3xyz=S(T-DP)=1, ou seja, o sistema de Lagrange tem uma
infinidade de solucoes!
Assim, a esfera T=1 e a superficie x^3+y^3+z^3-3xyz=1 se tangenciam com um
CIRCULO em comum no espaco... Deixa eu fazer uma figura... Ok, veja GIF
anexo!

2008/2/1 Klaus Ferraz [EMAIL PROTECTED]:

  Ache o minimo de x^2+y^2+z^2, onde x,y,z pertence a R e
 x^3+y^3+z^3-3xyz=1

 Alguem conhece alguma desigualdade que encaixa ai? Eu tentei usar os
 multiplicadores de lagrange mas caiu em um sistema que num consegui resolver
 não.
 vlw.

 --
 Abra sua conta no Yahoo! 
 Mailhttp://br.rd.yahoo.com/mail/taglines/mail/*http://br.mail.yahoo.com/,
 o único sem limite de espaço para armazenamento!



<    1   2   3   4   5   6   7   8   >